Está en la página 1de 194

Textbooks: Cecil Essentials of Medicine, Hospital Medicine Secrets, First Aid for Medical Clerkship Introductory Guide: Primer

to the Internal Medicine Clerkship 2nd Edition by Picchioni --------------------------------------------------------------------------------------------------Common Problems in Internal Medicine Cardiovascular: Acute Coronary Syndromes, Congestive Heart Failure, Valvular Heart Disease, Atrial Fibrillation and Anticoagulation, Hypertension Endocrine: Diabetes Mellitus, Hypothyroidism, Hyperthyroidism, Osteoporosis, Disorders of Calcium Metabolism Hematology: Anemia, Coagulopathies Gastro: Hepatitis, Peptic Ulcer Disease, Gastroesophageal Reflux Disease, Diarrhea and Constipation Oncology: Hematological Malignancy, General Care of the Cancer Patient, Management of Pain Nephrology: Electrolyte Disturbances, Acid-Base Disorders, Acute and Chronic Renal Failure Rheumatology: Rheumatoid Arthritis, Osteoarthritis, Monoarthritides, Polyarthritides Pulmonary: DVT and Pulmonary Embolism, Chronic Bronchitis and Asthma, Emphysema Infectious: Fever of Unknown Origin, Acquired Immune Deficiency Syndrome, Pneumonia, Urinary Tract Infection, Cellulitis, Subacute Bacterial Endocarditis Allergy: Urticaria Neurology: Cerebrovascular Disease, Headache, Dementia and Coma Dermatology: Dermatological Manifestations of Chronic Medical Disease --------------------------------------------------------------------------------------------------Procedures: NEJM Videos In Clinical Medicine: http://www.nejm.org/multimedia/videosinclinicalmedicine --------------------------------------------------------------------------------------------------Kaplan Videos (2001) Board Studying Thoughts from Dr. Conrad Fischer, MD * For USMLE Step 2 (and any exam for that matter), think from the point of view of the question writer. Questions you can expect include Which of the following is the most likely diagnosis? seem commonly in Internal Medicine sections, where physicians ponder over the diagnosis for hours. Diagnosis questions account for about 20-30%, more in Internal Medicine, less on Surgery. Which of the following is the best initial diagnostic test? or What would you do first to confirm your diagnosis? Also, What is the best diagnostic test or What is the most accurate diagnostic test? Treatment questions follow Which of the following is the best initial therapy? Always ask What is the next best step in the management of this patient?

* What is the problem? Pay attention to the question. If the question asks what the best initial diagnostic test is and you pick the best test (most accurate), you missed the question. If the question asks what is the best initial therapy and you choose the best therapy (most effective), you missed the question. * But you think to yourself, no no no I dont do this. I wont make this mistake. But, you do make this mistake because everybody makes this mistake. Examination results show this time and time again. * Answer questions based on what the question-writer is saying, now what you think they mean. * Youre sitting there looking up at that board exam and all you see is a huge exam. You feel scared and all you can think about is how big that exam is. But, if you could find a way to look past the exam and see good and beauty. You could connect with the goodness and beauty that is beyond the exam, which you cant do right now because all you see is a giant exam. But, if you can connect with the knowledge, the data that you need for the exam and connect it with the beauty and good, great things will happen. * First, the exam will seem much smaller and lighter in the palm of your hands (from William Blake). Second, you will be able to remember the knowledge longer and help someone later on with it, and that is a great good. Third, you will get a better grade anyway and get what you want. If doing well on the exam is your highest aspiration, it will become so painful. Fourth, your trip/voyage/journey will be filled with much more joy. This idea is the analgesic for your studying. --------------------------------------------------------------------------------------------------How To Succeed In Clerkship First Aid For The Medicine Clerkship (Stead, Stead, & Kaufman) Be On Time: Team rounds usually begin between 7am and 8am. Give yourself at least 10 minutes per patient for pre-rounding to learn about events that occurred overnight or lab/imaging results. Dress In A Professional Manner: Regardless of what the attending wears. A short white coat should be worn over your professional dress clothes unless it is discouraged (e.g. pediatrics). Act In A Pleasant Manner: The medical rotation is often difficult, stressful, and tiring. Smooth out your experience by being nice to be around. Smile a lot and learn everyones name. Dont be afraid to ask how your residents weekend was. If you do not under- stand or disagree with a treatment plan or diagnosis, do not challenge. Instead, say Im sorry, I dont quite understand, could you please explain Show kindness and compassion toward your patients. Never participate in callous talk about patients. Take Responsibility: Know everything there is to know about your patients: their history, test results, details about their medical problem, and prognosis. Keep your intern or resident informed of new developments that they might not be aware of, and ask them for any updates you might not be aware of. Assist the team in developing a plan; speak to radiology, consultants, and family. Never give bad news

to patients or family members without the assistance of your supervising resident or attending. Respect Patients Rights: 1) All patients have the right to have their personal medical information kept private. This means do not discuss the patients information with family members without that patients consent, and do not discuss any patient in hallways, elevators, or cafeterias. 2) All patients have the right to refuse treatment. This means they can refuse treatment by a specific individual (you, the medical student) or of a specific type (no nasogastric tube). Patients can even refuse life- saving treatment. The only exceptions to this rule are if the patient is deemed to not have the capacity to make decisions or understand situations, in which case a health care proxy should be sought, or if the patient is suicidal or homicidal. 3) All patients should be informed of the right to seek advanced directives on admission. Often, this is done by the admissions staff, in a booklet. If your patient is chronically ill or has a life-threatening illness, address the subject of advanced directives with the assistance of your attending. More Tips: Volunteer, be a team player, be honest, and keep patient information handy. Present In An Organized Manner: This is a [age]-year-old [gender] with a history of [major history such as HTN, DM, coronary artery disease, CA, etc.] who presented on [date] with [major symptoms, such as cough, fever, and chills] and was found to have [working diagnosis]. [Tests done] showed [results]. Yesterday, the patient [state important changes, new plan, new tests, new medications]. This morning the patient feels [state the patients words], and the physical exam is significant for [state major findings]. Plan is [state plan]. --------------------------------------------------------------------------------------------------Presenting A Chest Radiograph (CXR): 1) Technique: Rotation, anteroposterior (AP) or posteroanterior (PA), penetration, inspiratory effort. 2) Bony structures: Look for rib, clavicle, scapula, and sternum fractures. 3) Airway: Look for tracheal deviation, pneumothorax, and pneumomediastinum. 4) Pleural space: Look for fluid collections, which can represent hemothorax, chylothorax, and pleural effusion. 5) Lung parenchyma: Look for infiltrates and consolidations: These can represent pneumonia, pulmonary contusions, hematoma, or aspiration. The location of an infiltrate can provide a clue to the location of pneumonia: * Obscured right (R) costophrenic angle = Right lower lobe * Obscured left (L) costophrenic angle = Left lower lobe * Obscured R heart border = Right middle lobe * Obscured L heart border = Left upper lobe 6) Mediastinum: Look at size of mediastinuma widened one (> 8 cm) goes with aortic dissection. Look for enlarged cardiac silhouette (> 12 thoracic width at base of

heart), which may represent congestive heart failure (CHF), cardiomyopathy, or pericardial effusion. 7) Diaphragm: Look for free air under the right hemidiaphragm (suggests perforation). Look for stomach, bowel, or nasogastric tube (NGT) above diaphragm (suggests diaphragmatic rupture). 8) Tubes and lines: * Identify all tubes and lines. * An endotracheal tube should be 2cm above the carina. Common mistake is right bronchus intubation. * A chest tube (and proximal hole) should be in the pleural space (not in the lung parenchyma). * An NGT should be in the stomach and uncoiled. * The tip of a central venous catheter should be in the superior vena cava (not in the right atrium). * The tip of a SwanGanz catheter should be in the pulmonary artery. * The tip of a transvenous pacemaker should be in the right atrium. --------------------------------------------------------------------------------------------------Presenting A Chest Radiograph (Mnemonic Method): Mnemonic: RRR, RIP, ABCDEFGH * Right: patient, procedure, date * Rotation: spinous processes are to line up vertically, equal space between clavicles * Inspiration: should show 8 ribs * Penetration: spinous processes should just be visible through the vertebrae * Airway: carina and tracheal deviation * Bones: look at clavicles, vertebrae, scapula, and ribs for fractures * Cardiac silhouette: > 1/2 total chest width could be CHF, determine if edges are clear * Diaphragm: elevated or depressed, right should be higher, no air under diaphragm * Effusions: check borders and edges for fluid levels, hemothorax, atelectasis, pneumothorax * Fields: infiltrates, masses, objects, size (large in emphysema, small in chronic bronchitis) * Gadgets: ET tubes, central lines, chest tubes, pacemakers, ECG monitors, mention this after RRR RIP in ICU * Hilum: any masses or disturbances --------------------------------------------------------------------------------------------------Presenting An Electrocardiogram (ECG): 1) Rate: The rate is [number of] beats per minute (bpm): * The ECG paper is scored so that one big box is 0.20 seconds. These big boxes consist of five little boxes, each of which is 0.04 seconds. * A quick way to calculate rate when the rhythm is regular is the mantra: 300, 150, 100, 75, 60, 50 (= 300 / # large boxes), which is measured as the number of large boxes between two QRS complexes. Therefore, a distance of one large box between

two adjacent QRS complexes would be a rate of 300, while a distance of five large boxes between two adjacent QRS complexes would be a rate of 60. * For irregular rhythms, count the number of complexes that occur in a 6-second interval (30 large boxes) and multiply by 10 to get a rate in bpm. 2) Rhythm: The rhythm is [sinus]/[atrial fibrillation]/[atrial flutter] or other: * If p waves are present in all leads and upright in leads I and aVF, then the rhythm is sinus. Lack of p waves suggests a disorganized atrial rhythm, a junctional rhythm, or a ventricular rhythm. A ventricular rhythm (V Fib or V Tach) is an unstable one (could spell imminent death), and you should be getting ready for advanced cardiac life support (ACLS). * Normal sinus rhythm is usually a regular narrow-complex rhythm with each QRS complex preceded by a p wave. 3) Axis: The axis is [normal]/[deviated to the right]/[deviated to the left]: * If I and aVF are both upright or positive, then the axis is normal. * If I is upright and aVF is upside down, then there is left axis deviation (LAD). * If I is upside down and aVF is upright, then there is right axis deviation (RAD). * If I and aVF are both upside down or negative, then there is extreme RAD. 4) Intervals: The [PR]/[QRS] intervals are [normal]/[shortened]/[widened]: * Normal PR interval = 0.12 to 0.20 seconds: * Short PR is associated with WolffParkinsonWhite syndrome (WPW). * WPW syndrome is characterized by a delta wave, or slurred up-stroke of QRS complex. * Long PR interval is associated with heart block of which there are three types: * First-degree block: PR interval > 0.20 seconds (one big box) * Second-degree (Mobitz type I or Wenckebach) block: PR interval lengthens progressively until a QRS is dropped. * Second-degree (Mobitz type II) block: PR interval is constant, but one QRS is dropped at a fixed interval. * Third-degree heart block: Complete AV dissociation Normal QRS interval 0.12 seconds: * Prolonged QRS is seen when the beat is initiated in the ventricle rather than the sinoatrial node, when there is a bundle branch block, and when the heart is artificially paced with longer QRS intervals. Prolonged QRS is also noted in tricyclic overdose and WolfeParkinsonWhite syndrome. 5) Wave morphology: A. Ventricular hypertrophy: There [is/is no] [left/right] [ventricular/atrial] hypertrophy: * There are multiple criteria for determining right (RVH) and left ventricular hypertrophy (LVH). Clues for LVH: * RI>15mm, RI,II or aVF >20mm, RaVL>11mm, RV5 or RV6 >26mm, RI +SIII >25mm, R+S in Vlead>45mm, SV1 +RV5 or RV6 >35mm Clues for RVH: * RV1>7mm, SV1<2mm, R/S ratio inV1 >1, RAD of 110deg or more

B. Atrial hypertrophy: * Right atrial hypertrophy: tall or peaked p waves in limb or precordial leads * Left atrial hypertrophy: broad or notched p waves in limb leads C. Ischemic changes: There [are/are no] S-T wave [depressions/elevations] or [flattened/inverted] T waves. Presence of Q wave indicates an old infarct. D. Bundle branch block: There [is/is no] [left/right] bundle branch block. Clues: * Presence of RSR wave in leads V1-V3 with ST depression and T wave inversion goes with RBBB. * Presence of notched R wave in leads I, aVL, and V4-V6 goes with LBBB. --------------------------------------------------------------------------------------------------Top 100 Secrets Medical Secrets (4th, Zollo) 1) The treatment of severe sepsis syndrome should be based on efficient resuscitation, effective antimicrobial therapy, elimination of secondary infections, euglycemia, early targeted and specific drug therapy, and establishment of therapeutic goals. 2) Acute pulmonary embolism (PE) is a difficult diagnosis to establish despite newer advances in imaging; approximately 50% of cases are diagnosed post mortem. 3) In the approach to suspected PE, keep in mind the prudent use of key diagnostic tests: (1) rapid d-dimer by ELISA is an effective screening test; (2) chest CT can help detect most PEs; and (3) a negative Doppler venous ultrasound of the legs does not exclude the diagnosis of PE. 4) The most common etiologic agent implicated in acute bacterial meningitis in the U.S. is Streptococcus pneumoniae. 5) In the newly diagnosed HIV patient, in addition to routine adult immunizations, immunizations against pneumococcal pneumonia, influenza, and both hepatitis A and B are indicated. 6) Metabolic syndrome is diagnosed on the basis of abdominal obesity, hypertriglyceridemia, low HDL cholesterol levels, hypertension, and fasting hyperglycemia. 7) Pituitary tumors cause problems for patients by two main mechanisms: mass effect, which applies pressure to surrounding structures, and endocrine hyperfunction, which results in excessive secretion of a particular anterior pituitary hormone. 8) A key concept in evaluating patients with hyperfunctioning endocrine tumors is that biochemical diagnosis should always precede anatomic localization. 9) The best initial screening test for evaluation of thyroid status is the TSH, since it is the most sensitive measure of thyroid function in the majority of patients. The one exception is patients with pituitary/hypothalamic dysfunction, in whom TSH cannot reliably to assess thyroid function. 10) The most common presentation of hypogonadism is erectile dysfunction and decreased libido in men and amenorrhea and infertility in women. 11) All patients with coronary artery disease (CAD), CAD-equivalent diseases, or diabetes should be treated aggressively to reach the LDL-cholesterol target of 100 mg/dL.

12) Diabetics and patients with vascular disease should be treated with a statin lipidlowering drug to prevent heart disease and stroke, regardless of the blood lowdensity lipoprotein (LDL) cholesterol level, age (from 40 to 79 years), or gender. 13) The goal blood pressure is < 130/80 mmHg in hypertensive subjects with diabetes mellitus and/or chronic kidney disease. 14) The single most life-saving treatment strategy in patients with acute ST-elevation myocardial infarction is to rapidly achieve complete reperfusion of the infarct-related artery by mechanical (balloon angioplasty or stenting) or pharmacologic means (thrombolysis). 15) Angiotensin-converting enzyme inhibitors (or angiotensin receptor blockers) and beta-adrenergic blockers are effective in reducing cardiovascular complications and improving survival in patients with systolic heart failure and are recommended in all patients with no contraindications to these drugs. 16) Noninvasive stress testing has the best predictive value for detecting CAD in patients with an intermediate (30-80%) pretest likelihood of CAD and is of limited value in patients with very low (< 30%) or very high (> 80%) likelihood of CAD. 17) In patients with Coccidioides immitis infections, higher titers of complementfixing antibodies suggest more extensive disease, and rising titers suggest worsening disease. 18) Patients who present with flaccid paralysis during the summer months should be evaluated for West Nile virus infection. 19) A febrile patient with rash who presents to the emergency department during May to September in the South Atlantic and West South Central states should receive empirical doxycycline therapy for suspected Rocky Mountain spotted fever. 20) Community-acquired methicillin-resistant Staphylococcus aureus that is susceptible to clindamycin but resistant to erythromycin should not be treated with clindamycin because of the possibility for induction of resistance. 21) In patients with disseminated candidiasis, IV catheters should be removed and ophthalmologic examinations performed to evaluate for the presence of retinal disease. 22) Transmission of Borrelia burgdorferi (the causative agent of Lyme disease) from an infected Ixodes tick to a susceptible human requires the tick to have fed on the human for at least 40 hours. 23) Porcelain gallbladder is an incidental finding, more common in women who have gallstones. Because up to 50% of patients develop gallbladder carcinoma, prophylactic cholecystectomy is recommended. 24) Three liters of Coca-Cola administered via nasogastric lavage over a 12-hour period can dissolve gastric bezoars. It is thought that the cola acidifies the gastric contents and liberates carbon dioxide in the stomach, resulting in the disintegration of phytobezoars. 25) Regardless of what is done, GI bleeding stops spontaneously in about 80% of patients. 26) Patients with hereditary nonpolyposis colorectal cancer syndrome have a higherthan-average risk of developing colon and gastric cancer.

27) About 90% of patients with primary sclerosing cholangitis have underlying ulcerative colitis, but less than 10% of all patients with ulcerative colitis have primary sclerosing cholangitis. 28) In patients with suspected perforation, the minimum amount of free air that can be detected on an upright chest x-ray is 12mL. 29) The three major openings in the diaphragm through which hernias may occur are the esophageal hiatus (most common), foramen of Bochdalex (3-5%, usually leftsided), and foramen of Morgagni (rare). 30) In a patient who has a malignancy involving the right hilum, look at the hand veins. If the veins in the hands are distended and do not collapse when the arms are lifted over the head, there is a high chance of superior vena cava obstruction. 31) In high-risk patients, the chance of developing breast cancer can be reduced by about 50% with the use of tamoxifen. 32) If a patient with lung cancer presents with hoarseness, look for vocal cord paralysis, a sign of mediastinal involvement (recurrent laryngeal nerve) that renders the patient inoperable. 33) Patients with head and neck cancer have a 30% chance of developing another cancer somewhere in the aerodigestive tract (head and neck, lung, or esophagus), especially if they continue to smoke and drink. 34) If a patient presents with hypercalcemia, look for a squamous cell cancer (lung, esophagus, head and neck, cervix, anus). 35) Up to 15% of breast cancers may not be detectable by mammogram. If the patient has a clinically suspicious lump, perform a biopsy. 36) The presence of bilateral small kidneys in a patient with azotemia confirms chronic renal failure. 37) In a diabetic patient with proteinuria, the presence of concomitant retinal disease suggests strongly (90% correlation) that the renal manifestations are due to diabetes. 38) Treatment of anemia of chronic renal failure by recombinant human erythropoietin is highly effective, but correction of iron deficiency and iron supplementation by oral or intravenous route is simpler, cheaper, and often by itself effective therapy. 39) In resistant hypertension, especially in younger (< 20 yr) or older (> 70 yr) patients, consider and rule out renovascular hypertension. 40) New onset of nephrotic proteinuria in an elderly patient warrants exclusion of an underlying malignancy. 41) The principal mechanism of bicarbonate reabsorption in the proximal tubule is through Na+-H+ exchanger (NHE3) activity. 42) D-lactic acidosis is characterized by increased serum anion gap, metabolic acidosis, and episodic encephalopathy in patients with short bowel syndrome. 43) Ethylne glycol (antifreeze) toxicity is characterized by high anion gap metabolic acidosis, neurotoxicity in the form of ataxia, seizures, and calcium oxalate crystals in the urine.

44) Bartter's syndrome is a disorder associated with normotensive hyperaldosteronism, secondary to juxtaglomerular hyperplasia, hypokalemic metabolic alkalosis, and severe renal potassium wasting. 45) Hyperkalemia is an important side effect of both ACE inhibitors and ARBs, but the problem is less frequent and smaller in magnitude with ARBs because of their less pronounced effects on aldosterone levels. 46) Hypochromic microcytic anemias are the most encountered anemias in hospitalized and ambulatory patients. 47) Both iron-deficiency anemia and anemia of chronic disease have low transferrin saturation. In iron-deficiency anemia, the TIBC is often increased, whereas anemia of chronic disease is marked by an unusually low TIBC. 48) The main clinical manifestations of sickle hemoglobinopathies are hemolytic anemia, chronic end-organ damage, periodic vaso-occlusive disease ("crises"), and hyposplenism. 49) The triad of thrombocytopenia, fragmentation hemolysis, and fluctuating neurologic signs suggests thrombotic thrombocytopenic purpura (TTP), perhaps the most spectacular of the fragmentation syndromes. 50) The cytogenetic marker of chronic myelogenous leukemia is the 9:22 translocation, in which portions of the long arms of chromosomes 9 and 22 are exchanged, resulting in a shortened 22 or Philadelphia chromosome (Ph1). Some patients with acute lymphoblastic leukemia (ALL) also have 9:22 translocations (poor prognostic marker). 51) The classic cell seen in the lymph nodes of patients with Hodgkin's disease is the Reed-Sternberg (RS) cell, a large cell with two nuclei, each possessing a distinct nucleolus. 52) Secondary monoclonal gammopathy must be distinguished from the monoclonal gammopathy associated with multiple myeloma, benign monoclonal gammopathy of uncertain significance, solitary plasmacytoma, amyloidosis, lymphoma, and Waldenstrm's macroglobulinemia. 53) Deep venous thrombosis in a young person, a family history of thrombosis, thrombosis at unusual sites (such as the mesenteric vein), or recurrent thrombosis without precipitating factors suggests a hypercoagulable state. 54) Any condition that leads to V/Q mismatching can cause hypoxemia. Most pulmonary disorders are associated with some degree of V/Q mismatching. This is the most common cause of hypoxemia and is responsive to oxygen therapy. 55) Assuming that you are at sea level and breathing room air, an easy way to calculate the A-a difference is as follows: (150-40/0.8) - PaO2 measured by ABG. 56) Although the anterior segment of the upper lobes may be affected by TB, a lesion found only in the anterior segment suggests a diagnosis other than TB (e.g., malignancy). 57) Incidence of lung cancer now exceeds breast cancer in women. Women develop lung cancer at an earlier age and after fewer years of smoking. 58) Pleural fluid glucose < 30 mg/dL and pH < 7.30 suggest rheumatoid effusion, TB, lupus, or malignancy.

59) Mesothelioma, a pleural malignancy associated with asbestosis exposure, is not associated with tobacco use. 60) Early, aggressive intervention with disease-modifying antirheumatic drugs reduces the morbidity (deformity leading to reduced functionality and disability) and mortality associated with rheumatoid arthritis. 61) Antinuclear antibody (ANA) titers are not associated with activity of disease. 62) COX2 NSAIDs are no more efficacious than older standard NSAIDs but are significantly less toxic. 63) A patient with low positive rheumatoid factor (RF) and arthralgia should be checked for hepatitis C, which can produce a low-grade synovitis and cryoglobulins (which in turn can produce a falsely positive RF). 64) Always check for Sjgren's antibodies (SSA/SSB) and phospholipid antibodies in a young woman with lupus before conception. Sjgren's antibodies increase the risk of neonatal lupus (rash, thrombocytopenia, heart block), and phospholipid antibodies can significantly increase the risk for miscarriage, premature labor, or intrauterine growth delay. 65) Packed red cells in freshly acquired blood may include lymphocytes that can mount a graft-versus-host reaction if the patient's own immune system is unable to rapidly kill and inactivate these transfused allogeneic leukocytes. 66) Intranasal steroids are the single most effective drug for treatment of allergic rhinitis. Decongestion with topical adrenergic agents may be needed initially to allow corticosteroids access to the deeper nasal mucosa. 67) The clinical manifestations of anaphylaxis include flushing, sense of foreboding, urticaria or angioedema, pruritus, hoarseness, stridor, bronchospasm, hypotension, tachycardia, nausea, vomiting, abdominal pain, diarrhea, headache, and syncope. 68) ACE inhibitors are often-forgotten causes of angioedema and chronic cough. 69) Chronic urticaria may require treatment with a combination of both H1 and H2 antihistamines, reflecting the distribution of these receptors in the skin. Work-up for an allergic etiology is rarely informative. 70) Beta blockers should be avoided whenever possible in patients with asthma because they may accentuate the severity of anaphylaxis, prolong its cardiovascular and pulmonary manifestations, and greatly decrease the effectiveness of epinephrine and albuterol in reversing the life-threatening manifestations of anaphylaxis. 71) HIV infection is preventable and treatable but never curable. 72) If you are thinking of mononucleosis as a diagnosis, think about and test for HIV. 73) Adherence to anti-HIV therapy must be > 95% for a durable response. HIV treatment guidelines change frequently - always verify your information. 74) A person under care for HIV should not develop pneumocyotic pneumonia (PCP). It is entirely preventable. 75) There is a critical interaction between HIV and tuberculosis. If one infection is present, look for the other. 76) If you have diagnosed one sexually transmitted disease (STD), you must consider others, especially HIV. 77) Most back pain is not caused by a radiculopathy.

78) The most common cause of dizziness is benign paroxysmal positional vertigo. 79) The leading causes of death after a stroke are medical complications, not the stroke itself. 80) Heparin has no value in the acute treatment of strokes. 81) The sudden onset of a severe headache may indicate an intracranial hemorrhage. 82) Coma is usually caused by medical problems, not neurologic ones. 83) Elective surgery should be postponed for further evaluation if the patient has signs or symptoms of unstable or inadequately treated chronic disease. 84) Patients who have undergone coronary revascularization within 5 years of a proposed elective surgery and have no signs or symptoms of recurrent ischemia can usually undergo surgery without further evaluation. 85) Acute dyspnea in a patient who has had major surgery should raise the suspicion of pulmonary embolism, even if the patient has received prophylaxis. 86) All patients who take oral agents for diabetes may continue them until the day of surgery unless they have chronic liver or renal disease or are on a first-generation sulfonylurea. In these cases the oral agent should be held at least several days in advance of the surgery. 87) Pacemakers and implanted cardioverters/defibrillators should be assessed both before and after surgery, radiation therapy, or lithotripsy. 88) Surgery patients on any antiplatelet agent should be told when to stop the medication before surgery and when to resume it afterward to minimize perioperative bleeding. 89) Strict bed rest is not needed for the treatment of acute lumbosacral strain. 90) Influenza virus vaccination reduces hospitalization and death from influenza and its complications in elderly and high-risk patients. 91) Always examine the feet and pedal pulses of diabetic patients regularly, looking for ulcerations, injury, or reduced blood flow. 92) Closely monitor patients with blood pressure measurements defined as "prehypertension," and encourage lifestyle changes to prevent progression to hypertension. 93) Reduce the risk of hip fracture in elderly and high-risk patients with calcium and vitamin D supplements, exercise prescription, hip pads, and medications to treat osteoporosis, when indicated. 94) Assess a woman's risk of coronary disease, stroke, thromboembolism, and breast cancer before prescribing estrogen/progesterone therapy in menopause. 95) Older adults currently constitute the fastest-growing population in the United States - a trend that is expected to continue for the foreseeable future. 96) Commonly used instruments for a comprehensive geriatric assessment include the Mini Mental State Exam, the Geriatric Depression Scale, activities of daily living, instrumental activities of daily living, and assessment of stability and mobility (e.g., Tinnetti or "Get Up and Go" test). 97) Dementia and short-term memory loss are not caused by aging. 98) Delirium carries tremendous mortality and morbidity rates and should be identified, worked up aggressively, and treated as any medical emergency.

99) Diastolic dysfunction, as distinct from systolic dysfunction, results from impaired relaxation in heart failure with preserved ejection fraction and may account for half of all cases of heart failure in people over 80. Although the symptoms of diastolic and systolic dysfunction may be similar, the traditional therapy for systolic dysfunction can actually worsen ventricular filling and increase the risk of orthostasis and syncope in cases of diastolic dysfunction. 100) Fifteen percent of elderly patients who fall and fracture a hip report prior falls. It is essential to ask about falls, assess for fall risk, and then act accordingly, given the significant mortality and morbidity of hip fractures. --------------------------------------------------------------------------------------------------Kaplan Videos (2001) Neurology with Dr. Jacob Levy, MD --------------------------------------------------------------------------------------------------Spinal Cord Compression * A 61yo AAM is brought to the ED complaining of back pain that started gradually three days ago. He describes the pain as band-like around the abdomen without radiation. His past medical history is significant for prostate cancer diagnosed three years ago. First thing to think about is if this is an emergency or not an emergency; meaning do we need to intervene right now or can we treat with something like analgesics and follow-up. * The prostate cancer is important because metastatic disease to the spine could be compressing the spinal cord. Other worrisome cancers could be breast, lung, multiple myeloma, lymphoma. Other worrisome signs with back pain would be fever, urinary incontinence, urinary retention, fecal incontinence, sexual dysfunction in males, bilateral lower extremity weakness. So just the history of cancer with back pain means we should be evaluating this acutely. If were suspecting spinal cord compression, we should see upper motor neuron lesion signs below the level of the compression. Signs would include hyperreflexia, increased tone, positive Babinski sign, spastic paralysis. * Patients with spinal cord compression who are unable to lift up their limbs against gravity at the time of compression have a 5% or less chance of being able to ambulate after their episode of compression. If the compression is caught early enough when the patient is able to ambulate, their ability to ambulate after proper treatment is about 80%. * Most likely diagnosis is spinal cord compression. Next step in the management of this patient is give dexamethasone, not MRI of the spine, not x-ray, not bone scan. Consider therapy before diagnostics in management when the patient has an emergency. Best initial test is a spinal x-ray, sensitive maybe 80% of the time. Most accurate test is MRI of the spine. * Radiation therapy and surgical decompression are generally left for after the diagnosis is made. Say you gave dexamethasone and diagnosed, now there is an abscess or hematoma compressing the spinal cord, then do surgical decompression. Say it is lymphoma or cancer compressing the spinal cord, then do radiation. ---------------------------------------------------------------------------------------------------

Syringomyelia, Vitamin B12 Deficiency & Anterior Spinal Artery Infarction * A 25yo man comes to the Emergency Department status post motor vehicle accident. The ED physician has addressed the airway, breathing, circulation, and calls you to evaluate the patient. You perform a thorough neurological exam, finding (motor) lower extremity weakness 4/5 bilaterally with some hyperreflexia, (CN) cranial nerves intact, (sensory) pain/temperature lost on lower extremities, vibration/position intact, and light touch intact. * Most likely diagnosis is syringomyelia, with a pool of fluid developing in the spinal cord. Pain and temperature lost because these spinal cord tracts are centrally located (spinothalamic), while vibration/position and light touch are in posterior column. * Best initial test for syringomyelia is an MRI. Most sensitive test is an MRI. Treatment is often surgical. If there is an anatomic problem in the spinal cord, there should be an anatomic solution. * Vitamin B12 deficiency causes subacute combined degeneration of the cord. Characteristic signs of vitamin B12 deficiency are loss or proprioception and vibration with intact pain/temperature, usually with ataxia and pyramidal signs like hyperreflexia and positive Babinski. * Anterior spinal artery feeds the sensory neurons involved in pain and temperature. So an anterior spinal artery infarct would have lost pain/temperature sense, sudden onset of flaccid paresis, and the presence of intact proprioception/vibration sense. * Patient presents with signs of cord compression. An MRI shows osteomyelitis compressing the anterior cord. Best test is MRI, first initial step is give dexamethasone. Best initial test is x-ray, even if theyre showing you MRI. --------------------------------------------------------------------------------------------------Cerebrovascular Accidents (CVA) * 56yo woman is brought to the ED by her daughter, complaining of sudden onset of right upper extremity weakness that started while she was watching television in the morning. Her daughter became concerned when her mother was unable to talk in response to her questions. On neurologic exam, you note right upper extremity weakness with pronator drift and right facial palsy. When you question the patient, she seems to understand what is being said but she cannot clearly respond. * Most likely diagnosis is cerebrovascular accident (CVA). Pathophysiology is lack of blood supply to section of the brain. Could be an embolus coming from the atrium in a patient with atrial fibrillation. It could be a bleed in a patient with hypertension. It could also be thrombosis, where the clot forms in the brain itself (not an embolus from a distant site). Hypotension can also induce ischemia, such as in a watershed infarct. * Focal neurologic deficit of sudden onset is very likely to be a stroke. * Risk factors for cerebrovascular disease are the same as coronary artery disease, peripheral vascular disease, or carotid artery disease. General atherosclerosis risk factors include smoking, high cholesterol, diabetes, obesity, male gender, hypertension, increased age, HIV, and alcohol abuse.

* Important risk factors include atrial fibrillation, recent transient ischemic attack (TIA) with similar symptoms, valvular disease (especially mitral stenosis). * There is an anterior circulation and a posterior circulation to the brain. Anterior is the middle cerebral artery (MCA) and anterior cerebral artery (ACA). Posterior is vertebral arteries forming the basilar artery and the posterior cerebral artery (PCA). * Patients with MCA infarcts present with contralateral hemiparesis involving the face and the arm more extensively than the lower extremity. So, patient would have facial droop and weak upper extremity. * Patients with ACA infarcts presents with contralateral hemiparesis involving the leg more than the face/arm. Mnemonic is put your best foot forward for foot/leg affected with forward/anterior artery. ACA ischemia may involve urinary incontinence and personality changes (Phineas Gage). * Homunculus, little man on your brain, has to sleep comfortably at night. So he puts is feet up and lays back. Anterior circulation on interior (leg) and middle circulation on outside (arm/face). * MCA infarct also comes with aphasia. Broca aphasia is expressive aphasia with intact understanding. Broca is broken speech. Wernicke aphasia is wordy, nonsensical speech, unable to understand. Aphasia occurs with dominant hemisphere stroke. Most people are left-hemisphere dominant. * Basilar artery provides blood supply to cerebellum, pons, and brainstem. Brainstem (medulla/pons/midbrain) is where cranial nerves are. So, patient could have sudden onset of diplopia, blurry vision, dysphagia, focal cranial nerve palsies. Cerebellar signs include ataxia and vertigo. * With posterior circulation (basilar artery), deficit is contralateral if above decussation and ipsilateral if below. A cross syndrome is a cranial nerve deficit on one side, and a motor deficit (hemiparesis, ataxia) on the opposite side. Cross syndrome is most likely posterior circulation stroke. * A cranial nerve III deficit (cannot adduct eye) on one side and hemiparesis on the other is Weber syndrome. * A cranial nerve III deficit on one side and ataxia on the other is Benedikt syndrome. * A sensory loss on one side of the face with contralateral sensory loss on the body is Wallenberg syndrome. * PCA supplies occipital lobe, giving us the ability to see. PCA ischemia comes with hallucinations, visual loss. * Patient has arm/face hemiparesis and Broca aphasia. Most likely diagnosis is MCA infarct (left side). Next step in management is head CT scan, not aspirin, not clopidogrel, not ticlopidine, not heparin, not transthoracic echo, not transesophageal echo, not tPA. Now, do we give contrast or no contrast? We are trying to distinguish between a bleed and ischemia, because these two have different management. Answer is head CT without contrast. We are looking for the absence of blood. If there is no bleeding on the non-contrast CT, we assume ischemic stroke. * Most sensitive (accurate) test for diagnosing ischemic stroke is MRI. MRI most sensitive for posterior fossa lesions. CT scan without IV contrast is most sensitive test for hemorrhagic stroke.

* With ischemic stroke, look for reversible risk factors such as cardiac thrombosis (echocardiogram), carotid artery stenosis (carotid artery duplex, mainly for anterior circulation stroke), atrial fibrillation (24h Holter monitor). * Hemorrhagic stroke is managed by supportive care and consulting neurosurgery (poor prognosis). For ischemic stroke, we can think about giving aspirin, clopidogrel, heparin, tPA, ticlopidine. * If patient has acute onset focal neurologic deficit, CT shows no bleeding, you are sure it is ischemic stroke, and symptom onset is within 3 hours, then answer is tPA/PLAT (tissue plasminogen activator). * If patient was sleeping and woke up with symptoms, tPA should not be given even if you get the CT minutes later. * Must document time of onset and CT scan with diagnosis before giving tPA. If you give tPA past the 3-hour window, youre only increasing their risk of bleeding. So the risk to benefit ratio is not favorable after 3 hours. * Contraindications for tPA therapy are bleeding (e.g. GI bleed, urinary tract bleed) within past 21 days, surgery within past 14 days, BP > 185/110, platelet count < 100,000, PT > 15sec, ischemic stroke or head trauma within 3 months, intracranial bleed ever in the past. * Aspirin is used in ischemic stroke for secondary prevention. The benefit of aspirin in acute stroke is not as good as with acute myocardial infarction, but we give it anyway. * Clopidogrel is used if the patient has failed aspirin therapy, meaning the patient is on aspirin after an ischemic stroke then has another ischemic stroke. Clopidogrel side effects include TTP and neutropenia. * Ticlopidine is never the answer due to side effect, which are TTP and neutropenia. The difference between clopidogrel and ticlopidine is that ticlopidine causes the side effects (TTP, neutropenia) more often. * If the patient is allergic to aspirin, failed aspirin therapy, or is allergic to clopidogrel, add dipyridamole. * Heparin reduces the rate of recurrent CVA. For every stroke you prevent, you cause one complication (e.g. bleed). So the only time you give heparin is when you have a higher risk of CVA. There is a higher risk of recurrent CVA if the patient has ischemic stroke with atrial fibrillation, basilar artery thrombosis, or stroke in evolution. * Stoke in evolution is when symptoms are getting worse, a loss of brain function occurs with brain cell death. * Say you do a workup on the 56yo lady that has right-sided facial/arm hemiparesis. Carotid duplex finds 80% stenosis of the right internal carotid artery. Answer is not surgery. Indication for carotid endarterectomy (CEA) is >70% stenosis with symptoms. Symptoms mean you have to be able to blame a TIA or a stroke on the stenosis. If the patients symptoms were due to the carotid artery stenosis, the stenosis would be on the left side (not the right). * When looking at a CT scan to evaluate a suspected CVA, youre looking for the presence of white material in the parenchyma of the brain. In ischemic stroke, the tissue will look darker than the surrounding tissue.

--------------------------------------------------------------------------------------------------Seizures * 29yo man is brought to the ED by ambulance after his mother found him convulsing in his bedroom. The patients mother said that her son was unable to respond to her frantic cries during the convulsion and describes jerking motions that become more frequent then stopped after about 1 minute. The mother says he was tired and lethargic for 20 minutes after the episode. She then called the ambulance to bring her son to the hospital. * What is the most likely diagnosis? Answer is seizure. Differentiate seizure from syncope, say due to arrhythmia. This patient was having convulsions (tonic-clonic movements). Patients who syncopize can have tonic/clonic movements. Urinary or bower incontinence is an important part of the history, but not necessarily specific. Bite marks on the tongue may imply seizure, but not specific either. The most specific thing on history for seizure is the post-ictal state. Patients who have syncope and come out will not be lethargic, tired, or with achy muscles. Syncope has rapid recovery within minutes or seconds after unconsciousness. Here, the patient was tired/achy for 20 minutes. So this is the clear differentiation. * Urinary and bowel incontinence plus bite marks are seen more commonly in seizure than syncope. However, the post-ictal state still is the most specific symptom. * A seizure is defined as random firing of neurons in the brain. A seizure is considered a complaint, like a patient coming in with chest pain; there is a large differential diagnosis for seizure. * Ask yourself if there is an underlying cause for the seizure. Ask if the patient has a history of epilepsy. Do not assume that any seizing patient has a diagnosis of epilepsy. * Seizure differential mnemonic: VITAMINS. * Seizure: vascular (stroke, bleed, AVM), infection (encephalitis, meningitis), trauma, autoimmune (vasculitis, SLE), metabolic (electrolytes, glucose, drugs), idiopathic, neoplastic (metastatic cancer, primary tumors), s for Psi or psychiatric (patient faking). * If youre thinking vascular disease, look for risk factors and sudden onset of neurologic focality. Suspect infection if the patient has seizure with fever, or nuchal rigidity, or photophobia. Suspect autoimmune if history involves a rash, purpura, low-grade fever with weight loss, arthritis, SLE stigmata, positive ANA. For metabolic, look at things like low sodium, high sodium, low calcium, low oxygen, low glucose, low magnesium. If a young woman with breast cancer has a seizure, think about metastatic to the brain. * The next step in management is ABCs: airway, breathing, and circulation. Then benzodiazepine. * 29yo man comes in with new-onset seizure witnessed by his mother, convulsing with a long post-ictal state, and the patient is already intubated. Blood pressure and circulation are intact. Patient is continuing to seize. What is the best initial treatment now? Answer is give lorazepam or diazepam, for acutely seizing patient.

* Always look for a secondary cause. If you identify a secondary cause, treat it, hoping seizures resolve. So if patient comes in seizing with sodium of 106, treatment would be focused on hypertonic saline. If patient came in hypoxemic and seizing, treatment would be focused on intubation and high-flow oxygen (after ABCs). * If patient is continuing to seize without regaining consciousness between, this is status epilepticus. Management is lorazepam or diazepam. * Say you give the benzo and the patient is still seizing. Then what do you give? Answer is phenytoin or fosphenytoin. Note, giving phenytoin IV is not recommended due to lack of solubility and resultant precipitation, give fosphenytoin instead. * What if patient continues to seize after giving phenytoin or fosphenytoin? Give phenobarbital. * Next step after phenobarbital? Answer is midazolam and propofol (anesthesia). * Seizure Meds: ABC, lorazepam/diazepam, phenytoin/fosphenytoin, phenobarbital, and last midazolam/propofol. * Seizures are categorized into partial, generalized, and complex vs. simple. Categories help determine what medication to give. A complex partial seizure is treated differently from a complex generalized seizure. * A partial seizure only affects one part of the brain. So patient might have shaking/jerking of the hand, or the foot, or the leg. A generalized seizure affects the entire cortex. Either the seizure starts as generalized, or a partial seizure turns into a generalized seizure. A complex seizure implies loss of consciousness. A simple seizure has no loss of consciousness. An atonic seizure means lack of tone, so patient has drop attacks. An absence seizure is the opposite of an atonic seizure, so patient keeps postural tone but brain shuts down so the patient is not paying attention (no consciousness) and likely just blinking. Myoclonic seizures involve muscle jerks. * What is the best test to identify abnormal neural activity that predisposes to a seizure? Answer is electroencephalogram (EEG). So patient has first time seizure with no exact etiology, order an EEG. * What if EEG is negative, when do you start treating an idiopathic seizure? Answer is with recurrent seizures. * So patient has normal neurologic exam, negative EEG, negative family history, and one seizure. We follow the patient. Once the patient has another seizure, we start treatment. If the patient has positive family history, we would start right away at the first seizure. * Medication for partial seizure (even one that becomes generalized) is carbamazepine or phenytoin. If carbamazepine or phenytoin are not answers, pick valproic acid. * Medication for generalized seizure is valproic acid or lamotrigine. * Medication for absence seizure is ethosuximide. * Medication for atonic or myoclonic seizure is valproic acid. * Medication for unidentifiable seizure is valproic acid, as it is the most widely effective seizure medication. ---------------------------------------------------------------------------------------------------

Parkinson Disease * A 56yo man is brought in by his wife for evaluation of a resting tremor that she noticed recently. She also states that her husband has been moving very slowly as of late. When questioned, the patient states that he feels fine and does not know why his wife is dragging him from doctor to doctor. His past medical history is significant for mild hypertension treated with a thiazide diuretic. Physical exam finds a resting tremor noted in his right hand, and when walking the patient is stooped forward making small steps. You note cogwheel rigidity in his right upper extremity with a positive Myerson sign (patient unable to resist blinking with glabellar tapping). * Differential diagnosis for tremor includes Parkinson disease, essential tremor, and cerebellar disease. In Parkinson disease, the tremor occurs at rest and resolves with movement. In cerebellar disease, the tremor occurs primarily with movement (intention tremor). Exam for cerebellar disease would include finger-to-nose and heel-to-shin tests, with the patient have a compromised ability to reach the object. In Parkinson disease, the patient will have pill rolling tremor at rest. In essential tremor, the tremor occurs and worsens with movement, there is a family history usually, and there are no other stigmata of cerebellar or Parkinson disease. * So in the 56yo man with resting tremor, slow movement (bradykinesia, paucity of movement), history of falls (postural instability), and cogwheel rigidity (arm feels like cogs on a wheel, moving in distinct steps), what is the most likely diagnosis? Answer is Parkinson disease. So, diagnosis of Parkinsonism is clinical. * Normally when you walk and turn it is a smooth movement. With postural instability, patient turns in wide circle. * Causes of Parkinsonism are drugs (antipsychotics, metoclopramide, MPTP: an unintentional byproduct of the recreational drug MPPP), tumor, bleed or stroke, CO poisoning, cobalt, manganese. So ask yourself about secondary causes. Is this patient diabetic and taking metoclopramide for autonomic neuropathy with gastric paresis? * Most cases of Parkinson disease are idiopathic, meaning we cannot identify a cause. * Parkinsonism is defined as death of dopaminergic cells in the substantia nigra. * The drugs that cause Parkinson are antipsychotics, which are dopamine antagonists. You can decrease the amount of dopamine by antagonizing it (decreasing it directly) or by increasing acetylcholine. Acetylcholine acts to inhibit dopaminergic tone in the brain. So treatment is to either give dopamine to provide what is lost, or to take away acetylcholine so the dopamine in the brain can work more effectively. * Dopamine agonists are bromocriptine, carbidopa/levodopa, pergolide, pramipexole, and ropinirole. Carbidopa inhibits the conversion of levodopa to dopamine in the periphery, so that the levodopa can reach the brain and be converted to dopamine there; thus we can give less levodopa and get the same effect. * Selegiline (MAO-B inhibitor), COMT inhibitors, and amantadine increase dopamine amount/effect also. * Acetylcholine blocking with trihexyphenidyl primarily.

* So how do you choose what to answer as the best initial treatment for Parkinson disease? First question to ask your self is what is the functional status of the patient? Meaning, how do they function on a day-by-day basis? * Functional patients get amantadine or anticholinergic (e.g. trihexyphenidyl). * Non-functional patients get carbidopa/levodopa, the most effective treatment for Parkinson disease. We dont give carbidopa/levodopa to functional patients because it has the most serious side effects. * Always ask how much function you are getting from the treatment at the expense of the side effects. * Side effects are psychosis, hypotension, and acute GI upset. After long-term therapy on carbidopa/levodopa, patients get response fluctuations. Response fluctuations are the on/off phenomenon, akinesia (restlessness), and dyskinesia (abnormal movements). On/off phenomenon is quite distressing to patients. * Say patient is functional and just has a tremor, pick amantadine if patient is > 65yo. Pick trihexyphenidyl if patient is < 60yo. We avoid anticholinergic medications in elderly patients because they can become confused, have urinary retention, dry mouth, dry eyes, more so than younger patients. * Answer dopamine agonists (e.g. pramipexole, ropinirole) to treat response fluctuations for patients that are taking carbidopa/levodopa. You can also give a COMT inhibitor or selegiline. * If patient is on carbidopa/levodopa and it isnt enough, add the dopamine agonist. * The most preferred dopamine agonists are the newer ones, like ropinirole (not selegiline or COMT inhibitor). * Say patient has stigmata of Parkinson and you are asked what medication is thought to arrest the progression of Parkinson disease, the answer is selegiline. --------------------------------------------------------------------------------------------------Huntington Disease * 34yo man comes to your clinic for evaluation of strange spontaneous movements that have been occurring lately. Recently while sitting at a family dinner, the patient experienced uncontrolled grimacing with grunting. His family history is significant for his father who died at age 41 of dementia. * Most likely diagnosis is Huntington disease. Know that patients will present with abnormal movement (chorea), abnormal behavior, loss of inhibition, changes in personality, and a family history of a similar thing happening to a first-degree relative (Huntington is autosomal dominant). * Clinical diagnosis is presence of chorea with personality changes, usually in a patient 30-40yo with a positive family history. True diagnosis done with genetic analysis, looking for chromosome 4p CAG repeat. * Treatment is supportive care; there is no formal treatment. --------------------------------------------------------------------------------------------------Multiple Sclerosis (MS) * A 32yo woman comes to the ED with numbness and tingling in her right hand. She states that her symptoms began several days before admission and have progressively worsened over the last several hours. When asked, she states that three

years ago she had an episode of seeing double that lasted two days that resolved on its own. Physical exam is significant for hyperreactive reflexes bilaterally in the lower extremities. You also note increased spasticity in her lower extremities. * Most likely diagnosis is multiple sclerosis (MS). * Differential includes MS, CVA, brain tumor, cervical spinal disease, and carpal tunnel syndrome. * CVA less likely because she is 32yo with no other listed risk factors. * Brain tumor less likely as there is no specific tumor location giving lower extremity weakness along with diplopia. * Cervical spinal disease or carpal tunnel less likely because they do not explain the diplopia. * The essential point for clinically suspecting MS is a patient with multiple neurologic deficits that are separated by space (anatomically) and by time (temporally). * Testing includes MRI with gadolinium, CSF for oligoclonal banding, and olfactory/visual evoked potentials looking for abnormal transmission. * What is the most sensitive/accurate test? Answer is MRI or brain and spine, 8595% sensitive. Looking for multiple lesions and paraventricular lesions consistent with MS. * The best initial test is also MRI, not CT scan, not lumbar puncture. * If MRI is inconclusive or equivocal for the clinically suspected MS diagnosis, then get the lumbar puncture. * Olfactory/visual evoked potentials are not used much today with the development of MRI. * Relapsing/remitting disease is a form of MS where patient goes through waxing/waning episodes of symptoms. * Relapsing/remitting disease can become progressive (secondary progression). Primary progressive disease has worsening MS symptoms right from presentation, least common, and worse prognosis. * Treatment for acute exacerbation of MS is IV high-dose steroids with a 4-week taper on oral prednisone. Steroids do not slow down the progression of the disease, but they help symptomatically for the disease relapse. * Medications useful to arrest the progression of MS are interferon beta 1a, interferon beta 1b, or glatiramer acetate. None of these three is better than the other. They have been shown to help with relapsing/remitting forms, but not in primary progression. No medication has been shown to help with primary progressive disease. * Treatment for spasticity in MS is baclofen. * Treatment for fatigue in MS is amantadine. * Treatment for urinary incontinence in MS is oxybutynin. * Treatment for urinary retention in MS is bethanecol. --------------------------------------------------------------------------------------------------Dementia * A 67yo woman is brought to your clinic complaining of forgetfulness. She states that recently she has been forgetting telephone numbers and cannot remember the name of her mailman who she has known for 25 years. Her past medical history is

significant for hypertension, coronary artery disease, and high cholesterol. Physical exam is unremarkable. * With memory loss, consider dementia and ask yourself if there is a reversible cause. * Reversible causes: hypothyroidism (check TSH), vitamin B12 deficiency (check B12 level), frontal lobe neoplasm and chronic subdural hematoma (check for focality on exam or trauma history), syphilis (check RPR: rapid plasma reagin), uremia and cirrhosis (check creatinine, LFTs, physical exam), central or obstructive sleep apnea (look for obesity, speak with spouse), HIV. * Non-reversible causes: Creutzfeldt-Jakob disease and prion disease (check for myoclonus, rapid dementia course usually weeks to months), Lewy body disease (delirium-like course, waxing/waning), multi-infarct dementia (stepwise progression, tie temporally dementia onset with CVA, difficult to differentiate from Alzheimer). * Most common cause of dementia is Alzheimer disease. In Alzheimer disease, there is initial memory loss that becomes chronic and gradual with relative preservation of social function and personality until late in the disease. * Pick disease (frontal lobe degeneration) has personality changes initially, such as agitation, violence. * Dementia is generally memory loss plus some other deficit in cognitive function, such as concentration, praxis, or executive function. This is where the mini-mental status exam (MMSE) is important. * Patient presents with memory loss. Perform MMSE to find dementia. Then rule out or treat reversible causes. Then rule out non-reversible causes like Pick (personality changes first), CJD (myoclonus, rapid progression), multiinfarct dementia (temporal tie to CVA), and Lewy body (delirium-like). Now, we diagnose Alzheimer disease. * Treatment for Alzheimer disease was tacrine, but no longer used as the drug of choice due to liver toxicity. * Medication of choice for Alzheimer now is donepezil. Do not pick tacrine for first line. --------------------------------------------------------------------------------------------------Vertigo * A 53yo woman is brought to the ED complaining of dizziness. The patient describes walking to her bathroom and experiencing a sudden feeling of nausea. She managed to reach the bathroom where she vomited once and fell to the floor a second time. She was unable to get up off the floor and called 911. She describes the feeling of the room spinning around her even though she realizes she is not moving. * Most important question is to identify what the patient means by dizziness. Dizzy is very nonspecific and does not have much significance medically. Does the patient mean they are having vertigo or do they mean pre-syncope? * Pre-syncope associated symptoms are feeling like going to black out, light headed, palpitations, chest pain, shortness of breath. This implies cardiac disease. * Vertigo is the sensation of movement in the absence of movement. Patient may say the room is spinning around me, the earth is rolling in front of my feet, like they are falling forward.

* Differentiating between vertigo and pre-syncope is a clinical determination. * Next question after determining vertigo is if it is central (in the CNS, e.g. MS, posterior fossa tumor, medication) or peripheral (ear semicircular cannals). For central, we do MRI, for peripheral, we do symptomatic treatment. * Central vertigo is usually chronic, neighborhood signs (cranial nerve deficits via brainstem), pure nystagmus (in one direction, usually vertical), nystagmus nonsuppressible with fixation, multi-directional nystagmus. * Peripheral vertigo is usually acute, hearing loss, tinnitus, mixed nystagmus (usually horizontal but with another component), nystagmus suppresses with fixation, unidirectional nystagmus. * Posterior fossa imaged more effectively with MRI than with CT scan. * Mnire disease is a triad of (peripheral) vertigo, hearing loss, and tinnitus unrelated to head movement. Treatment of choice is diuretics, low salt diet, and surgical decompression if medical management fails. Disease is thought to be caused by swelling in the semicircular canals. * Benign positional paroxysmal vertigo (BPPV) is peripheral vertigo of sudden onset related to movement of the head. Patient may say I move my head then 5-10 seconds later I get dizzy. Physical exam can reproduce symptoms via Dix-Hallpike test (head movement testing). Etiology is thought to be an otolith in the semicircular canals. Treatment of choice is movement exercises to try and move the otolith out of the canal. * Labyrinthitis is peripheral vertigo that occurs when vertigo follows an upper respiratory tract infection. Treatment is meclizine (antihistamine) or diazepam in severe cases. * So, if you find central vertigo signs on physical exam, do MRI imaging of the posterior fossa. If you identify peripheral vertigo, seek Meniere, BPPV, or labyrinthitis with their treatments listed. * If you cannot identify a disease, give meclizine initially then diazepam secondary. --------------------------------------------------------------------------------------------------Headache * A 32yo woman comes into the office complaining of a headache that started two days ago. She locates her headache to the right side of her head and describes it as pulsating and throbbing in quality. The headache is worsened by walking up stairs or around the block. She admits to nausea but denies vomiting. She also states that loud noise and bright light exacerbate her pain. On exam, there is no focality on physical exam, she has had these headaches a few times before. Most likely diagnosis is a migraine. * Ask yourself, what is the likelihood that this headache is secondary to some serious underlying pathology? Is this a brain tumor? Meningitis? Subarachnoid bleed? Do we need to diagnose and intervene immediately? * History/physical implying serious underlying cause: first time headache with severe pain, fever, progressively worsening headache, symptoms aggravated by cough, worse in the early morning, aggravated by valsalva maneuver, nuchal rigidity, worst headache of my life (subarachnoid), thunderclap headache (time to peak

pain was seconds), focality on CNS exam, headache occurs after vomiting, any headache starting at age > 55yo. * Primary headache disorders include migraine, cluster, and tension. * Migraines are associated with triggers, such as eating a specific food (e.g. chocolate), sleeping too little, stress, or during menstruation. A trigger is very specific for migraine. * Pulsatile headaches occur in cluster and migraine headaches (both vascular). * Photophobia and phonophobia (ligyrophobia) classically occurs in migraine but may occur in cluster also. * Cluster headaches are usually unilateral (like migraine), no trigger, has rhinorrhea, red eye, possibly Horner syndrome (ptosis, miosis, anhidrosis). * Tension headaches are triggered by stress but not foods or emotion, are bilateral, vice-like, radiating to neck or back of the head. * Migraines peak within 4-72 hours from onset of pain (gradual onset). Cluster headaches peak usually within 5 minutes (quick onset) and only last 45-90 minutes. * Clusters happen 2-3 times per day over a 4-8 week period; occurring in clusters. * Management for tension headache is analgesics, like acetaminophen or NSAIDs. * Management for cluster headaches starts with 100% oxygen. Next line is sumatriptan. Prophylaxis for cluster headaches includes lithium, prednisone, or verapamil. * Management for mild migraine headache (no nausea or vomiting), give NSAIDs. If migraine is moderate to severe, abortive therapy is sumatriptan or ergot alkaloid. Prophylaxis for migraines is indicated when patient has more than 3 headaches per month. Prophylaxis is propranolol, timolol, or methysergide (generally not used because it causes retroperitoneal fibrosis). * Never give sumatriptan to a patient with a history of coronary artery disease. Migraines are generally caused by vasodilation and sumatriptan constricts the vessels. So if a patient has a 70% left main coronary lesion and you give them sumatriptan, you just gave them an MI. Do not give it even if you have high suspicion of CAD. --------------------------------------------------------------------------------------------------Acute Inflammatory Demyelinating Polyradiculoneuropathy (AIDP) * A 46yo man is brought to your office complaining of rubbery legs. The patient states that his symptoms began two days ago. Approximately three weeks ago the patient states that he experienced several episodes of diarrhea that resolved spontaneously. On neurological exam, the patient is noted to have bilaterally lower extremity weakness with loss of reflexes. * Most likely diagnosis is Guillain-Barr syndrome, an AIDP. CIDP (chronic) has a slower course, seen in HIV. * The distribution of weakness is important when differentiating between GuillainBarr, myasthenia gravis (MG), and botulism. In Guillain-Barr, distribution starts in distal areas and moves proximal. Paresthesias (tingling) and autonomic instability (labile blood pressure, sweating) are also seen.

* Campylobacter jejuni infection (gastroenteritis) associated with Guillain-Barr syndrome (GBS). * Hyporeflexia seen in any peripheral neuropathy. * The best initial test for the diagnosis of GBS is lumbar puncture, looking at CSF for high protein that is not accompanied by pleocytosis (a high number of cells). So youre looking for high protein and few cells. * The best test (most accurate) for GBS is an electromyogram (EMG). * Best initial treatment for GBS is intravenous immunoglobulin. You can also do plasmapheresis. IVIG and plasmapheresis are equivalent in their effect. Do not give prednisone or systemic steroids in the treatment of acute Guillain-Barr syndrome. IV Ig is easier to give than arranging for plasmapheresis, so IV Ig best initial treatment. --------------------------------------------------------------------------------------------------Myasthenia Gravis (MG) * A 35yo woman comes to the clinic complaining of double vision that seems worse near the end of the day. The patient also complains of difficulty chewing meat and other hard foods. She notices that her symptoms improve following a good night sleep. On neurologic exam you notice snarling appearance when the patient is asked to smile and a nasal tone to her speech. You also note weakness of the upper extremities when the patient is asked to clench her fists around your fingers repeatedly. * Most likely diagnosis is myasthenia gravis (MG). * In MG, there typically is not a distal to proximal movement. Patients complain of easy fatigability, especially in the muscles of the throat and eyes (dysphagia, ptosis, diplopia). Symptoms are not acute either. * MG is defined as muscle fatigue after repetitive motion of chronic onset with preferential involvement of the ocular and the pharyngeal muscles. * The best initial test for diagnosing myasthenia is serology for the anti-acetylcholine receptor antibody. A tensilon test is not the initial test. In the context of clinical suspicion of myasthenia, anti-acetylcholine receptor antibody is extremely specific for the diagnosis. * A positive tensilon test is not specific for the diagnosis; false positives seen in other diseases like amyotrophic lateral sclerosis (ALS, Lou Gehrig disease). * Only answer tensilon testing when there is no answer choice for acetylcholine receptor antibody. * The best (most accurate, most sensitive) test for the diagnosis is electromyography (EEG) looking for a decrease in the action potential spike on repetitive stimulation. * MG is essentially an autoimmune disease against the acetylcholine receptor. * Treatment is symptomatic for weakness and for the disease with autoimmune suppression. * Symptomatic treatment is with anticholinesterase medication (neostigmine, pyridostigmine) to raise the level of acetylcholine in the synapse, improving the weakness.

* To treat the autoimmune disease, give prednisone, azathioprine, IV immunoglobulin, plasmapheresis, and thymectomy. Prednisone is best initial immunosuppressive because it takes 1-3 months to have an effect. Patients may initially have a worsening of their disease. Azathioprine takes 3-6 months to have an effect. * If acute myasthenic crisis, use IVIg or plasmapheresis to improve the patients crisis immediately and acutely. * In patients with GBS or MG, you must follow their respiratory status. The diaphragm is a skeletal muscle and when the disease involves the diaphragm the patient can go into respiratory failure. So, always consider the possibility of an intubation and respiratory support to get them through the crisis. --------------------------------------------------------------------------------------------------Kaplan Videos (2001) Pulmonology with Dr. Asher Kornbluth, MD --------------------------------------------------------------------------------------------------Read the last line of the question stem to help determine what organ system the question is asking about. This will help you develop a better differential diagnosis as you read the question. --------------------------------------------------------------------------------------------------Pulmonary Function Tests * A pulmonary function tests (PFT) is generally used to differentiate between obstructive and restrictive disease. There may be a great deal of overlap between these. * Tidal volume is normal in-and-out respiration at rest. Good rule of thumb is 10mL/kg, so 700mL for a 70kg patient. Vital capacity is deep breath then exhaling maximally, does not empty lungs completely. Residual volume is left over lung volume after vital capacity. Vital capacity plus residual volume is total lung capacity. Residual volume is a difficult measurement to make and not done in clinical practice, so you never know the exact total lung capacity. * In restrictive pattern, all lung volumes are decreased. Therefore total lung capacity is decreased. Lungs are restricted and cannot expand due to disease such as sarcoidosis, interstitial fibrosis, and chronic tuberculosis. * In obstructive pattern, there is an obstruction in getting air out of the lungs; a hallmark of asthma and chronic obstructive pulmonary disease (COPD). Residual volume will be higher because you cannot expel all of it. Over years, the total lung capacity will increase with increased residual volume. This is what causes the large AP diameter, giving the barrel chest appearance. Measures of air outflow are reduced in obstructive disease. * FEV1 measured by having patient take deep breath in then breath out as fast as possible, with the forced expiratory volume measured over 1 second. * FVC is the forced vital capacity, basically the same as vital capacity. * FEV1/FVC ratio is decreased in obstructive disease, mostly because FEV1 is low. So FEV1 and FEV1/FVC low. * FEV1/FVC ratio is normal in restrictive disease, with low FEV1 and low FVC.

* FV25-75 is forced volume between 25% and 75% during a vital capacity. FV25-75 reduced in obstructive. * Residual volume high in obstructive lung disease, low in restrictive lung disease, but RV not easy to measure. --------------------------------------------------------------------------------------------------Alveolar-Capillary Membrane Diffusion * Diffusion capacity is a measure of alveolar capillary membrane, not an easy measurement to do. Alveolar-capillary membrane gets destroyed/fibrosed in emphysema. If the DLCO (diffusion of the lung carbon monoxide) is low, implies destruction of the alveolar-capillary membrane, so emphysema or ARDS (acute respiratory distress syndrome). ARDS patients are generally in the ICU on a respiratory so a DLCO isnt measure. * In bronchitis DLCO may be normal, in emphysema DLCO may be low. * Simpler way to measure gas exchange effectiveness is by measuring alveolararterial difference in oxygen, the AaDO2 or Aa gradient. This involves getting an arterial blood gas (ABG) after performing an Allen test to ensure ulnar collateral circulation (not sensitive but still done). On ABG, lab gives you pH, PCO2, PO2. * Say PO2 80mmHg (normally 90-100). Alveolar O2 at room air (21% O2) is 150 1.2*PCO2. * Normal PCO2 is 40, so Alveolar O2 would be 150 - 1.2*(40) = 102. Difference is achieved by subtracting what O2 you got in the blood gas (80mmHg), so 22 here. This is mildly elevated as normal is 5-15. * If there is a pulmonary process affecting the membrane, A-a gradient may be as high as 40. * Say our ABG comes back as 7.4/20/100. Alveolar is 150-1.2*20 = 126. 126-100 (PO2) gives 26 for Aa gradient. * Think about this physiologic process. The patient is breathing very fast to get PCO2 down to 20, so you know there is some problem. That is why measurement of PO2 does not tell you what kind of gas exchange you have. * So in summary, A-a gradient tells you what is going on for gas exchange at the alveolar-capillary membrane. * Given 7.4/36/96. Find Aa gradient. 150-1.2*36=107. 107-96 is 11 for Aa gradient (normal range). --------------------------------------------------------------------------------------------------Solitary Pulmonary Nodule * A solitary pulmonary nodule is a single nodule less than 6 cm. Lung cancers are rarely cured. The only time you can cure is if you have a solitary cancer and resects before it metastasizes. * So you found a < 6cm nodule on chest x-ray. What are the criteria for just following this nodule? Young patient (<50yo) and non-smoker. If this is the case, repeat the chest x-ray every 6 months for 2 years. If the nodule has not changed size, we know it is benign. Tell patient to take CXR with them next time they go for an x-ray. * If you are looking for calcification within a nodule, do a chest CT scan due to increased sensitivity.

* Popcorn appearance (scattered areas) of calcification within nodule, think hamartoma (e.g. hair, tooth). * Central or bulls-eye calcification within nodule, think granulomatous disease (e.g. tuberculosis). * Eccentric (near edge) single area of calcification within nodule, think possible malignancy. * Say patient is 60yo and 50pack*year smoker. You find a 2cm nodule on CXR. Next step is do a biopsy, most of the time by open lung biopsy. Usually nodule is not central enough to reach with simple bronchoscopy. If the biopsy shows malignant cells, next step is surgery to remove. If benign, follow with CXRs. --------------------------------------------------------------------------------------------------Pleural Effusion * The pleura covering the organ (the viscera) is the visceral pleura. The pleura outside that is the parietal pleura, with fluid between the two pleurae. When fluid increases between these two, this is a sign of a pleural effusion. * Early sign of pleural effusion is loss of costophrenic angle at the diaphragm, requires about 300mL of fluid. * Lay patient on the side of the effusion and do a lateral decubitus x-ray. If the effusion is free-flowing, the fluid will layer out into the dependent portion. This is useful to know to determine if you will get fluid via thoracentesis. If the fluid were trapped, we have to hit the trapped portion itself. * Pleural effusions classified into transudate and exudate. In general, exudate is seen in sicker patients. Transudate is seen in high-pressure differences (e.g. CHF) or low blood oncotic pressure (e.g. nephrotic syndrome, cirrhosis). Cirrhosis causes hypoalbuminemia (protein not created) typically resulting in right pleural effusion. * To differentiate, need pleural LDH and protein, with comparison to serum LDH and protein. Transudate has low LDH (<200) and low protein. Pleural to serum LDH < 60% and pleural to serum protein < 50%. * Transudate typically caused by systemic diseases. Exudates typically caused by lung diseases. * Exudate: pleura has some process that causes it to exude fluid. The larger the effusion, the safer the thoracentesis. * Say we do the tap and the LDH is 300 and protein is 3grams (high). This is an exudate. * Exudate causes include cancer, tuberculosis, pulmonary embolus with infarction, pneumonia. If infected material exudes out (parapneumonic), we have a complicated effusion and need a chest tube to drain else it scars and the patient loses lung volume. This scarring can happen quickly. * Complicated exudative effusion (requiring chest tube) includes infection with pH < 7, pus on tap, polys on gram stain, and low glucose in fluid. Next step is chest tube, not antibiotics, not intubation, not further diagnostic studies. * Parapneumonic effusion leaks fluid but no low pH, no pus, no polys on gram stain, not low glucose. So this is an uncomplicated parapneumonic effusion. This patient does not need a chest tube. So patient has fever, pneumonia, and effusion. Next step

is put a needle into the space to determine if it is complicated. If not, just treat pneumonia. * Empyema is an effusion with pus in the space, requires a chest tube. --------------------------------------------------------------------------------------------------Atelectasis * A 62yo man is dyspneic 24hours after cholecystectomy. His respiratory rate is 22, and pulse is 112, has a mild fever, and decreased breath sounds are noted in the left lower lobe. CBC shows leukocytosis of 27,000. * Patient is post-op with anesthesia medication causing decreased lung expansion, is on narcotic meds and likely not taking in deep breaths because of pain. Most likely diagnosis is atelectasis. * Atelectasis is collapsed alveolar airways, lung itself is not condensed on itself. * Massive ascites pressing up on lungs and impairing ability to take deep breath can cause atelectasis. * Broken ribs causing pain on inspiration can cause atelectasis. * This can give rhonci or crackling at the bases, perhaps a little wheezing. * You can get mild fever with atelectasis. Post-op surgical Ws of fever: Wind (atelectasis), Water (UTI), Walking (DVT), Wound (incision infection), Wonder drugs (drug causes) for days 1, 3, 5, 7, and 9. * Next step in diagnosis is chest x-ray. * Treatment is inspired spirometry breathing device, also known as incentive spirometry. --------------------------------------------------------------------------------------------------Chronic Obstructive Pulmonary Disease (COPD) * Patient has COPD, is a non-smoker, liver problems, and has relatives that died of lung or liver problems. Think alpha-1 antitrypsin deficiency. Patient can have either liver or lung involved, not necessarily both. * Treatment is symptomatically, recurrent antibiotics, hoping they do not need a lung or liver transplant. * COPD patients can develop pulmonary hypertension, giving a loud second sound on heart exam (P2). During inspiration, right-sided heart pressures increase. So here the second heart sound will get louder on inspiration. Patient may have a rightventricular heave, and ECG may show right-ventricular hypertrophy. * Worry that this patient will develop cor pulmonale. This is similar to CHF in a patient developing left ventricular failure due to hypertension. * Patient with COPD has obstruction so you hear increased wheezing, large AP diameter (barrel chest), and history including smoking, past episodes of bronchitis. * Patient with chronic bronchitis will have increased markings on chest x-ray. Patient with emphysema will have blebs that can become bullae, flattening of the diaphragm, widened silhouette so heart looks smaller, breath and heart sounds may sound very distant. * Pulmonary function tests will be diminished FEV1 and FEV1/FVC ratio.

* DLCO test can help differentiate between bronchitic patient and emphysematous patient. There is destruction of the alveolar-capillary membrane in emphysema (low DLCO) but not bronchitis. * In an asthmatic, we give them beta2 agonists acutely and try to keep them well between attacks with steroids. * Ipratropium tends to work better in COPD than asthma for bronchodilitation. * Beta2 agonists help in COPD. Antibiotics for frequent bronchitic infections; They get pneumococcus, h. influenza, moraxella catarrhalis. We use sulfa antibiotics, penicillin antibiotics, cephalosporins, sometimes tetracycline. We try to rotate these antibiotics. So say patient had a good response to TMP-SMX; next choice when they get an infection should be something new else they will develop resistance, so ampicillin or erythromycin. Macrolides (azithromycin) and fluoroquinolones (levofloxacin) also good choices. * Aminophylline is a phosphodiesterase inhibitor with minor bronchodilation effect that has some use in COPD. Toxicities of aminophylline include tachycardia, tachyarrhythmias, tremors, and seizures. We have much better and safer bronchodilators in asthma, namely the beta2 agonists like albuterol. * Aminophylline used in COPD to increase central respiratory drive, improves diaphragmatic contractility, it makes the work of breathing smoother. * Say patient is long-time smoker and getting tachyarrhythmia. Next step is stop theophylline or aminophylline and get an aminophylline level. * Aminophylline is not cleared well in patients on erythromycin, ciprofloxacin, and cimetidine. Drugs that decrease aminophylline levels include warfarin and phenytoin. * Steroids should be monitors. Even with inhaled steroids patient can get cataracts, osteopenia, osteoporosis, hypertension, fluid overload, diabetes, and so on. Demonstrate benefit (baseline PFTs) pre- and post-steroids. * Most patients do not benefit from steroids if you do the measurements before and after a couple of weeks. * Most important treatment for COPD is stopping smoking. Even stopping smoking after decades of damage has benefit. Smoking cessation improves symptoms and mortality. Combine pharmacologic and counseling. * When is the patient hypoxic enough to get chronic home O2? Criteria is PO2 < 55mmHg at rest. If PO2 between 55 and 60mmHg, likely you will be less than 90% oxygen sats. So in this group, look for end organ damage, so erythrocytosis (Hct increased) as a compensatory mechanism or signs of cor pulmonale. * So how much oxygen? Its cheap; why not just give 100%? These patients have respiratory acidosis, 7.30/50/50, so they have high PCO2, low PO2, and 50/50 club. Home O2 not only makes this patient feel better, it makes them live longer. We want this patient into the low 90% oxygen sat range. We dont want to knock out their hypoxemic drive to breath, else were in big trouble. This patient is already acidemic, so they need some hypoxia as a central CNS respiratory drive. Otherwise they hypoventilate more, PCO2 goes up, pH goes down. * Typical dose is home O2 1-2L via nasal cannula.

* COPD patients should get pneumococcal vaccine every 5 years. They should get influenza virus vaccine annually. They can also get the hemophilus influenza (faintly staining gram negative rod) vaccine once in a lifetime. * What is the most important test for COPD? Answer is FEV1 as a measure of lung disease. --------------------------------------------------------------------------------------------------Bronchiectasis * Destruction of the elasticity of the bronchial walls, leading to recurrent infections. * Bronchiectasis seen in chronic bronchitis patients and in kids with recurrent infections (tram track sign on chest xray), think about cystic fibrosis. * Be aggressive with pulmonary physical therapy. Postural therapy, have them bend over, clap on their back to get mucus moving. In cystic fibrosis, infection is likely to be pseudomonas aeruginosa. * Patients may develop episodes of blood-tinged sputum or hemoptysis. * Given a chest x-ray of a bronchiectasis patient where you see the liver impression on the right and gastric air bubble on the left. This is dextrocardia with situs inversus. Think Kartagener syndrome (primary ciliary dyskineasia). This is immotile ciliary syndrome, not helping to brush out infection. Also have immotile sperm. * Pulmonary toilet (pulmonary hygiene) is helpful here too, suctioning of the airways, chest physiotherapy, blow bottles, and nasotracheal suction. * These patients require the same vaccinations as in COPD, pneumococcal, influenza virus, hemophilus influenza. --------------------------------------------------------------------------------------------------Interstitial Fibrosis * Almost always idiopathic. What PFTs would you expect? Pattern is similar to restrictive, all volumes are diminished. So FEV1 and FVC diminished but ratio of FEV1/FVC is the same. * These patients are generally not smokers. Chest x-ray will show increased white markings (from scars). If they are a smoker, you would think emphysema. * On exam you find pulmonary hypertension, cor pulmonale, finger clubbing (not specific, but implies chronic). * Rarely will these patients respond to steroids. Measure pre-steroid PFTs and poststeroid PFTs. If no improvement, then stop the steroids. --------------------------------------------------------------------------------------------------Sarcoidosis * A 27yo woman comes to your office with painful erythematous papules that occurred one day before your visit. She has no other complaints except joint swelling and pain that occurred three days ago. The physical exam reveals low-grade fever, symmetric swelling of the knees, PIP joints, and MCP joints, with well demarcated 34cm papules over the anterior aspects of the legs. The papules are red and tender. What is the next step in confirming diagnosis? * The painful red nodules on the shins are erythema nodosum. Add in joint swelling and we could be talking about Crohn disease. We dont necessarily have information to know this is sarcoidosis. Generally no lung involvement in Crohn disease.

* On chest x-ray with sarcoidosis, you see bilateral hilar lymphadenopathy. Other causes of bilateral hilar adenopathy are granulomatous diseases (e.g. tuberculosis), bilateral lymphoma, metastatic adenocarcinoma. * Sarcoid more commonly seen in younger patients and in African American patients. * Sarcoid causes scarring, so you expect a restrictive lung pattern (decreased PFTs, normal FEV1/FVC). * Sarcoid associated with Bell palsy, parotid gland enlargement, skin lesions (lupus pernio on face, erythema nodosum on the lower extremities), uveitis (ocular emergency requiring steroids), bilateral hilar lymphadenopathy leading to lung scarring and eventually honeycombing pattern, liver granulomas with elevated LFTs, CNS involvement (treated with steroids), PFTs similar to restrictive disease. * Uveitis usually occurs rather abruptly, pain in the eyes, photophobia. So either give steroids or refer immediately to the ophthalmologist for slit lamp exam. * Other disease involving the head that can cause blindness, seen in older patient, temporal arteritis. * Unilateral hilar lymphadenopathy not seen in sarcoid. Worry more about things like TB, cancer, and metastasis. * Hypercalcemia seen in sarcoidosis due to elevated levels of vitamin D so absorb more calcium, hypercalciuria too. * Diagnosis is with tissue, so do a mediastinoscopy to biopsy nodes. Could biopsy supraclavicular node if it is available. Tissue should show non-caseating granulomas. * Kveim (Nickerson-Kveim, Kveim-Siltzbach) test is the equivalent of the PPD/mantoux test for TB. Kveim test is a purified antigen from the granulomas of sarcoid patients. This is not a test of choice, want biopsy. * Patients may have angiotensin converting enzyme (ACE) elevations, not sensitive or very specific. --------------------------------------------------------------------------------------------------Occupational Lung Disease * Typically occurs 10-20 years down the road. Pneumoconiosis, asbestosis, silicosis, and coal workers lung. * Smoking exacerbates and is superimposed on these diseases. * All have a restrictive lung pattern so come with decreased PFTs. * Pneumoconiosis seen in the mining industry or heavy metal exposure, gold, copper, silver, nickel, lead inhalation. This causes a chronic inflammatory response, are ingested by macrophages, and causes scarring over the years. * Pneumoconiosis comes with opacities and irregular densities on CXR, ground glass appearance. * No good treatment, remove patient from environment if still there, have patient quit smoking. * In asbestosis there is a synergistic cancer effect with smoking. * Asbestos found in older building insulation, pipes, boilers, old building construction, shipyard workers. * No specific signs or symptoms, diagnosis found via occupational history.

* CXR shows calcification of the pleura (white areas). Primarily you see thick pleural bases, eggshell white. * Mesothelioma is a specific and rare cancer seen with asbestosis, nearly always fatal. Detected by biopsy only. * What is the most likely type of lung cancer a patient with asbestosis will get? Answer is adenocarcinoma or squamous cell carcinoma primarily of the lung parenchyma, especially if a smoker. Answer is not mesothelioma. * 70-fold increase in primary lung cancer in asbestos exposure with concurrent tobacco smoking. * By the time you think to biopsy a cancer, it is likely too late. If you happen to get a biopsy, there will be barbell shaped ferruginous body (asbestos fiber coated with iron-rich material). * Treatment is to get patient to stop smoking. * Silicosis is seen with rock mining, digging tunnels, digging quarries. Silica fibers are released from rock blasting. * Restrictive disease. CXR nodules favor the upper lobes (similar to TB). These patients are at increased risk for TB. * Patients with silicosis should get annual PPD testing. * Coal miners lung associated with mining coal. Restrictive disease. * Patients develop interstitial lung disease, small round densities typically mid to lower lobes. * Encourage patient to quit smoking. * Patients with coal miners lung who have joint complaints, such as symmetric swollen joints possibly with hand involvement, you should think about rheumatoid arthritis (Kaplan syndrome) and check rheumatoid factor. --------------------------------------------------------------------------------------------------Pulmonary Embolism (PE) * A 32yo woman is brought to the ED with acute onset shortness of breath and pleuritic chest pain (sharp pain, worse with deep breath, coughing, or moving) that occurred while she was shopping. If you look down and see answer choices like venous Doppler, ventilation perfusion scan, spiral CT scan, you know it is likely a question about pulmonary embolus. This patient is not taking any medications except oral contraceptives. She has never been sick before. Her respiratory rate is 26, is tachycardic, lungs are clear, rest of exam is normal. ABG shows mild evidence hypoxemia. ABG 7.52/25/75. Drop PCO2 by 10, pH goes up by 0.08. * If you see oral contraceptives (hypercoagulability due to estrogen) and sudden chest pain, thing PE. * A-a gradient: 150 1.2*25 = 120. So 120 - 75 = 45 (normal < 15). So the problem is with gas exchange. * A thrombus is a clot at its point of origin. An embolus is a broken off clot that travelled. A PE usually originates from the deep veins of the legs or pelvis, thighs > calves.

* In a PE, the alveolus is just fine, the airway is just fine. But there is a clot blocking blood in the pulmonary artery. So there is an area that is ventilated but not perfused, waste of ventilation, ventilation/perfusion (V/Q) mismatch. * Physical exam will reveal an absence of problems, no wheezing, no rhonchi. So you need to do testing. * To detect a V/Q mismatch, you order a V/Q scan. Patient breathes in nuclearlabeled air and an image is taken to determine ventilation. Then IV nuclear material is labeled to RBCs and another image is taken to determine perfusion. Normally there is a superimposed pattern of ventilation and perfusion. If there is a PE, you will see ventilation but no perfusion in a particular area. * V/Q scan is not gold standard. Results come as low, intermediate, or high probability of PE. The radiologist has a large list of criteria to determine probabilities. With intermediate, you may be forced to do more tests. * What is the first step in detecting a pulmonary embolus? Answer is V/Q scan (non invasive study). * Another non-invasive test is a Doppler sonogram of the lower extremities. * What is the quickest test to determine if a patient needs treatment for a PE? Answer is Doppler. If there is a blood clot, that is enough to start with anticoagulation. * Gold standard to determine if a patient has a DVT is a venogram. The test is painful; injection of dye into small vein of the foot with imaging to see the dye. This is rarely necessary; only done if you have no choice. * Gold standard to detect a PE is a pulmonary angiogram. Say the Doppler is equivocal and the V/Q scan is equivocal, then you do the pulmonary angiogram. * Pulmonary angiograms are used less widely now due to more acceptance of the spiral CT scan with contrast. * A Doppler is a valid initial approach in a pregnant patient, since the presence of DVT warrants anticoagulation. * Risk factors for pulmonary embolus (Wells criteria): immobilization (e.g. post-op, especially hip replacement), cancer, HR > 100, PE is likely #1 diagnosis, clinical signs/symptoms of DVT, previous PE/DVT, and hemoptysis. Other hypercoagulable criteria are not included in Wells, such as use of oral contraceptives. * The DVT risk for hip surgery is so high that standard of care includes pre-op prophylactic anticoagulation (subcutaneous heparin, 5000U subq q12h) as well as post-op anticoagulation. * Patients at risk for DVT get anticoagulated (subcutaneous heparin) or get sequential compression devices (SCDs). * Patient may be ambulatory but have dementia so they dont leave bed, use subcutaneous heparin. * Patient is normally ambulatory busy businessman, takes long flight across seas and is suddenly short of breath, this is most likely a PE case. Even sitting in a car for a long trip. * Patients with CHF are at increased risk for DVT/PE due to poor cardiac output and blood stasis.

* Clots in strange places, like upper extremities (axillary vein) or in any artery, think hypercoagulable state. * Factor V Leiden is the most common cause of thrombophilia. Factor V is necessary for protein C to work. Protein C and S as well as antithrombin III (AT III) are necessary to prevent constant coagulation. Factor V Leiden causes activated protein C resistance, a qualitative problem, so quantitative levels of protein C are normal. * If the pulmonary embolism makes it peripheral to the pleura, the pleura can infarct and cause a pleural rub. You could see this on CXR possibly. This is not the normal case; even with a massive PE the CXR will be normal. * If a large pulmonary vessel is clotted, you can get signs of right ventricular strain, distended neck veins, still in the presence of a clear lung exam. May see signs of cor pulmonale acutely, loud S2, P2 louder with inspiration, acute right ventricular heave, right axis deviation on ECG, or ECG S1 Q3 T3 pattern (abnormal S wave in I, Q wave in III, and flipped T wave in III, not specific or sensitive). * When do you need to know for sure that this is a pulmonary embolus, so much so that you do the invasive and possibly risk pulmonary angiogram or pulmonary venogram? You do it when patient is at very high risk of bleeding if you anticoagulate them for treatment of the PE. Examples are recent major surgery, recent CVA, recent duodenal bleeding ulcer, recent major trauma. * A patient has a positive V/Q scan with symptoms. You do not think the patient is going to die from this clot. So you want to prevent the clot from getting bigger. Treatment is anticoagulation. The fastest acting anticoagulation agent is IV heparin, which will work within the hour. Keep patient on continuous heparin infusion. Heparin is not thrombolytic therapy, so it will not break the clot. You expect the patients own endogenous system (protein C, protein S, antithrombin III) to break up the clot and after time the vessel will recanalize. Heparin is given in bolus (say 5000U) then infusion (say 10 or 15U/kg) at say 1000U/hour. * Check heparin level with PTT (partial thromboplastin time) test. Aiming for time of 1.5-2.5 times the control. If control in assay is 30 seconds, you want 45 to 75 seconds. If youre at a PTT of 100, stop heparin for a few hours to let the level come down as patient is at high risk of bleeding. If PTT low, increase heparin infusion rate. * Treat patient with IV heparin for about 5 days. Start patient on oral anticoagulant (warfarin/coumadin) that they will stay on for the next 3-6 months. * Heparin works by potentiating antithrombin III effect. * Warfarin works by inhibiting vitamin K dependent coagulation factors (II, VII, IX, X, plus protein C). * Measure warfarin via PT (prothrombin time, pro time). Prothrombin no longer measured directly, it is done by INR. Problem with PT test is control time varies too much. International normalized ratio (INR) range for a DVT is between 2 and 3. If for stroke or prosthetic valve, INR between 3-4. So if INR is 1.7, increase oral warfarin dose to get them above 2 for the DVT. If INR is 5, we need to reverse the effect of warfarin.

* Warfarin has a sizeable morbidity and mortality associated with it because there is variation in week-to-week or even day-to-day dosing. So, we have to regularly check INRs. * If patient bleeding with high INR, we give factors II, VII, IX, X, more specifically fresh frozen plasma (FFP). * FFP comes from other human beings thus it can come with infections and transfusion reactions. * If the patient has a high INR without bleeding, give vitamin K. Oral vitamin K will lower INR in 24-48 hours. IV vitamin K will lower INR in 6-8 hours. Subcutaneous vitamin K not recommended. * Drug interactions are important with warfarin, as many drugs affect the same P450. * We do not start warfarin at the same time as we start the heparin. Warfarin inhibits protein C. Protein C has the shortest half-life, shorter than factor VII. So the first effect warfarin has is to make the patient protein C deficient, so they have more tendency toward hypercoagulability. * So once PTT is 1.5 to 2.5 times control, then we can start the warfarin. * 25yo woman stopped taking her birth control pills because she is scared of clots after reading about them on the Internet. She has been gaining weight and stays at home most days. So, this patient got pregnant and now is hypercoagulable due to estrogen, has a baby pressing on her pelvic veins, and is not moving around much. Pregnancy increases risk for DVT. If this patient does get a DVT, then we can give heparin. Heparin is safe during pregnancy. Warfarin is contraindicated in pregnancy, it is teratogenic. So we treat the patient with IV heparin for 5 days then send them home on subcutaneous heparin (10,000U subq TID as we need 20k to 30k units daily). * Low molecular weight heparin (LMWH) has more consistent effect on PTT, so much so that we do not need to follow repeated PTT levels. Once the patient has a stable PTT, we just follow the dose. LMWH is a once a day dose. * The problem with LMWH is that they are very expensive, could be a few hundred dollars a day. * Say patient has life-threatening PE, like a saddle embolus at the main pulmonary artery. This causes a ventilation perfusion mismatch at the entire lung. So even if you intubate the patient and deliver 100% O2 directly into the trachea, no blood will be there to transfer the oxygen into the system. * One sign this is a potentially fatal PE is that we are delivering 100% oxygen and the sats keep dropping, or blood gas shows increasing hypoxemia. Another sign is acute right ventricular failure with hypotension. Hypotension in the setting of pulmonary embolus is an ominous finding; the right ventricle is trying to pump against tremendous resistance, so it will acutely fail, left atrium has no input thus no LV output. * Treatment for life-threatening PE is thrombolytic therapy. Give tissue plasminogen activator (tPA) or streptokinase (SK). This will lyse the clot, if there is enough time to save the patient.

* Why dont we just give tPA in every patient? About 1% of patients getting thrombolytics die of a bleed. About 10% of patients getting thrombolytics have a major bleed. * Say patient had major PE then became hypotensive and fell, resulting in an intracranial bleed. Or patient has major PE and a recent GI bleed. Do you give tPA or streptokinase? Answer is emergently take patient to the OR or via interventional radiology (IR) to do an embolectomy. * We anticoagulate the embolus 3-6 months if there is a reversible cause. For instance, a post-op patient who got a DVT or a patient who got a DVT after a plane ride. What about patients who have an on-going risk, like CHF or an underlying coagulation disorder? Then we anticoagulate for as long as the risk exists. Say patient got a DVT in the first month of pregnancy due to increased estrogen levels, then we anticoagulate for the entire pregnancy. * What if the patient has a PE, is hemodynamically stable, but anticoagulation is contraindicated (e.g. recent major hemorrhage). We have to protect the patient against future emboli, but cant anticoagulate. So we put in an inferior vena cava (IVC) filter. In less common instances, SVC filters can be placed as well. Some IVC filters are shaped like an umbrella (IVC umbrella), some are disk shaped with pores, others may be mesh patterned. * Say the patient has a PE, we diagnose and treat with the correct PTT then INR. What if the patient gets another pulmonary embolism? Answer is place an IVC filter. The filter itself is thrombogenic, thus we do not stop the anticoagulation if we do not have to. --------------------------------------------------------------------------------------------------Acute Respiratory Distress Syndrome (ARDS) * 32yo man is admitted to the ICU with presumed gram-negative sepsis. He is placed on double gram-negative antibiotic coverage and remains stable for the next 24 hours. Blood cultures grow pseudomonas sensitive to ceftazidime and ciprofloxacin, which the patient has already been started on. The patient seems to improve over the next day, until he suddenly develops severe dyspnea. Physical exam reveals diffuse crackles. An ABG shows hypoxemia and hypercarbia. Diffuse alveolar densities are seen on CXR, compared with an unremarkable admission chest x-ray. The lungs are flooded with fluid, so oxygen cannot diffuse through the fluid to the membrane. * Premature newborns get similar problems (gas exchange problem at the alveolarcapillary membrane) when born without enough surfactant. * ARDS also known as non-cardiogenic pulmonary edema. * Thus to prove ARDS, you have to rule out cardiac so normal pulmonary wedge pressure and LV end diastolic. * ARDS occurs in gram-negative sepsis, burns, trauma with or without multiple PRBC transfusions, drowning, narcotic overdose. * Giving these patients oxygen does not do much to fix hypoxemia. Definition is PO2 < 60% on FIO2 > 60%, real 60% oxygen into trachea with patient intubated.

* Treatment is intubation and delivery 100% inspired oxygen. If an ABG shows PO2 low (like 45), then we add PEEP (positive end expiratory pressure). PEEP holds the alveoli open, which helps to force the air through the fluid. * Risk of giving PEEP is blowing a hole in the alveolus, barotrauma, causing pneumothorax. * If you cause a pneumothorax and do not recognize it, the PEEP will continue to push air out, leading to a tension pneumothorax. Patient will become hypotensive, mediastinum shifts away. * As intrathoracic pressure gets increased with PEEP, you increase right-sided venous return at the expense of the left ventricle; so think of it as compressing the left ventricle, resulting in hypotension. * Patient on ventilator, CXR normal, and patient getting hypotensive. Answer for next step is to decrease PEEP. * Oxygen toxicity can occur with FIO2 > 50%, which causes fibrosis. * To measure capillary wedge pressure (PCWP), insert a pulmonary artery catheter (Swan-Ganz catheter). Insert into a large vein, like internal jugular or subclavian (IJ safer because further from lung). Catheter placed through RA, RV, pulmonic valve, into outflow tract with balloon deflated. Inflating the balloon too much could rupture the pulmonary artery. On the catheter is a pressure transducer, so we can measure the pressure in the pulmonary artery. * At the end of diastole, the ventricle is maximally full, aortic valve is closed, so this is the left ventricular end diastolic pressure; the mitral valve is still open so the pressure is transmitted back to the left atrium, that pressure transmits back to the pulmonary veins, and back to the pulmonary capillary. * So we inflate the balloon, it travels down and gets wedged in the pulmonary capillary. We get a continuous column of pressure at that capillary; which is in continuous communication with the left ventricle. * Say LVEDP normally is 7-12mmHg, what if the pressure is 5mmHg? The low pressure suggests it is noncardiogenic pulmonary edema (ARDS). So pulmonary edema with low LVED pressures suggests ARDS. * Say pulmonary capillary wedge pressure (PCWP) is 25mmHg with edema? That is cardiogenic. * You can also measure cardiac output via the Fick equation with a Swan-Ganz catheter. * Catheter has a thermometer on the tip along with balloon and pressure transducer. Thermometer tells you what the temperature of the blood is. Take chilled saline that you know the temperature and fluid amount. You inject the saline and the catheter measures continuous temperature. The quickness and degree to which the temperature falls in response to the saline helps determine the rate of flow and thus the cardiac output (L/min depending on BSA). * Cardiac index should be > 2.2 L/min/BSA. Below 2.2 is very poor flow. * Cardiac index may be normal or high in ARDS, but in cardiogenic pulmonary edema it is low.

* You can follow cardiac index to help determine how much diuretic or inotrope to give. --------------------------------------------------------------------------------------------------Sleep Apnea * Sleep apnea is cessation of breathing during nighttime sleep, occurring at least 1015 times per hour and at least 10 seconds per episode. * Patient does not realize this is happening. So either the patient has daytime somnolence (fall asleep at my desk, drink more and more coffee and still falling asleep, falling asleep while driving) or spouse notices loud snoring. * Major cause of sleep apnea is obstruction of the upper airway at the larynx and epiglottis. * Seen in obese patients. Will have tremendous intolerable (to others) snoring. * About 5% of cases are central apnea where the issue is not in the upper airway but in the respiratory drive. * Treatment is with CPAP (continuous positive airway pressure) mask at night, similar concept as PEEP. * The positive pressure in CPAP is being delivered to the mouth, so no risk of barotrauma like in PEEP. * Testing done in sleep lab, patient hooked up to EEG, ECG, pulse ox, respiratory rates measured. It is not a popular test because the patient is trying to fall asleep while hooked up to all these devices. * More invasive treatment if CPAP fails is surgical intervention to open the airway; not always successful. --------------------------------------------------------------------------------------------------Lung Cancer * Most common cause of cancer death in both men and women is lung cancer. * Women: 2nd cause is breast cancer (1st if non smoker), 3rd is colorectal cancer. * Men: 2nd cause is prostate (1st if non smoker), 3rd is colorectal. * A non-smoker that gets lung cancer is likely adenocarcinoma. * Most common initial presentation is a cough. This is a very non-specific symptom; smokers usually cough. So, look for a change in the cough. Maybe they have superimposed bronchitis. This is a difficult diagnosis at times. Taking all lung cancers that present to the physicians office, 5-year survival rate is about 5%. * We have no good diagnostic test to improve survival in smokers who develop lung cancer. * Cough with another symptom, like pleuritic chest pain, blood-tinged cough, postobstructive pneumonia. * Once a patient has about 10% weight loss with a new lung cancer diagnosis, nearly all are unresectable. * Post-obstructive pneumonia is when patient has recurrent pneumonia in the same location. Patient gets pneumonia, gets better, CXR 3 weeks later still shows a patchy spot. That is because secretions cannot clear due to obstruction.

* So any smoker with pneumonia you must follow CXR until it is clear. If not cleared, must get bronchoscope. * Chronic smokers generally develop chronic hoarseness/laryngitis. If a patient suddenly develops hoarseness, you should worry that metastasis has travelled to the recurrent laryngeal nerve (outside lung). Testing is with laryngoscopy and patient phonating to determine if a vocal cord is paralyzed (ENT consult). * Superior vena cava syndrome implies a big mass in the chest is impinging the SVC, this implies unresectable cancer. Patient will get swelling in face with redness, called facial plethora (Pemberton sign). * If cancer is found and centrally located, you can do a bronchoscopy and biopsy. * If the lesion is peripherally, one can do a CT guided needle biopsy. * If the lesion is somewhere in the middle, must do an open biopsy (can be done laparoscopically these days). * Squamous cell carcinoma is generally centrally located, so more likely to be bronchoscopy with biopsy. Associated with paraneoplastic syndrome of hypercalcemia due to parathyroid hormone like substance secretion. * Small cell (oat cell) carcinoma generally centrally located, associated with SIADH (hyponatremia with euvolemia), Eaton-Lambert syndrome, early SVC syndrome. * Eaton-Lambert syndrome is neuromuscular end-plate injury, differentiated from myasthenia in that muscles improve with use (MG patients worsen with use). * Adenocarcinoma carcinoma and large cell carcinoma tend to be more peripheral, so needle aspiration biopsy or thoracotomy to get tissue for diagnosis. * Non-small cell cancers can only be cured if they are resected prior to metastasis. To determine if the patient can be cured, look for lack of metastasis. Signs of metastasis would be malignant pleural effusion, brain involvement, bone involvement, local contiguous structures (e.g. liver, esophagus), local extension into nearby organ. * You can have local nodes and be cured with resection only if the nodes are on the same side of the cancer. * Even if you have a tiny lung cancer and a single node on the contralateral mediastinum, its too late to cure. * Patients may still be resected for palliative measures, such as a large tumor causing obstruction. * Non-small cell lung cancer can be palliated with chemo and radiation, but this is not a cure. * Small cell cancer metastasizes early. So by the time you make the diagnosis, you must assume there has been micro metastasis and thus resection is not an option for cure. * One could be cured with chemotherapy in small cell cancer, but 95% are not cured at 6 months and die. * The only good news about lung cancer is the rates are decreasing as people are smoking less and less. * The greatest impact to reduce lung cancer rates is through education of teenagers before they start smoking. ---------------------------------------------------------------------------------------------------

Kaplan Videos (2001) Nephrology with Dr. Barry Weiss, MD --------------------------------------------------------------------------------------------------Questions should be core, straightforward topics with obvious answers if you know the content. The questions are not meant to trick the test taker. These are classic cases, classic scenarios. --------------------------------------------------------------------------------------------------Acute Renal Failure * If youre told a patient has bad breath, you can be sure were talking about Zenker diverticulum. A canoe trip in Connecticut, were talking about Lyme disease. There are similar key words and patterns to know for nephrology. * 62yo man comes to the ED for nausea and vomiting. Lab tests show creatinine of 3.6, it was normal two months ago. Why are the kidneys failing? * Three main causes of acute renal failure: pre-renal azotemia (non kidney problems, inflow issues like circulation), intra-renal azotemia (intrinsic disease), and post-renal azotemia (outflow). * Pre-renal azotemia is due to poor perfusion of the kidney. Causes include volume depletion (dehydration, burns, third-spacing into abdomen from pancreatitis), cardiovascular (hypotension, poor cardiac output especially CHF), decreased oncotic pressure (low albumin edematous states like cirrhosis, nephrotic syndrome), renal artery vasoconstriction (meds like NSAID, ACE-I, and anatomic like renal artery stenosis). * Some of the blood flow into the kidney is prostaglandin-dependent and NSAIDs block prostaglandin. * Pre-renal azotemia is important because it represents low blood flow to the kidneys; best considered as transient ischemia. If untreated, can progress to renal damage (acute tubular necrosis) and intrinsic renal failure. * Must be distinguished from intra-renal and post-renal azotemia because the treatments are very different. * Post-renal azotemia causes include bladder outlet obstruction (prostate enlargement, pelvic tumors, urethral stricture), ureteral obstruction (tumors, stones, papillary necrosis). Significance is back pressure damages kidneys. * Intra-renal failure is damage to the kidney itself, also called intrinsic renal disease or acute tubular necrosis (ATN). * Tests for post-renal azotemia include prostate exam, catheterization or bladder scan (to detect post-void residual urine volume), best test is ultrasound of the kidneys, ureters, and bladder (U/S KUB). * You expect about 50mL of urine left in bladder after urination, so if a post-void scan shows 300mL of urine, you should worry about outlet obstruction. * Normal BUN/Creatinine ration is 10 to 20. Post-renal could also have a BUN/Cr of 10-20. < 10 is intra-renal. * Tests for pre-renal azotemia include high BUN/Cr ratio (> 20) is the most important. In low flow states, functional kidneys retain sodium to maintain vascular volume, so low urine Na+ (< 10-20) and low fractional excretion of Na+, or FENa, (< 1%).

* What happens is the kidneys dont get enough blood, they think you are bleeding to death (evolutionary response), so they retain lots of sodium to help build up intravascular volume, thus urine sodium is very low. * In low-flow states, functional kidneys will concentrate urine to retain fluid in the body, so high (concentrated) urine specific gravity and high (concentrated) urine osmolality. * 62yo man comes to the ED with nausea and vomiting, creatinine was normal now is 2.6. Has orthostatic hypotension. BUN 82, urine Na+ 6, urine specific gravity 1.028. Answer to why they are failing is pre-renal azotemia. BUN/Cr is high, urine sodium is low so kidneys are retaining it. * Treatment for pre-renal azotemia is fluids. * Intra-renal azotemia: BUN/Cr is < 10-15. Urine sodium >20-40 because kidneys are failing so they lose the ability to protect sodium, it just leaks out. Cant concentrate urine, so urine specific gravity < 1.015, urine osmolarity < 350. Also look for casts, granular and RBC. Casts come from the kidney. * If patient on diuretics, like furosemide, then that may cause high urine sodium do dont assume intra-renal failure. * So in pre-renal BUN/Cr is high with low urine sodium, in intra-renal BUN/Cr is low with high urine sodium. * In summary, youre faced with a high creatinine level that is new (azotemia). Follow azotemia algorithm. * Algorithm: First thing to do is exclude pre-renal azotemia by looking for lack of BUN/Cr > 20, urine Na < 10, FENa < 1%, Osm > 500, high specific gravity. If you find pre-renal, you treat the cause (most causes you give them fluid, or treat CHF or renal artery stenosis). Next thing to do is exclude post-renal azotemia by sonogram and/or catheter looking for lack of obstruction. If you find post-renal, treat it. So, if you dont see pre-renal or post-renal problems, it is most likely intrinsic renal. Signs of intra-renal disease are BUN/Cr < 10, urine Na > 20, FENa > 1%, Osm < 200, low specific gravity. --------------------------------------------------------------------------------------------------Intra-Renal Disease: Tubulo-Interstitial Disorders * Acute tubular necrosis (ATN) occurs in prolonged ischemia (hypotension, prolonged pre-renal azotemia, arterial insufficiency/occlusion, previously called shock kidney), toxins (radiologic contract, drugs like aminoglycosides and amphotericin B, pigments like myoglobin). * Elderly man dropped pen under bed and went to go retrieve it. Once uncer the bed he couldnt get out. Neighbor realizes man hasnt come out of the house for a few days and finds the man stuck under the bed. Patient has been lying on the big muscles of the legs for a long time with no good circulation. The broken down muscle tissue releases myoglobin, which goes through the renal tubules and is toxic to the tubules. * Phases of ATN are prodromal, oliguric (when kidney starts showing changes), and post-oliguric (diuretic).

* Patients who have diabetes (and are taking metformin) and get contrast dye should have the metformin stopped prior to the procedure. * Allergic interstitial nephritis is an allergy reaction to a drug. Typical drugs are betalactams (e.g. cephalosporins, penicillins), sulfonamides, rifampin, allopurinol. Presentation is fever, rash, peripheral eosinophilia. Key findings are urine eosinophils. If you see urine eosinophils, think allergic interstitial nephritis and look for what drug needs to be stopped, most commonly a cephalosporin. Treatment is stop drug and short-term steroids to cool down reaction. * Deposition disease causes include hemoglobin (hemolysis), myoglobin (rhabdomyolysis), protein (multiple myeloma with Bence Jones protein), oxalate crystals (ethylene glycol, vitamin C), uric acid crystals (chemotherapy, tumor-lysis, gout), hypercalcemia (hyperparathyroidism). * Infection causes include pyelonephritis with WBC casts and bacteria in urine. * Drugs and toxin causes include analgesics (NSAIDs), antibiotics (aminoglycosides, amphotericin B, cephalosporins, expired tetracyclines leading to Fanconi syndrome), cancer drugs (cisplatin, methotrexate, cyclosporin, mitomycin C), radiation nephritis, heavy metal poisoning (lead, mercury, gold, lithium). --------------------------------------------------------------------------------------------------Intra-Renal Disease: Glomerular Disorders * Vasculitis causes include Wegener granulomatosis (lesions in lung, nasal, sinus, and ANCA anti-neutrophil cytoplasmic antibodies), Henoch-Schonlein purpura, polyarteritis nodosa, thrombotic thrombocytopenic purpura, cryoglobulin disease, hypertension, diabetes. * Patient has lung lesions and recurrent sinus infections. Also has kidney issues. Think Wegner and order ANCA. * Glomerulonephritis causes include post-streptococcal (do ASO titer, follows strep skin or throat infection), Berger disease (IgA mesangial deposition after viral illness, young children), collagen vascular disease (e.g. SLE), Goodpasteur syndrome (antiGBM antibodies in lung and kidney), Alport syndrome (renal failure and deftness). * Teenager presents with intrinsic renal disease. Had a sore throat a week before. Answer is order ASO titer. --------------------------------------------------------------------------------------------------Glomerulonephritis * Intra-renal disease has high urine sodium (> 20) and high FENa (> 1%), abnormal urinary sediments (hematuria, RBC casts). Renal biopsy is the most definitive test; not done often in clinical practice because of risk. * Nephrotic syndrome occurs in vasculitis and glomerulonephritis. Massive proteinuria > 3.5g/day, this is 4+ dipstick positive with grams on measurement. These patients will lose their oncotic pressure and develop edema. They can run their total albumin way down and get anasarca (marshmallow man appearance). Hyperlipidemia also seen. Best test to establish a diagnosis of nephrotic syndrome is 24-hour urine protein.

* So you have a patient with azotemia. You rule-out pre-renal and post-renal. You see blood in the urine and RBC casts. Dipstick is positive for protein and patient has edema. How do you prove it is nephrotic? 24h urine protein. * Best test to determine the type of glomerulonephritis causing the nephrotic syndrome is renal biopsy. * Best initial test to distinguish causes of intrinsic renal disease is a urinalysis. * Urinalysis can show infection (WBC, bacteria), drug hypersensitivity (eosinophils), glomerulonephritis (RBC casts), nephrotic syndrome (lots of protein), myeloma (Bence Jones), uric acid (gout), myoglobinuria (blood on dipstick due to myoglobin, no RBCs, high creatine kinase CK in blood). * 62yo man comes to the ED for nausea and vomiting, creatinine 3.6 (new), no medications. Exam is normal except large prostate. BP 11/70, pulse 110, BUN 72. Urine sg 1.023. Urine Na+ 9. What is the best initial treatment? You may be tempted to pick sonogram due to outlet obstruction, but a 62yo man probably has a large prostate due to BPH. If you look at the BUN/Cr, its 20 plus concentrated urine. So pre-renal azotemia. Patient has high pulse and nausea with vomiting, so likely dehydrated. Best initial treatment is fluids. * 62yo man comes to the ED with nausea and vomiting. Creatinine is 3.6 (new) and is on NSAIDs. Exam normal except large prostate. BP 110/70, pulse 110, BUN 41, urine sg 1.004, RBCs, no casts. Urine Na+ 42. BUN/Cr is close to 10, cant dilute urine even though he is vomiting/dehydrated. This is likely intrinsic kidney (intra-renal) azotemia. Could have hypoperfusion due to NSAIDs. Could have BPH or cancer causing large prostate obstruction. Could have bladder cancer or kidney stones due to RBCs in urine. What is the next step though? Answer is sonogram to exclude postrenal azotemia. Once that is out of the picture, we assume renal disease and follow that. So we ruled out pre-renal with urine originally, but still need to rule out postrenal. * 14yo boy comes to see you with swollen hands and legs. He has a recent sore throat, afebrile, BP 140/90 (pretty high), exam confirms edema. Urine shows proteinuria and RBC casts. This is post-streptococcal glomerulonephritis. What blood test would be most helpful for confirming diagnosis? Answer is ASO titer. --------------------------------------------------------------------------------------------------End Stage Renal Disease (ESRD) * ESRD is renal failure that is incompatible with survival. In order to survive, you need dialysis or a transplant. * Most common causes are diabetes (30-50%), hypertension (10-25%), glomerulonephritis (15%), polycystic kidney disease (5-10%), and unknown (10-15%). Most common cause of dialysis is diabetic kidney disease. * Need for dialysis is not determined by creatinine level. It is determined by the presence of acute manifestations. * Chronic manifestations of ESRD occur whether or not you are on dialysis. Hypocalcemia with osteopenia occurs. There is a lack of 1,25-dihydroxy vitamin D, which is involved in calcium reabsorption in the GI tract. This causes secondary hypoparathyroidism, calcium is leached from bone causing osteopenia.

Hypermagnesemia occurs due to failure to excrete magnesium, usually asymptomatic. Accelerated hypertension (renin-angiotensin action) occurs and is a big deal because it may be hard to control blood pressures; the combination of accelerated hypertension and hyperlipidemia leads to accelerated atherosclerosis leading to cardiovascular disease. Anemia occurs due to lack of erythropoietin production in the kidneys, start to occur when the creatinine gets to about 3. Immunosuppression occurs so increased infections. Pruritus occurs presumably due to toxins circulating in the blood that are normally excreted by the kidney. * Acute manifestations of ESRD include hyperkalemia (look for peaked T-waves on ECG), the mnemonic used is the T-waves are so sharp and pointed that you dont want to sit on them meaning do not sit and wait before treating. Metabolic acidosis occurs but kidneys cannot compensate, so must get dialysis. Fluid overload occurs to the point of pulmonary edema and the kidneys cannot remove the fluid, so need dialysis. Pericarditis occurs due to toxins causing inflammation of the pericardium and build-up in the pericardial sac of fluid, needs dialysis. Encephalopathy occurs at to high creatinine levels, and can lead to them being obtunded and even comatose, needs dialysis. All acute manifestations are indications for dialysis. * Dialysis can be done via blood (hemodialysis via a machine) or peritoneal dialysis (fluid instilled into peritoneum then removed). * 62yo dialysis patient returns from a 4-day fishing trip and feels poorly. This is a clue up front because most people on dialysis get it every 2-3 days. Patient has nausea, vomiting, and shortness of breath. ECG shows tall peaked T waves. Cardiac exam shows peripheral edema and signs of CHF. Creatinine is 5.2, bicarb is 16 (metabolic acidosis). What is the best treatment for this patient? Answer is dialysis now, not furosemide for CHF, not kayexalate for potassium, not bicarbonate for acidosis. This is an indication for dialysis, not to mess around with other treatment. * Some key words in renal disease to remember. Eosinophils in urine, think drug reaction like cephalosporins. RBC casts, think glomerulonephritis. Heavy proteinuria, think nephrotic syndrome. High triglycerides think nephritic syndrome. Hemoptysis, think Goodpasteur. Radiologic contrast, think acute tubular necrosis (ATN). Urine Na+ < 10, think pre-renal and give fluids. BUN/Cr > 20x normal, think pre-renal give fluids. --------------------------------------------------------------------------------------------------Electrolyte Disorders: Hyponatremia * Hyponatremia is arguably the most common of the sodium and potassium abnormalities seen in the hospital. * Hyponatremia mild or gradual change the patient will most likely be asymptomatic. With moderate change, symptoms include headache and confusion. With severe or rapid change, symptoms include seizures, coma. Emergency treatment is hypertonic saline (3% or 5%). * Causes of hyponatremia include pseudo-hyponatremia, where sodium level measures low but artifacts from other substances make it appear low, such as in high osmolality of hyperglycemia (1.6mg% sodium per 100mg% glucose). So in

hyperglycemia, the patients sodium is not truly low thus do not treat the sodium, treat the glucose. * Pseudo-hyponatremia can occur also with normal osmolality, lab artifact from hyper-trigleridemia or hyperproteinemia (e.g. multiple myeloma). Not likely to show on the exam but can happen in clinical practice is when a phlebotomist draws blood at the antecubital fossa when D5W is running at a distal hand IV site, showing hyponatremia (diluted) and hyperglycemia (from the dextrose). * Real hyponatremia is lab hyponatremia (from a basic metabolic panel) with low osmolality. * Hyponatremia with high osmolality (hyperglycemia), hyponatremia with normal osmolality (high lipids or protein), and hyponatremia with low osmolality (true hypoglycemia, multiple causes). * First step in determining why a patient is hyponatremic is determining the osmolality. Osmolality can be determined by direct measurement (best) or by calculation (osmolality = 2*Na + glucose/18 + BUN/2.8, or a simpler but less perfect method is 2*Na + 10). Normal blood osmolality is between 280 and 300. * Hyponatremia with high osmolality caused by increase solute in the blood, usually glucose. Causes osmotic fluid shift from intracellular space into the blood. Fluid shifted into blood dilutes Na+ level. There is a magnitude effect, where sodium falls 1.6 per 100mg/dL increase in glucose. Say glucose is 342 and sodium is 126; that glucose accounts for about 3.2 in sodium drop, so sodium is actually 129 adjusted. So the glucose does not explain all of the drop in the sodium. * Patient presents with diabetes. Labs show glucose of 402, sodium of 131. Does the glucose explain the level of hyponatremia? Here, the glucose is about 300mg/dL high, so 3*1.6 is 4.8 difference for sodium. Thus adjusted sodium is 136. Thus sodium is normal so the glucose explains this pseudo-hyponatremia. * With normal osmolality pseudo-hyponatremia, in the healthy patient there is a small amount of protein/lipid per block of blood, with a normal amount of sodium. When protein/lipid increase, there is less space for the sodium and water component of blood. So per unit of blood, there is less sodium even though the concentration has not changed. * Patient presents with weakness, sodium is 128, glucose is 142. Serum osm is 292 (normal). Total protein is 7.2 (normal). What test will show the cause of the patients hyponatremia? Answer is serum lipids (triglycerides). We have low sodium with normal osmolality, hyper-proteinemia was ruled out. So hyper-triglyceridemia is next. * Hyponatremia algorithm: once we rule out high osmolality and normal osmolality, lab shows osmolality is low. In this case, the next step is to measure the extracellular fluid volume, so the patients volume status. High volume status assessed with edema, low volume status assessed with tachycardia and orthostatic hypotension. * If the ECF is high (hypervolemia), the patient is fluid overloaded and the cause is almost always an edematous condition, including cirrhosis and congestive heart failure. If ECF is low (hypovolemia), the patient has fluid loss with causes including

GI, kidney, diuretics, sweat, and burns. If the ECF is normal (euvolemia), there are multiple causes including SIADH. * Low osmolality with hypervolemia, edematous conditions. Sodium is gained, accompanied and exceeded by secondary water gain. Diagnoses to consider are congestive heart failure, nephrotic syndrome, and hepatic failure or cirrhosis. * Low osmolality with hypovolemia, volume depletion. Extrarenal loss (urine Na+ < 10) due to GI loss from vomiting, diarrhea, tube drainage, bowel obstruction, or due to skin loss from burns, sweat. Renal loss (urine Na+ > 10) due to diuretics, saltwasting nephropathy (renal failure), hypoaldosteronism, glucocorticoid deficiency. * Low osmolality with euvolemia, normal volume states. SIADH, water intoxication (> 12-20L/day, also known as psychogenic polydipsia), and hypothyroidism. * SIADH has urine osmolality inappropriately high in the face of hyponatremia with low serum osmolality. Normal renal, adrenal, thyroid function. May be a drug side effect. So again, ADH is secreated causing the patient to retain fluid and excrete concentrated urine. Blood osm is low/dilute, urine osm is high/concentrated. * Causes of SIADH include lung disease, brain disease, neoplasm-related (most common is small/oat cell in the lung), drug side effect. *Drugs that increase ADH secretion include TCAs, MAO inhibitors, fluoxetine, carbamazepine, narcotics, phenothiazines, clofibrate, vincristine, cyclophosphamide, vinblastine. * Drugs that increase ADH action include chlorpropamide, tolbutamide, theophylline, NSAIDs, carbamazepine. * Drugs that mimic ADH function include oxytocin, used in labor induction. * Drugs to pay attention to here include TCAs, phenothiazines, vincristine, cyclophosphamide, chlorpropamide. * Diagnosis of SIADH is accomplished by seeing hyponatremia, low serum osm, high urine osm. * Treatment is to reverse the cause if possible (e.g. fix pneumonia, stop drug), restrict fluids, furosemide with normal saline (to raise blood sodium), and if severe and cant find a reversible cause then demeclocycline or lithium. * Emergency treatment for serious hyponatremia is hypertonic saline. * 82yo man presents with 1 week of vomiting and 2 days of confusion. Takes thyroxin, fluoxetine, and chlorpropamide. Sodium is 126, glucose 342, and has pneumonia. What are the possible causes of hyponatremia? Vomiting could cause dehydration. Hypothyroid can cause ADH syndrome. Fluoxetine and chlorpropamide can cause SIADH. High glucose could cause pseudo-hyponatremia. How do you determine the cause? You need osmolality level and volume status. You also need urine sodium if volume status is low. Get a urine osmolality if volume status is normal. * Key points about hyponatremia: determine serum osmolality, high and normal osmolality are artifacts of high glucose/lipids/protein, with low osmolality you need to determine ECF.

* Correcting hyponatremia too quickly can cause central pontine myelinolysis, resulting in quadriplegia. The more appropriate term now used is osmotic myelinolysis. --------------------------------------------------------------------------------------------------Electrolyte Disorders: Hypernatremia * Hypernatremia is likely the least common of the sodium and potassium abnormalities. * Causes include fluid loss in excess of Na+ loss commonly occurs in insensible loss from sweat (extreme exercise), burns, and commonly occurs in GI loss from diarrhea (e.g. children), and less commonly in diabetes insipidus (DI) where water loss is up to 20L per day. DI can occur due to central causes (idiopathic, tumor) and nephrogenic causes (lithium, various renal diseases). * What is the most common cause of hypernatremia? Answer is dehydration. * Symptoms include lethargy, confusion, seizures, and coma. * Treatment for dehydration is isotonic fluids with slow correction of sodium (1mEq/2h). If you correct the sodium too rapidly, it can cause seizures due to cerebral edema. * If you cannot correct the sodium, think about diabetes insipidus. * If you think the patient has DI, do a water restriction test. In a normal person, the urine volume will decrease with time and the urine output will be more and more concentrated. In diabetes insipidus, there is no change in urine volume because there is no ADH to do it. The urine will not change concentration either. Then, the patient is given ADH. If there is no response to the ADH, the problem is the kidneys (nephrogenic DI). If there is a response to the ADH, meaning urine output drops and becomes more concentrated, then the problem is in the brain (central DI). * Treatment of central diabetes insipidus is to correct the underlying problem if possible (e.g. lithium, brain tumor), give vasopressin in the form of desmopressin (DDAVP) usually intranasal or sometimes subcutaneous or IV. * Treatment of nephrogenic diabetes insipidus is to give thiazides or NSAIDs. Thiazides cause Na+ depletion and water resorption in the kidney. NSAIDS increase prostaglandin (PG1), which impairs urinary concentration ability. --------------------------------------------------------------------------------------------------Electrolyte Disorders: Hypokalemia * Hypokalemia common causes are GI loss from diarrhea, tube drainage, also medication such as diuretics causing renal loss, beta-agonists (e.g. albuterol) that increase potassium entry into cells, and insulin that increases potassium entry into cells. * Rare causes of hypokalemia include hyperaldosteronism, like Conn syndrome primary hyperaldosteronism from renal disorder, or Cushing syndrome from adrenal excess, or Bartter syndrome from volume depletion secondary to impaired renal resorption of NaCl. Other causes include licorice in high quantities due to mineralocorticoid effect of glycyrrhetinic acid.

* 73yo man with COPD and CHF has had diarrhea for 3 days. Now has increased SOB (shortness of breath). Meds include furosemide and albuterol. Received multiple albuterol treatments in the ED (emergency department). Identify three possible causes of hypokalemia. Causes include diarrhea with loss of potassium in the stool, the furosemide with loss of potassium in the urine, and the albuterol with increased cellular resorption of potassium. * Symptoms of hypokalemia if mild are asymptomatic, moderate include muscle weakness, muscle cramps, and non-specific T and U wave changes. With severe hypokalemia, cardiac arrhythmias occur. At any level of hypokalemia, digitalis/digoxin toxicity is potentiated. * Treatment of hypokalemia is done by replacing potassium. Whenever possible, replace orally. IV replacement maximum is 10mEq/hour via peripheral IV or 20mEq/hour via central line. An excessive rate causes cardiac conduction defects including heart block and/or asystole. Potassium is one of the chemicals used for lethal injection because it causes heart block and asystole. --------------------------------------------------------------------------------------------------Electrolyte Disorders: Hyperkalemia * Causes include increased potassium intake, such as an error in the hospital with excessive potassium administration. This is uncommon except in conjunction with certain medications (e.g. potassium sparing diuretics, ACE-inhibitors). Pseudohyperkalemia can occur as an artifact from venipuncture or cell lysis from shaking the blood tubes. Hemolysis of cells causes the potassium to be released from the cells, leading to falsely high potassium levels. This can also occur if blood is drawn proximal to a running IV drip that contains potassium. Movement of potassium from cells into the blood can cause hyperkalemia, occurs in acidosis (H+ into cells, K+ out), rhabdomyolysis, and rarely familial periodic paralysis. Another causes is deceased renal excretion from renal failure or medications like spironolactone and ACE-Is. Hypoaldosteronism and adrenal insufficiency are other cause. * Symptoms include weakness that increases with potassium level, sudden cardiac conduction defects preceded by classic ECG changes (talk pointed peaked T-waves). Patients needs treatment if ECG changes present. * 67yo woman with diabetes and hypertension is on glyburide and ACE inhibitor. She presents for a routine checkup. Labs reveal creatinine increased to 4.5 and potassium is 6.6. Why did her potassium become high? There are a number of reasons include the ACE-I, could be renal failure. How will you treat this? * Treatment should start with protection of the heart. Give CaCl or Ca++ gluconate infusion. Next step is to lower the potassium level by driving potassium into the cells. This is done with glucose and insulin, also with bicarbonate as the alkalosis drives potassium into cells. They still have potassium in their body but it is no longer in the circulation where it would induce cardiac arrhythmia. Last step is to get rid of the potassium from the body. This is done with cation-exchange resin (kayexalate) or dialysis if renal failure. ---------------------------------------------------------------------------------------------------

Acid-Base Disorders * A normal blood pH is 7.4 +/- 0.03. A pH < 7.37 is acidosis and pH > 7.43 is alkalosis. * Arterial blood gas (ABG) gives you pH, where you can determine if this is acidosis or alkalosis. You get PCO2 (normal 35-45) and should determine respiratory rate/volume, these changes are rapid. Hyperventilation lowers PCO2 and breath holding or shallow respirations will increase PCO2. You get HCO3 (normal 20-28), with bicarbonate working as a metabolic buffer, these changes are slower (2-5 days). ABG also gives O2 and PO2, which can vary by the altitude you live at. O2 sat typically in mid-90s, should be above 90% normally. * Respiratory alkalosis is caused by hyperventilation, anxiety (acute pain, pain attack), acute pulmonary disorder (pulmonary embolism, pneumonia), early aspirin toxicity. Drives down the PCO2 with no time for metabolic compensation. * Respiratory alkalosis will have increased pH, decreased PCO2, HCO3 unchanged because not enough time. * 24yo female comes to the ED feeling like she is suffocating, has tightness in her throat and chest. She appears anxious, RR is 36. ABG shows 7.52/25/PO2/27/99%. PCO2 at 25 is low. This is respiratory alkalosis, likely due to the hyperventilation. * What if the patient above had an O2 sat of 76%? This is not acute anxiety or panic attack. This is a lung problem. Could be a patient on birth control pills with acute pulmonary embolism. Either way it is a lung problem. * Metabolic alkalosis is either a loss of H+ or a gain of HCO3. Common causes of H+ loss include prolonged vomiting or other GI loss and diuretics (H+ and Clinterchange). Common causes of HCO3 gain include NaBicarb ingestion (multiple antacid pills) and milk-alkali syndrome (CaCO3). * Metabolic alkalosis is a chronic process, with compensatory slowing of respiratory rate and increase in PCO2. * Metabolic alkalosis has increased pH, increased HCO3, respiratory rate slows to raise PCO2 thereby increasing pH almost back to a normal value, never overcompensation. * 67yo woman with HTN is on HCTZ, complains of fatigue. Exam shows orthostatic hypotension. Electrolytes show low potassium, low sodium, low chloride. ABG is 7.42/47/PO2/33/97%. So PCO2 of 47 is high, and HCO3 of 33 is high. This is metabolic alkalosis defined by high pH and high HCO3. The PCO2 being high is respiratory compensation. * Respiratory acidosis caused by hypoventilation (acute or chronic), an inability to blow off CO2, and if sustained the kidneys will retain HCO3 in an attempt to lessen the acidosis. Acute causes include narcotics, acute COPD, or asthma. Chronic causes include COPD, obesity, Pickwickian (obesity hypoventilation syndrome), and sleep apnea. * Respiratory acidosis has decreased pH, increased PCO2, HCO3 unchanged because not enough time. HCO3 rises with time to compensate almost back to a normal pH.

* 27yo man found unconscious on the street. Respiratory rate is 7. Needle tracks on arms. Pupils small. ABG shows 7.28/56/PO2/28/84%. This is a narcotic overdose with respiratory acidosis due to hypoventilation. * 72yo woman has COPD worsening over weeks. Comes to the ED because she is becoming gradually SOB. ABG shows 7.37/5/PO2/34/90%. Her PCO2 is high, so likely a CO2 retainer with the COPD. How do we know this is not primary metabolic alkalosis (i.e. inc HCO3 with inc PCO2 compensating)? We know this because the pH of 7.37 is not alkalosis; the body does not overcompensate. * Metabolic acidosis is the most complicated of the group. * Metabolic acidosis is a relative increase in the quantity of acid, with causes including addition of acid, inability to excrete acid, and loss of base (HCO3). * Metabolic acidosis has decreased pH, decreased HCO3, and increased respiratory rate as compensation in an effort to blow off PCO2 thereby increased pH almost back to normal, occurs rapidly. * 17yo with diabetes has not been taking his insulin, has altered mental status. ABG shows 7.29/29/PO2/17/97%. This is metabolic acidosis as bicarb is low. As compensation, body is breathing fast to bring down PCO2. * Must determine which type of metabolic acidosis, either high-anion gap (anion gap) or normal anion gap (nonanion gap). Anion gap is normally 11 +/- 3, so 8-14. Anion gap = (Na + K) (Cl + HCO3). * High anion gap acidosis is an excess of non-HCl acid. Normal anion gap acidosis is excess HCl or loss of HCO3. * Anion gap > 14 caused by overproduction or under-excretion of an acid other than HCl. Chloride levels are unaffected normochloremic. There are four major causes: ketoacidosis (alcoholic, diabetic, starvation), lactic acidosis, renal failure (uremia), or intoxication (ethylene glycol, methanol wood alcohol, salicylate, others). * MUDPILES: methanol, uremia, diabetic ketoacidosis, paraldehyde/phenformin, iron/INH, lactic acidosis, ethylene glycol, salicyclates. * Ketoacidosis is due to overproduction of ketone bodies. Lactic acidosis is due to overproduction of lactic acid. Renal failure causes inability to excrete organic (nonHCl) acids. Intoxication causes addition of non-HCl acids. * Anion gap < 14 (normal) usually caused by HCO3 loss in GI tract or urine or inability to excrete acid in urine. Common causes are diarrhea and renal tubular acidosis (RTA). * Renal tubular acidosis key clue is the inability to acidify urine, urinary pH typically > 5.4 or even overtly alkaline. * Type 1 RTA occurs in distal tubules of the kidney. Inability to acidify urine. Seen in kidney stones, amphotericin B, lithium, sickle cell. Diagnosed by giving acid load/challenge test and still cant acidify urine. Treatment is oral bicarbonate. * Type 2 RTA occurs in the proximal tubules of the kidney. Inability to absorb bicarbonate until levels very low. Seen in myeloma, Fanconi syndrome, vitamin D deficiency, Wilson disease. Diagnosed by seeing bicarbonate lost in urine in the face of acidic blood. Treatment is volume restriction.

* Type 4 RTA occurs due to hypoaldosteronism. Occurs with aldosterone or adrenal insufficiency and diabetes. Presents with hyperkalemia. Diagnosed by high urine sodium with salt restriction (salt restriction test). Treatment is mineralocorticoid replacement, fludrocortisone. * 72yo man has 4 days of diarrhea and presents to ED with weakness. ABG shows 7.31/30/PO2/17. Lytes are Na 140, K 2.9, Cl 113, HCO3 18. Is this acidosis or alkalosis? Answer is acidosis. Is this metabolic or respiratory? Answer is metabolic acidosis, PCO2 being low is compensatory. What is the anion gap? Gap is 12, so normal anion gap (11 +/- 3). What is the most likely cause of the this patients normal anion gap metabolic acidosis? Most likely diarrhea. Could be RTA. What would be the best test to distinguish diarrhea from RTA? Answer is urine pH, which should be acidic (< 5.4) in the presence of blood acidemia; urine would be inappropriately high pH in RTA. --------------------------------------------------------------------------------------------------Kidney Stones (Nephrolithiasis) * Urolithiasis occurs in 1-5% of the US population, more common in desert southwest (e.g. Arizona, Nevada). * Five kinds of stones: calcium oxalate (most common), calcium phosphate, magnesium/ammonium/phosphate (triple phosphate or struvite), uric acid, cysteine (rare). * Oxalate stones account for about 70%, phosphate about 10%, struvite about 5-10%, urate about 5%, cysteine rare. * Oxalate, phosphate, and struvite have calcium and are radio-opaque. Urate may not be seen on plain x-ray. * Main causes of calcium oxalate/phosphate stones include increased calcium absorption (sarcoid, vitamin D intoxication), idiopathic hypercalciuria, hyperparathyroidism, destructive bone disease (myeloma, metastatic cancer), hyperoxaluria (fat malabsorption as fat binds to calcium, leaving oxalate for excessive absorption), vitamin C excess. * Main causes of struvite stones include infection with urea-splitting bacteria (proteus most common, klebsiella, pseudomonas), alkaline urine favors crystallization of struvite and can result in large stones (staghorn calculi). * Main causes of uric acid stones include gout, tissue breakdown (e.g. hematologic malignancies), are radiolucent and may not be seen by CT or other x-ray. * Kidney stones present with severe flank pain radiating to the groin, pain onset often sudden, pain may be constant or spasmodic, nausea and vomiting common. Pain medication may be one of the first priorities. * 35yo male has sudden onset of severe flank pain radiating to groin, with nausea and vomiting. What is the best first test to support the diagnosis of kidney stone? Answer is urinalysis for hematuria. The absence of hematuria argues strongly against kidney stone. What is the next step in determining diagnosis, or what is the best test to confirm diagnosis? Answer is non-contract helical (spiral) CT scan. * Spiral CT is the test of choice as it is fast, most sensitive, identifies radio-opaque and even some radiolucent stones. KUB (kidneys, ureter, bladder x-ray) is useful for

patients with known stone disease, difficult to interpret as gas may be in the way. Sonogram is the test of choice for identifying stones in pregnant women. IVP (intravenous pyelogram) requires contrast, no longer test of choice, bowel prep needed for good pictures. * 26yo female presents with sudden onset of severe flank pain radiating to groin, with nausea and vomiting. Urinalysis shows hematuria. What is the next step in the diagnostic evaluation? Answer is pregnancy test. If pregnancy test is positive, then do sonogram. If negative, do spiral CT. * Say the 26yo female is not pregnant and the spiral CT shows a small radiolucent stone in the ureter. What is the most appropriate initial treatment? Answer is analgesics (e.g. morphine) and high flow IVF. * For small stones (<5mm) in lower ureter is observation. Small stones at the ureteropelvic junction (UPJ) almost always pass. Large or with significant obstruction requires surgical intervention. * Ureteroscopy involves putting cystoscope through urethra up into bladder then fragments stones and pulls them out with forceps or baskets. * Lithotripsy (ESWL) can be used for small renal (<2.5cm) and ureteral (<1.5cm) stones. * Percutaneous nephrostomy is used for obstructed stones to drain the kidney via a tube and prevent backflow. It can be combined with lithotripsy. * For complex stones (e.g. struvite) or extensive anatomic abnormalities, open surgical removal is performed. * You should always try to determine the type of stone the patient has. To do this you strain the urine and send for urinalysis. So the patient will urinate at home into a funnel until a stone or particles are found. * Initial tests in a stone workup include looking for increased calcium absorption (24h urine calcium), idiopathic hypercalciuria (24h urine calcium), hyperparathyroidism (serum Ca level, PTH level), destructive bone disease (serum Ca level), fat malabsorption (history, greasy stools), vitamin C excess (history). * Treatment for increased calcium absorption (thiazides), idiopathic hypercalciuria (thiazides), hyperparathyroidism (treat problem), destructive bone disease (treat problem), fat malabsorption (treat problem), vitamin C excess (discontinue vitamin C). --------------------------------------------------------------------------------------------------Renal Cysts * Three types of cystic kidney disorders: polycystic kidney disease (PKD) in autosomal dominant form (1:300 to 1:1000) and autosomal recessive form (rare, infancy), simple cysts (incidental), complex cysts (rule out cancer). * PKD is autosomal dominant in 90% of cases, 10% are sporadic mutations without family history. * PKD causes 5-10% of end stage renal disease needing dialysis. Cause is uncertain. * PKD involves multiple renal cysts, asymptomatic in childhood, symptoms develop in 20s and 30s including pain from kidney size pushing on renal capsule, hematuria, infections, stones, high blood pressure via RAAS.

* PKD manifests with hypertension in the kidneys, impaired circulation due to distortion of renal vasculature leads to increased renin-angiotensin. Renal failure occurs with progressive decline in function, half have ESRD by age 60. Stones and infection also occur. * Extra-renal manifestation of PKD include hepatic cysts (50%), also pancreas, ovaries, spleen cysts. Intracranial aneurysms (10%) berry aneurysms that can lead to subarachnoid hemorrhage. Mitral valve prolapse (25%). Colon diverticula more often than in general population. * PKD diagnosed with renal ultrasound, diagnose symptomatic patients, screening of relatives at risk, only 2/3 have cysts in their teens, cysts not universally present until about 30s. * Genetic diagnosis can be done with prenatal amniocentesis and chorionic villus sampling (CVS). For adults and children at risk, gene linkage studies are useful if considering kidney donation to affected relative. * Treatment of PKD is manage complications, control hypertension, treat UTIs/stones, dialysis/transplant for ESRD. * 32yo man is seen for hypertension and hematuria. Tells you that father and cousins had a similar problem and both died of hypertension-related renal failure. Creatinine is 1.5. What is the best test to make a diagnosis? Answer is renal ultrasound to rule out PKD. * Simple renal cysts are usually incidental findings on sonogram or CT. Can be single or multiple, usually single. Is a homogenous cysts without echos (fluid filled sac). No treatment is needed unless large or symptomatic. * Complex renal cysts are usually incidental findings on sonogram or CT, or found during workup of hematuria or abdominal pain. By definition, a complex cyst is cancer until proven otherwise. Must do a percutaneous aspiration biopsy (CT needle aspiration), regardless of size. * Complex renal cysts vary in size, multiple (complex) echos/septations seen on CT or ultrasound. * Again, complex renal cysts are cancer until proven otherwise and need CT-guided needle biopsy. * 62yo man is being evaluated for RUQ abdominal pain. Sonogram to study gallbladder reveals gallstones, but also a small (2cm) cyst on left kidney with complex echos. CT scan confirms findings. Patient has no symptoms from left kidney, and no hematuria. What is the next step in the evaluation? Answer is cyst aspiration/puncture. --------------------------------------------------------------------------------------------------Hypertension (HTN) & Hypertensive Emergencies * Optimal is systolic < 120 and diastolic < 80. Normal is systolic < 130 and diastolic < 85. * Hi-normal is systolic 130-139 or diastolic 85-89. * Stage 1 hypertension is systolic 140-159 or diastolic 90-99. * Stage 2 hypertension is systolic 160-179 or diastolic 100-109. * Stage 3 hypertension is systolic 180+ or diastolic 110+.

* Epidemiology shows 50million Americans with HTN, that is 1/4 of the adult population. 60% of persons over age 65 have HTN and 75% are not controlled. * High risk groups for HTN include African Americans, obese, diabetics, and males > females. * Most common presentation is incidental finding in an asymptomatic individual. Hypertensive emergency is uncommon. Chronic complications can develop; patient may present with heart attack or retinal haemorrhage. * Chronic complications include heart disease with >1million MIs per year, 400,000 new CHF per year. * Chronic complications include CVA (400,000 strokes/year), renal failure (5,000 ESRD/year), eye disease. * Essential hypertension has no clearly identified cause or reversible process. < 5% of HTN is secondary, due to some other condition that is often reversible. * BP measurement method: patient resting quietly for 5 minutes, seated with arm at heart level, bladder encircles 80% or more of arm, average of 2 readings, confirm on 2 subsequent visits. * White-coat hypertension is thought to occur in about 20% of cases of diagnosed HTN. * Ambulatory BP monitoring gives data for a 24h period, useful for suspected white coat hypertension. * After you get a first blood pressure reading, when do you have to get the patient back for a confirmation? If BP systolic > 180 or diastolic > 110, then 1 week. If BP systolic 160-179 or diastolic 100-109, then 1 month. If BP systolic 140-159 or diastolic 90-99, then 2 months. If BP systolic 130-139 or diastolic 85-89, then yearly. If BP systolic < 130 or diastolic < 85, then every other year. * Hypertensive emergencies need immediate treatment: angina or CHF, cerebral impairment (ischemia, subarachnoid hemorrhage, severe headache), acute eye findings (hemorrhages, papilledema, blurred vision). * Treatment for hypertensive emergency is nitroprusside infusion. Also nitroglycerine (with MI), labetalol, enalapril. * 32yo previously normotensive man sees you for a work physical, BP is 170/99. His history and exam are otherwise unremarkable. What is the next step in evaluating his blood pressure? Answer is return in 1-2 months. * Say the patient comes back and their HTN is confirmed. Perform a physical exam and laboratory investigations to seek secondary causes, seek end-organ damage, and seek/control atherosclerotic (ASHD) risks. * Investigating secondary causes can be done by history, exam, and basic labs. Medication causes include birth control pills and alpha-agonists (e.g. seasonal rhinitis spray). Hyperthyroidism can cause HTN (tremor, goiter). Coarctation of the aorta can be found by pulse lag, pulse striking earlier in radial than femoral arteries. Renal artery stenosis is found by bruit on abdominal exam or sudden abrupt onset of HTN. Polycystic kidneys can be found by mass or hematuria. Cushing disease will have striae, buffalo hump, rounded moon-facies. Hyperaldosteronism will come with low potassium. Pheochromocytoma presents with sympathetic episodes. Renal

failure can be discovered by creatinine level elevation. Aortic regurgitation has high systolic pressure and low diastolic pressure, murmur. * So for secondary HTN causes, do history, physical, creatinine, potassium. * What is the best test if a secondary cause is suspected? Medication-induced is stop meds and follow BP, hyperthyroid is ultra-sensitive TSH, coarctation is ultrasound studies, renal artery stenosis is captopril renogram in young (fibromuscular) and Doppler flow in elderly (atherosclerotic), polycystic kidney is renal sonogram, Cushing is dexamethasone suppression test and/or cortisol, aldosteronism is aldosterone level, pheochromocytoma is VMA and metanephrines and catecholamines, renal failure is BUN/Cr, aortic regurgitation is echocardiogram. * What is the initial treatment if a secondary cause is suspected? Medication-induced is stop meds and follow BP, hyperthyroid is beta-blockers and anti-thyroids, coarctation is surgical, renal artery stenosis is angioplasty, polycystic kidney is ACE-I (maybe) and renal transplant, Cushing is remove pituitary adenoma, aldosteronism is remove aldosterone tumor, pheochromocytoma is alpha-blockade and surgery, renal failure is renal transplant, aortic regurgitation is ACE-I and valve replacement. * 45yo previously normotensive man sees you for work physical and BP is 170/99. Multiple BP readings over 2-3 months confirm hypertension. History is unremarkable except vague LUQ abdominal pain for past 6 months, exam shows LUQ fullness. Urinalysis shows hematuria. Other labs are normal including creatinine. What is the most appropriate test for a secondary cause of hypertension? Answer is renal ultrasound to rule out polycystic kidney. * End-organ damage can be investigated by looking for retinopathy, carotid bruits, CHF, ECG (vs echo vs nothing) for left ventricular hypertrophy, AAA (exam vs sono), lower extremity pulses. * Risk factors for atherosclerotic heart disease (ASHD) are smoking, sedentary lifestyle, obesity, diabetes, hyperlipidemia, newer risk factors (C-reactive protein, homocysteine levels). * Homocysteine levels can be lowered with vitamin B6, vitamin B12, folic acid, no evidence it improves heart disease. C-reactive protein level also not proven to be linked to heart disease, but being investigated. * 45yo previously normotensive man sees you for work physical and BP 165/92. Multiple BP readings over 2-3 months confirm hypertension. History reveals 1PPD smoker for 30 years, otherwise no ASHD risk factors. Physical exam and all lab tests are unremarkable. What is the best initial step in his treatment? Answer is smoking cessation. * Hypertension treatment initially for stage 1 and maybe stage 2 is lifestyle modification: weight reduction, exercise, smoking cessation, salt reduction, possibly alcohol reduction is in large amounts. * Medications are first-line for uncomplicated essential hypertension (stage 3, most stage 2, stage 1 if it does not respond to lifestyle modification). Diuretics (thiazides) are first or beta-blockers.

* Special circumstances: ACE-I used in diabetes, mild renal insufficiency, CHF due to systolic dysfunction, or recent MI. ACE-I will improve renal blood flow a bit and lower risk of progressive renal damage. * Calcium-channel blocker used in isolated systolic hypertension in elderly, possibly better than beta-blockers in African American population if adding to diuretic. * Double-duty situations: calcium-channel blocker for Raynaud, SVT, atrial fibrillation. Alpha blocker for prostatic hypertrophy. Beta-blocker for migraine headache. * Hypertension in pregnancy, recommended drugs: methyldopa is first line. Labetalol or hydralazine also acceptable. Contraindicated drugs include ACE-I and angiotensin receptor blockers (ARBs), teratogenic. * What if you start treating a patient and the hypertension is not responding? Most common cause is noncompliance, patient may be confused about treatment regimen. Consider alcohol abuse and NSAIDs. Re-consider secondary causes. * Osler phenomenon, usually in elderly, is artery calcification that prevents full compression of the artery by the blood pressure cuff, thus giving a falsely high reading. Clue to this palpable artery with inflated cuff. --------------------------------------------------------------------------------------------------Kaplan Videos (2001) Infectious Disease with Dr. Conrad Fischer, MD --------------------------------------------------------------------------------------------------The organisms that cause certain diseases do not change over the years. The antibiotics do however. --------------------------------------------------------------------------------------------------Overview of Antibiotic Therapy * Most common cause of pyelonephritis is E. coli, most common cause of osteomyelitis is S. aureus. * So what antibiotics go along with staph/strep, gram negative rods, and anaerobes? * If you do PCR resting on Egyptian mummies from 5 thousand years ago, you will find there was beta-lactamase in the staphylococcus. * Ampicillin, penicillin, amoxicillin alone is good for streptococcus, but not for staphylococcus. * Antibiotics for staphylococcus and streptococcus are oxacillin, cloxacillin, dicloxacillin, nafcillin (not methicillin). * With methicillin-sensitive staphylococcus, what do you use? Not methicillin. Use ox, clox, diclox, naf. Why dont we use methicillin? Because it causes interstitial nephritis. * Another alternative for staphylococcus and streptococcus (e.g. penicillin allergy) is a beta-lactam drug, first generation cephalosporins like cephalexin, cephadroxil, cefazolin. 5% cross-reaction with penicillins at most and almost never anaphylaxis, so dont worry if the allergy is a little rash. * What if patient says they have a penicillin allergy, that causes a little rash, and hypotension, and stridor, and laryngeal edema so they had to get intubated, then got IV steroids, then dopamine drip in the ICU. For life threatening penicillin allergies, then we should avoid all the beta-lactam antibiotics.

* Macrolides: erythromycin, azithromycin, clarithromycin. Macrolides with clindamycin will cover staph/strep, but are not first choice, choice for lifethreatening penicillin allergy. Vancomycin works, but should be reserved for methicillin-resistant staph aureus (MRSA). Vancomycin not superior in efficacy to ox/clox/diclox/naf, just superior in its resistance pattern. * Gram negative rods include E. coli, klebsiella, proteus, enterobacter, citrobacter, pseudomonas. * Antibiotics for Gram-negative rods are aztreonam, aminoglycosides (gentamycin, streptomycin, tobramycin, neomycin, amikacin), fluoroquinolones (ciprofloxacin, levofloxacin, trovafloxacin, gatifloxacin, sparfloxacin, moxifloxacin), secondgeneration cephalosporins (will not cover pseudomonas), third-generation cephalosporins (ceftazidime, ceftriaxone), carbapenems (imipenem, meropenem), extended spectrum penicillins (piperacillin, ticarcillin, azlocillin, mezlocillin). * Carbapenems cover Gram-negative and Gram-positive, but you should not use them for Gram-positive infections (like strep throat) because there are better drugs exclusively for Gram-positives. Do not swat a fly on your friends head with a hammer. * TMP-SMX is only good for uncomplicated cystitis and prophylaxis against pneumocystis pneumonia. * Chloramphenicol is never the right answer, it causes aplastic anemia and Gray Baby Syndrome. * Cephalosporins range from strong Gram-positive coverage at first-generation (poor Gram-negative) to strong Gram-negative coverage at third-generation (poor Grampositive). Fourth-generation cephalosporins (cefepime) are a combination of firstgeneration and third-generation, so they cover Gram-positives and Gram-negatives. * Antibiotics for anaerobes are metronidazole (particularly good for abdominal), clindamycin (does not cover bowel well), carbapenems as well. If you have an exclusively anaerobic infection, dont jump to imipenem. * Tetracycline is not the correct choice anymore; use doxycycline instead. Doxycycline used for Lyme (mild disease: rash, facial nerve palsy), Chlamydia, rickettsia, as these are intracellular organisms. * Nitrofurantoin used for UTI in pregnancy, no other use. --------------------------------------------------------------------------------------------------Meningitis * 57yo man comes to the ED with fever and a headache. Photophobia or seizures or nausea or vomiting are too nonspecific here. A specific additional symptom to lead you down the right path would be stiff neck (nuchal rigidity), implying meningitis. What if they want you to know its a brain abscess? Fever, headache, and focal findings. What about encephalitis? Fever, headache, altered mental status (confusion, encephalopathy). * So 57yo man has fever, headache, stiff neck. What is the first test? Do you do a lumbar puncture or a CT scan? What is the first test? Lumbar puncture (LP). Only do a CT scan prior to a spinal tap if papilledema or focal findings that would imply a brain mass/abscess. Patient also needs to be able to follow commands, such as

squeeze my hands, because you cannot determine focality. If you had a mass lesion that was so small it did not cause focality, an LP would not cause herniation. * If you are going to do a CT prior to the LP, you should start the patient on antibiotics first (dose of ceftriaxone). Doesnt that ruin the sensitivity of the LP culture? Yes, but youd rather be alive with a clean LP than dead with an accurate LP. Other information from the LP can help tell you the diagnosis. * The most common neurologic problem after meningitis is deftness, CN VIII damage, it can happen within hours. * With papilledema you worry about herniation. You go to do a rectal later and say my thats a smooth prostate and the patient says, thats not my prostate, thats my pons. * When you have infectious diseases, what is the most accurate diagnostic test? Answer is culture. All other tests are compared to culture. When do you wait for results of the culture? Never. * CSF analysis: Which form of meningitis can give an elevated protein and decreased glucose in the LP? Answer is all. Which gives it most often? Bacterial, strep pneumonia. Gram-stain is 50-60% sensitive, so wont be positive most of the time. So what will tell you right now what type of organism you have? Answer is cell count. * CSF cell count: Only bacterial gives you 1000s of polys, where others give 10-100s of lymphocytes. If you see 1000s of polys, give ceftriaxone. What about 10-100s of lymphocytes? Could be fungal (cryptococcus), Rocky Mountain Spotted Fever, Lyme disease, syphilis, tuberculosis, viral meningitis. * For cryptococcus meningitis, expect HIV patient with < 50 CD4 T-cell count. Best initial test? Answer is not India Ink. However, India Ink is about 50% sensitivity (good if positive, means nothing if negative). Answer is cryptococcal antigen testing, positive in 95-98%, very sensitive and specific. Should you start this patient with fluconazole? No, use amphotericin B. Amphotericin B (better than fluconazole for saving life in meningitis), then follow up with fluconazole forever. Fluconazole is continued life-long, else cryptococcal meningitis recurs. * Rocky Mountain Spotted Fever (RMSF) is not seen on Gram stain because it is intracellular (rickettsia). Lyme also not seen on Gram stain because it is a spirochete. You also cant see TB, viral, Legionella, or syphilis on Gram stain. * So, CSF from lumbar puncture shows 10-100s of lymphocytes, Gram stain negative, culture negative. Now what? Answer is serologic testing to look for specific antibody against Lyme and RMSF. This helps you determine if you should use doxycycline or ceftriaxone. So this is very easy, right? We just get some IgM acutely and IgG chronically. Remember the Weil-Felix test? Forget it. * So are with atypical results (Gram stain negative, culture negative), are we just going to order Lyme, RMSF, cryptococcal antigen testing, AFB (acid-fast bacillus) stain and culture, viral serologic testing, syphilis serology with a VDRL and FTA on all of them? Nope. You have to suspect the specific disease. * If RMSF, centripetal (moves centrally) rash and outdoor activity. If Lyme, target rash (erythema migrans) and outdoor activity. Lyme in Connecticut, Massachusetts, New York, New Jersey (North East). RMSF in Alabama, Kentucky, Tennessee,

Carolina, it shouldnt matter, look for the centripetal rash. If TB, diagnose with TB stain and AFB cultures. If viral, diagnosis is by exclusion of the others, there is no treatment. * Test TB (meningitis and pulmonary) with TB stain, TB culture, INH, rifampin, pyrazinamide, ethambutol. * When do you answer steroids for meningitis in adults? Answer is TB meningitis. Some mild benefit possibly with steroids given prior to starting antibiotics and LP. * When do you answer intrathecal antibiotics? Answer is never, not necessary. Methotrexate is given intrathecal in ALL (acute lymphoblastic leukemia). Intrathecal antibiotics is like intra-cardiac epinephrine, nifty idea but not necessary. * Elderly neonate is HIV positive, on steroids for CLL, has a lumbar puncture with elevated protein and decreased glucose, Gram stain negative, 3200 polys. What is the next best step in the management of this patient? Do we give amphotericin or ceftriaxone? HIV is a risk factor for fungal meningitis. This isnt fungal because there are polys present. This patient is immunocompromised. So what organism do we have to cover? Answer is listeria. Answer for management is give ceftriaxone and ampicillin, because listeria is resistant to all forms of cephalosporins. * Things that put you are risk for listeria include immunocompromised, neutrophil and T-cell defects, HIV (T-cell defect), steroids (neutrophil and T-cell defect), leukemia/lymphoma elderly or neonate then do empiric coverage. --------------------------------------------------------------------------------------------------Encephalitis * Fever, headache, and confusion (encephalopathy). What is the next step? Head CT. The head CT will most likely be normal. Lots of encephalitis in the world, eastern equine encephalitis, western equine encephalitis, Colorado tick fever, Congo Crimean fever, Venezuelan hemorrhagic fever, Bolivian hemorrhagic fever, viral encephalitis. * If you add all those cause of encephalitis and multiply by ten, they still are not as common as herpes encephalitis. If the case is clearly encephalitis and they ask diagnosis, your best bet is to answer herpes. * Lumbar puncture should come with PCR, which has largely replaced the biopsy. What is suggestive of herpes on a lumbar puncture? Answer is increased red cells, mildly hemorrhagic. How do you know it is not a subarachnoid hemorrhage? There will also be increased WBCs from infection, plus clear CT scan favors against subarachnoid. * Treatment of herpes encephalitis is acyclovir. --------------------------------------------------------------------------------------------------Brain Abscess * Fever, headache, and focal findings. Next step is CT scan (or MRI). Head CT will show ring- or contrast enhancing lesions. Two groups of diseases that give ringenhancing lesions are cancers and infections. Staph, strep, glioblastoma, anaerobes, toxoplasmosis, oligodendroglioma, astrocytoma. * Next step depends on HIV status. If patient is HIV positive, assume toxoplasmosis and give pyrimethamine and sulfadiazine for a couple of weeks and repeat CT scan.

In HIV, 90% of the time it is toxoplasmosis or lymphoma. If the CT scan is the same, do a biopsy to see what is at the ring-enhancing site. * If patient is HIV negative, much broader range of disease. Next step is not lumbar puncture because the patient will herniate and die. Next step is brain biopsy; the only way to know what it is. * So again, we have a lesion with mass effect giving focal findings. Do not do LP. Do a CT to see ring-enhancing lesion. Then do a biopsy. You dont want to start antibiotics and have the patient come back with a permanent deficit saying, doc, I heard antibiotics dont work well for glioblastoma. Or you radiate the patient for 6 weeks and they say, doc, the radiation isnt working for my staph aureus. You have to biopsy to know. --------------------------------------------------------------------------------------------------Otitis Media & Sinusitis * What is the most sensitive physical finding for otitis media? Answer is immobile tympanic membrane on insufflation. You may see a red bulging tympanic membrane, occasionally fever, pain on pulling pinna. * Treatment is amoxicillin, based on American Society of Otolaryngology, half the time the otitis media resolves on its own because they are viruses, if you havent recently been treated (i.e. resistance) the efficacy of amoxicillin is just as good as any of the other antibiotics. * Second and third generation cephalosporins, cefuroxime, cefixime, cefaclor would work. Macrolides azithromycin, clarithromycin work. Amoxicillin/clavulanic acid works too. If not child, new fluoroquinolones like levofloxacin, moxifloxacin, gatifloxacin work as they cover strep pneumonia. * 28yo female with headache, rhinorrhea, sinus tenderness, tooth pain, decreased transillumination of sinuses. What is the next step in management? Answer is give amoxicillin. * Acute otitis media (AOM) is about 40% strep pneumonia, 30% h. influenza, 20% moraxella catarrhalis. * What is the most accurate diagnostic test for this sinusitis patient? Answer is culture. * The best initial diagnostic test is an x-ray. A CT is more accurate than an x-ray, but not first. --------------------------------------------------------------------------------------------------Pharyngitis * Symptoms are sore throat, adenopathy, and exudate. Answer for treatment is oral penicillin, Pen VK. Most accurate test is Gram stain. Why do you not do a Gram stain for pharyngitis? It will not tell you if you have strep pyogenes; it cant tell you what type of strep is there. * The rapid strep tests are latex agglutination tests, looking for the surface antigens of strep pyogenes. * Why dont we Gram stain the vagina? Gardnerella is normal flora, so we need clue cells to see when there is too much. We Gram stain sputum because strep pneumonia

with lancet-shaped diplococci are not normal flora in the mouth. We do Gram stain the cervix to look for neisseria gonorrhea. * If patient is allergic to penicillin, what do we use for pharyngitis? Second or third generation cephalosporins, azithromycin, clarithromycin, levofloxacin, moxifloxacin, gatifloxacin. * Erythromycin has an advantage over the others as it can prevent rheumatic fever. --------------------------------------------------------------------------------------------------Bronchitis & Pneumonia * Fever, cough, with or without sputum. You cant say this is influenza, pneumonia, bronchitis. All you know is this is a respiratory infection. What test should you order first? Answer is x-ray, first for respiratory infections. If CXR is normal (no infiltrates, effusions, consolidations), then this is bronchitis. * Treatment for bronchitis (with sputum) is second or third cephalosporins, azithromycin, clarithromycin, levofloxacin, moxifloxacin, gatifloxacin. Ampicillin would work also. * What is the best diagnostic test? Answer is biopsy/aspiration. * How do we know if the patient has an abscess? Halitosis due to anaerobes. To get an abscess, the patient must have an increased risk for aspiration and an oral infection. * Intubation aspiration, stroke, poor dentition, intoxicated, loss of gag reflex, you increased your risk of aspiration. * For anaerobes above the diaphragm we use clindamycin. For below the diaphragm we use metronidazole. * Why dont we Gram stain or culture sputum for an abscess? Everyone has anaerobes in their sputum. * How do we determine the types of pneumonia if infiltrates are found on CXR? There will be lobar infiltrates or bilateral interstitial infiltrates. * Lobar infiltrates is regular bacterial pneumonia, strep, hemophilus, staph, klebsiella. If bilateral interstitial infiltrates, could be mycoplasma, chlamydia, legionella, viral pneumonia, pneumocystis pneumonia. * Young healthy person admitted for lobar pneumonia is pneumococcus. Elderly patient with lobar pneumonia is pneumococcus. Elderly patient with lobar pneumonia and has COPD or smokes often, then it is strep pneumonia. * Pseudomonas pneumonia seen in the ICU in intubated patients, hospital acquired infection. * When do you wait for the results of culture? Answer is never. Get culture and start antibiotics. * Why is sputum stain not first? Even when you do the culture, you dont get the diagnosis half the time. * In long-term alcoholics with lobar pneumonia, most common is strep pneumonia. In smokers and COPD patients with lobar pneumonia, most common is strep pneumonia. Klebsiella pneumonia is more likely to be seen in alcoholics (but not most common). H. influenza most likely to be seen in smokers and COPD patients (but not most common).

* Mycoplasma pneumonia usually seen in young, healthy, patient, walking pneumonia. Mycoplasma not seen on Gram stain (no cell wall) and not grown in regular culture. Atypical means it is not seen on a Gram stain and not grown in regular culture. Look at IgG and IgM serology. Cold agglutinin testing for mycoplasma is not sensitive or specific enough, so not a good way to make a diagnosis. * Treatment for mycoplasma or legionella is macrolide (Erythromycin, Azithromycin, Clarithromycin) or quinolone. Doxycycline also works. * Patient has bilateral interstitial infiltrates. Test for legionella pneumonia (diagnosis) is urine antigen. Urinary antigen for legionella is 99% sensitive and specific. If you dont screen for it with the test, you will not see the diagnosis. It accounts for about 5-10% of community acquired pneumonia. Many people will not think about ordering this type of test unless they know it exists. A patient presents with lots of coughing and youre getting a urine test; it doesnt seem logical unless you understand the test. * How will you know who to do a legionella urinary antigen test for? Not a person exposed to air conditioning or a person exposed to water. Look for a patient with GI and CNS problems; confusion, diarrhea, abdominal pain. * Most specific test for legionella is buffered charcoal yeast extract agar (BCYE), but it takes a couple of weeks. Antibody testing also takes weeks and is never available to make a treatment decision. * What is the fastest way to increase the incidence of sexually transmitted diseases in your community? Answer is bring in a STD specialist into your community, theyll test everyone, and the incidence will rise. People will be like, Hmm, ever since this new guy came to town everyone is getting Gonorrhea... * Any cause of pneumonia can cause hyponatremia. Anything in the lung can cause SIADH. Anything in the brain can also cause SIADH. * With pneumocystis pneumonia, patient is HIV positive with < 200 T-cells. Presenting symptom is usually dyspnea. CXR shows bilateral interstitial infiltrates. PO2 is 65. What is the next best step in the management of this patient? Answer is oxygen, then give TMP-SMX (co-trimoxazole). Steroids will increase the inflammation and thus increase the ability to transfer oxygen across the interstitial membrane. But isnt it dangerous to give steroids to someone who is immunocompromised? Maybe, but isnt it better than dying of hypoxemia today? Youd rather be alive with re-activated TB then dead with no TB. Also, short-course steroids for 2-3 weeks will not hurt. * Steroids used if hypoxemia is severe. Severe is PO2 < 70. * On hospital day 2, this HIV patient develops the most common side-effect of IV TMP-SMX. That side-effect is a diffuse maculopapular hypersensitivity rash. How what do you put the patient on? Answer is pentamidine. What is the major sideeffects of IV pentamidine? Answer is pancreas problems. Pentamidine is only second-line if patient has IV therapy. For prophylaxis, the second-line is dapsone. But there is no IV dapsone.

* Patient has bilateral infiltrates after sniffing a placenta, has cough. Sniffing a sheep (Yemenite sheep) placenta. Serology diagnoses Q-fever (coxiella burnetii). Treatment is doxycycline. Coxiella is the only airborne rickettsia. You would need the animal exposure history. So people around animals at the time or birth, the coxiella aerosolizes at the time of birth and they get the pneumonia. Without that history, you cant figure it out. --------------------------------------------------------------------------------------------------Tuberculosis (TB) * 37yo male prisoner at maximum security prison. There are over 2 million in prison in the U.S. Up to 60% of prisoners are PPD positive (tuberculosis), due to ventilation and crowds that you get there. Patient has fever, cough, sputum, weight loss, night sweats. What is the next best step in the management of this patient? Answer is CXR. It shows an apical cavity and infiltrate. You put the patient in isolation. Now, what is the next best step? Answer is sputum, not PPD skin test. You check sputum because PPD is not to diagnose acutely ill patients. A interferon gamma quantification test (QuantiFERON-TB) may be used. PPD is used for screening the asymptomatic, for screening populations at risk. * Say sputum is positive for AFB (acid-fast bacteria) stain. What is the next best step? Answer is treatment. How many drugs should patients be started on? 4: isoniazid (INH), rifampin (RIF), pyrazinamide (PZA), ethambutol. You give 4 drugs because of the possibility of resistance. * Which TB drug cause hepatotoxicity? Answer is all of them. * Which TB drug causes hyperuricemia? Pyrazinamide. What do you do for this patient? Answer is nothing in asymptomatic hyperuricemia, at any time. * Ethambutol affects what organ? Eye, optic neuritis. Ethambutol, Eye. * Abnormal red color with all your body fluids with rifampin. Red, Rifampin. * Neuropathy occurs with isoniazid. Neuropathy, iNh. INH causes increased urinary loss of vitamin B6. * Tuberculosis is essentially an economic disease. The amount of TB in a country is proportional to the amount spent on public health. * If your PPD is positive, what does this mean? It means you have been exposed. How much risk do you have of developing the disease? Answer is 10% in a lifetime. Except in HIV, where it is 10% per year. That means 90% of people who are PPD positive, never develop TB. That means 10 people will get isoniazid for 9 months to prevent that one single case. * What is a positive PPD? 10mm of induration. What about 10mm of erythema? No. What about 20mm of erythema? No. What about 2 meters of erythema? Nope. Induration is the only important point. Except 5mm induration in HIV positive patients because immune system cannot make it to 10mm. * What effect does BCG (Bacillus Calmette-Gurin vaccine) have on these recommendations? None. What? If Im from India I dont want to eat isoniazid for 9 months, thats for sick people, not me. Sorry, it doesnt matter no matter how much you think it does. BCG will not give you 10mm of reactivity.

* PPD is 5mm induration in HIV, close contacts, steroid use, organ transplant recipients. * 71yo woman who has never been tested, PPD is 16mm. CXR is negative. No symptoms. What is the next step in management. Answer is 9 months of INH. What about a 3,791yo Egyptian mummy, found to be PPD positive, asymptomatic, negative CXR? What do you do? 9 months of INH. What if the mummy had BCG? 9 months of INH. * Interferon gamma testing is not affected by BCG at all. * Who gets the 15mm cutoff for PPD testing? People who should have never been tested in the first place. Not HIV, not homeless, not healthcare workers, not IV drug users, not any form of disease, just normal people. People who living in Wyoming (100% sensitivity to TB medications, few cases, always sensitive). * If PPD is positive, you do a CXR. If CXR is negative, you get INH for 9 months. If positive, you check the sputum to make sure youre not dealing with active TB. If sputum is negative, you give INH for 9 months. If sputum shows TB, you give the 4 drugs, INH, RIF, PZA, ethambutol (EMB). * If the patient is asymptomatic and you get a positive PPD, you just caught early reactivation of TB. --------------------------------------------------------------------------------------------------Infectious Diarrhea * 27yo female medical student has a lunch today at 3pm, Chinese rice. At 6pm, she has a big, bloody, smelly, watery, poopy, diarrhea, with a temp of 102 and abdominal pain. What is the cause of her diarrhea? Not staph aureus, not bacillus cereus, not E. coli, not salmonella, not shigella, not viral, not protozoal (crypto, giardia). Most likely cause is campylobacter. * When you compare risk factors with individual presentation, the most important is individual presentation. This patient has blood in their stool. It could not be viral, protozoa, staph, B. cereus. It could be campylobacter, salmonella, shigella, E. coli, Yersinia. Risk factors do not matter if the patient has blood per rectum and a fever. C. difficile can give you blood, but patient has no antibiotics in the history, most of the time it does not cause blood. * Also, the last thing you ate is not necessarily the thing that gave you diarrhea. This patient got her diarrhea from what she had from dinner the night before or what she had for lunch the day before. Yes, staph and B. cereus cause symptoms in 2-6 hours, but they predominately present with upper GI symptoms, like vomiting. Viral is often seen in children. * If you are given no history and bloody diarrhea, campylobacter is statistically the most likely. E. coli will be associated with thrombocytopenia, renal failure, HUS. Yes the travelers and undercooked red meat. HUS is a rare and uncommon association, but you need to know it. For salmonella, association is poultry. Shellfish association is Vibrio parahaemolyticus. Campylobacter is the most common cause of invasive diarrhea in the U.S. * What if you do not get blood in the history? Check leukocytes, WBCs. The only way to distinguish the cause of bloody diarrhea is by culture.

* C. difficile associated with antibiotics. Test is via toxin. Why not culture? It is difficult to culture. That is why it is named difficile (Spanish for difficult). Treatment is metronidazole first, not vancomycin. * Protozoans, you look for ova and parasites in the stool. HIV associated diarrhea is cryptosporidium and there is no therapy and there isnt going to be a therapy. * Giardia associated with camping and homosexual men. Treatment is metronidazole. * Which form of food poisoning (diarrhea) causes symptoms the earliest after ingestion? Answer is scombroid. This also is associated with wheezing and a rash. It is from spoiled/rotten fish, tuna, mahi-mahi, sardines. The fish eats organisms that make histamine in the flesh. Treatment is anti-histamines. * Summary: individual presentation of the patient matters the most. Yes, for basic science we learned staph aureus and bacillus cereus is 2-6 hours, clostridium perfringens is 12 hours, salmonella/shigella and campylobacter is 24-36 hours. Yea, but that is after youve already diagnosed it and you did an outbreak investigation, not in an individual patient. Legionella and air conditioners, yea after youve diagnosed it and we culture all the air-conditioning systems, but not when the patient is walking in. Your decision point with diarrhea and food-poisoning is blood or no blood, then white cells or no white cells. * When you have to treat the non-blood diarrhea causes, use ciprofloxacin. You treat if they have very severe disease, like hypotension, febrile, septic. Isnt erythromycin better for campylobacter? Yes, but you dont know if its campylobacter, because you have to wait for the culture results. --------------------------------------------------------------------------------------------------Hepatitis * Hepatitis A, B, C, D, E, G. There was a hepatitis F for a little while, but they found it was C. Hepatitis G is a virus with an identity crisis. It doesnt make you turn yellow, get transaminitis, lose weight, make dark urine, or make light stool. Hepatitis G does not cause clinical disease. A patient is found to test positive for hepatitis G after a blood donation, what should you tell the patient to expect? Answer is nothing. * Hepatitis A transferred via food and water (fecal-oral). Can you get hepatitis A sexually? Well, it depends on what you eat. Hepatitis B, C, and D through blood, sex, perinatal. Hepatitis E (enteric) through food and water. HepE in the East (SE Asia, Cambodia, Vietnam, Laos), through food and water. HepA is most common in the U.S. * With acute hepatitis, you get increased ALT, increased bilirubin, malaise, fatigue, tiredness, dark urine (bilirubin in urine), light stool (bilirubin not in stool anymore), hepatomegaly, splenomegaly, weight loss (turns off appetite). * Patient presents and is yellow. How do you distinguish via history and physical exam which type of hepatitis we have? You cant. What are you going to ask, how long was your asymptomatic incubation period, have you had sex with any yellow people recently? There are 5 types of hepatitis with 2 transmission methods and 1 presentation.

* Diagnosis is via anti-hepatitis A, C, D, E serology, IgM acutely, IgG chronically. Hepatitis B is the only with surface antibody, surface antigen, core antibody, core antigen, and e antigen. * If you are positive for surface antigen (sAg), you are infectious. * Acute or chronic is determined by the surface antigen being present beyond six months. That means you cannot tell just from the serology. You have to have the time-course with it. * Core antigen could be positive or negative, it doesnt determine if HepB is chronic. * Vaccination against HepB gives you surface antibody only, so serology with just positive sAb is vaccinated. * Treatment for acute hepatitis is nothing. Chronic HepA is treated with nothing, there is no chronic HepA. * Chronic HepB is treated with interferon or (or) lamivudine. Lamivudine, an HIV retroviral, gets rid of HepB sAg. Most common cause of cancer death in the world is hepatocellular carcinoma. Most common in U.S. is lung cancer. * Chronic HepC is treated with interferon and (and) ribavirin. Interferon alone is 515% cure rate, combined with ribavirin is 40-50% cure rate. HepC is the most common reason for needing a liver transplant in the U.S. For every organ that becomes available (heart, lung, liver, pancreas) there are 4-5 people waiting. * It helps memory to think about what this actually means. Lamivudine is an oral pill to prevent the most common cause of cancer death in the world. Ribavirin/interferon can help prevent the most common cause of liver transplantation in the United States. * Chronic HepE does not exist. Chronic HepD is treated the same as HepB because you have to have HepB to get D. --------------------------------------------------------------------------------------------------Sexually Transmitted Diseases * Patient presents saying their penis is burning, has urinary frequency. How do we tell if it is urethritis or cystitis? Ask if there is discharge. If discharge, this differentiates urethritis. Next step is swab, culture, and treat. Stain, try to look for Gram-negative diplococci, try to find intracellular Gonorrhea. Same for cervicitis, swab, culture, treat. * Treatment is ceftriaxone (for gonorrhea) and azithromycin (for chlamydia). Treatment could be ciprofloxacin (for gonorrhea) and doxycycline (for chlamydia), or any combination, just so you cover both gonorrhea and chlamydia. * 27yo man comes to clinic with genital ulcer and adenopathy. Decision point is painful versus non-painful. Presentation seen in syphilis chancre, chancroid, herpes (HSV), and lymphogranuloma venereum (LGV). -oid means similar, something like cancer would be carcinoid, something like leukemia would be leukemoid. * Syphilis chancre is painless and firm/indurated, chancroid is painful and soft. * 27yo man has adenopathy, painless and firm ulcer. What is the next step? Is it RPR, VDRL? Only about 75% sensitivity with VDRL and RPR in primary syphilis because it needs time to make an antibody response. You wont find a dark field scope anywhere. However, initial test is a dark field, because it has greater sensitivity for

primary syphilis. Once you get to secondary syphilis, VDRL and RPR are at 100% sensitivity. In tertiary syphilis, you lose some sensitivity because you start to lose antibody over time. The FTA-ABS does not go away, so you do the RPR/VDRL then FTA. * You do not need to know dose, but you do need to know route of administration. * Primary and secondary syphilis, treatment is IM penicillin. Tertiary syphilis, treatment is IV penicillin. * What is patient is allergic to penicillin? Answer for primary/secondary syphilis is doxycycline. In tertiary syphilis, doxycycline will not cross the BBB very well, so treatment is desensitize and treat with penicillin. * Desensitize and treat with penicillin in pregnancy as well. * In real inflammation, like meningitis, anything will pass through the BBB, even an aminoglycoside. * Neurosyphilis takes about 10 years to develop, but about 1 year in HIV patients. * Syphilis is rarely seen in our population. * Chancroid is painful, H. ducreyi, pleomorphic Gram-negative bacillus. Testing can be done with Gram stain and culture. Lots of treatments available. Answer for treatment is single dose azithromycin. * Mycobacterium avium prophylaxis with a single weekly dose of azithromycin. * Azithromycin is fantastic, covers chancroid, chlamydia, cervicitis, urethritis, MAC, sinusitis, otitis, bronchitis, staph, strep, hemophilus. * 27yo man comes to clinic with multiple small painful genital fluid-filled vesicles. What is the next best step in the management of this patient? Answer is treat herpes simplex with acyclovir, famciclovir, or valacyclovir. You see the genital vesicles, so you do not need to Tzanck prep. If the vesicles become unroofed, meaning they get rubbed and the top comes off showing an ulcer, then you do a Tzanck and culture with Gram-stain for H. ducreyi and with serology for LGV. Tzanck smear involves swab and putting on slide, looking for cellular atypia. * LGV is big beefed-up matted up lymph nodes, ulcers around them. Do serology. Treat with doxycycline. * Patient has genital warts (condyloma acuminata). Poxvirus gives small little warts. Molluscum contagiosum gives warts with an umbilicated center. Diagnostic test for warts not smear, stain, biopsy, swab. Test is visual identification of wart. Treatment is remove, freeze (cryo), burn, chop, melt, cut, whatever, just remove it. None will cure the virus anyway. Imiquimod is a local immunostimulant and the only treatment that does not go too far and burn the skin (e.g. like cryo would). It stimulates the T-cells to react to the wart as foreign and sloughs it off. --------------------------------------------------------------------------------------------------Medication Side-Effects * Interferon major adverse effect is flu-like symptoms. Why do board exams emphasize adverse effects? Say were in 1970, treatment of CHF would be digoxin and diuretic. In 1985 for a short time we used hydralazine and nitrates, dig, diuretics. We found hydralazine wasnt as good as ACE inhibitors. So care was ACE-I, diuretics, dig. Now we know dig is lousy and does not decrease mortality. So now treatment is

ACE-I, beta blockers, diuretics. What is the most common side-effect of digoxin, GI symptoms (nausea, vomiting). Cough with ACE-I. No matter what the standard of care is, the adverse effects will not change. So if youre writing board questions and you choose side effects to ask about, you wont have to edit questions much in the future. * Interferon causes aches, pains, irritability, depression, because you make interferon yourself. That is interferon in your body. You get the same symptoms with the flu. --------------------------------------------------------------------------------------------------Cystitis & Pyelonephritis * 39yo women with urinary burning, frequency, urgency, dysuria. Is this cystitis or pyelonephritis? It is the same for both. Best initial diagnostic test for both is urinalysis. Most important thing to look for is WBCs. Bacteria in the urine is only significant in one patient population, pregnant patients. 1/3 of women with bacteria will develop pyelonephritis; preterm delivery associated with bacterial vaginosis. * Best diagnostic test for cystitis and pyelonephritis is a culture. You can tell these apart by the pain. Fever can help a little too because there is a little fever in cystitis and big fever in pyelonephritis. Suprapubic pain in cystitis and flank pain in pyelonephritis. * Cystitis treated with TMP-SMX or ciprofloxacin for 3 days. Usually length of treatment is a matter of local tribal customs. Cystitis is such a routine problem that there is a standard length of therapy, 3 days for uncomplicated. 7 days is for stones, strictures, tumors, pregnant, or other anatomic complication. * Pyelonephritis is a drug that covers Gram-negatives. That could be aminoglycosides, ampicillin, fluoroquinolones, aztreonam, piperacillin, ticarcillin, azlocillin. There is no specific drug, just pick what covers Gram-negatives. * An ultrasound looking for pyelonephritis will show nothing. You use the ultrasound because you know the patient has pyelonephritis and want to see what the cause is. Is there an anatomic defect (stone, tumor, hydronephrosis)? * Say you put patient on antibiotics and 3 days later they are still sick. Now what is the next step? Ultrasound or CT or MRI and you find an abscess around the kidney. The most accurate test for abscess biopsy or aspirate. You cant stain a CT, you cant get sensitivity on an MRI, you cant Gram stain an ultrasound. * Do you have to drain the abscess or can you treat with antibiotics? Most of the time you drain abscesses. The problem is this happened while you were on antibiotics. So, treatment is to drain the abscess. --------------------------------------------------------------------------------------------------Skin Infections * Scabies, 5 times smaller than lice, sarcoptes scabeii. Hes digging and pooping. Diagnostic test is scraping. Might see trails dug under skin in skin creases. There is itching because of the eggs and feces, contact dermatitis. Treatment is lindane or permethrin. Lice (pediculosis, crabs if genital) is found near hair areas. Treatment is lindane or permethrin cream. * What is the most superficial of all the bacterial skin infections involving just the epidermis? Weeping, oozing, honey-colored crusting of skin because it lifts up the

epidermis. Answer is impetigo, strep and sometime staph. Staph here is staph aureus (not staph epidermidis). Staph epidermidis lives on your skin and therefore does not cause infections of the skin. * Which skin infection involves the epidermis and extends into the dermis? Causes swelling in dermal lymphatic channels and bright-red skin. Answer is erysipelas, caused by strep. The strep in these skin infections is group A beta-hemolytic strep, strep pyogenes. You cant get rheumatic fever from this, but you can get glomerulonephritis. * Which skin infection involves the dermis and subcutaneous tissues? Answer is cellulitis, staph and strep. It does not get to the dermal-epidermal junction so it cannot cause oozing above skin. * If youre not sure what type of skin infection this is, what do you use? Answer is oxacillin, cloxacillin, dicloxacillin, nafcillin. If you have a mild penicillin allergy then use first-generation cephalosporin. If life threatening penicillin allergy, vancomycin or macrolides. * Impetigo can sometimes be treated with topicals like bacitracin or mupirocin. It is the only one you can treat with topicals because it is superficial. --------------------------------------------------------------------------------------------------Superficial Fungal Infections * Any form of superficial fungal infections has the best initial test of KOH and culture. Scrape area, put some KOH on it, some acetic acid, heat it up and then look at the slide. KOH melts away the epithelial cells, leaving fungi. Fungi has chitin in the cell wall of the fungus, it is the same stuff that makes the lobster/crab shell hard. * The KOH will show the fungi. The only way to know which specific fungus is to do a culture. Molds (e.g. athletes foot) take weeks for the cultures. Oral thrush, vaginal candidiasis, cryptococcus are yeasts. * Fungus is the group, inside the group are molds and yeasts. Inside you at body temperature is yeast. Environment growing in the refrigerator or plants tends to be molds. Mold in the cold, yeast in the beast. Only mold to worry about in general is skin infections, like athletes food. Others would be risopus and mucor mycosis in diabetics. * Molds include epidermophyton, dermatophyton, malassezia furfur, microsporon. Treatment is based on whether or not there is hair/nail involvement or no hair/nail involvement. * Mold treatment without hair/nail involvement would be any topical, clotrimazole, lotrimin, miconazole, ketoconazole, nystatin, terconazole, miconazole, econazole. * Mold treatment with hair/nail involvement cannot be topicals. Must be systemic therapy, terbinafine. Kid comes with toenail infection, athletes foot spreading, can also give itraconazole. Treatment for 12-weeks, fingers for 6- weeks, 80-90% cure. * Griseofulvin had to be used for 12-18 months for nail infections, only worked 3040% of the time. * We dont use ketoconazole usually; most common side -effect is gynecomastia. Ketoconazole also affects the liver due to P450 interaction. It is very anti-androgenic

that systemically you are more likely to use it for metastatic prostate cancer; it is very close to orchiectomy at high doses. * Fluconazole is a yeast drug, good for cryptococcus, vaginal candidiasis, oral thrush, yeast/candida in blood. It does not work on molds. Fluconazole is very safe. --------------------------------------------------------------------------------------------------Osteomyelitis * 57yo male diabetic with peripheral vascular disease and has pain in leg for last 10 days. It hurts just below his knee, has an ulcer over the area over past 5 days with a draining sinus tract. Rubor, dolor, tumor, calor. What is the best initial diagnostic test? CT, bone scan, x-ray, MRI, culture? Answer is x-ray, even if the person has only had the disease for 5 days. Doesnt it take 2-3 weeks for the bone to dematerialize enough to see it on an x-ray? Yes, but do the x-ray anyway. You dont skip an ECG and jump straight to the stress test. If the x-ray shows osteomyelitis, periosteal elevation, destroyed bone involucrum and sequestrum. * X-ray shows osteomyelitis. What is the next step? Answer is biopsy, then treatment. So get the culture then initiate treatment. How do you know what organism to treat? Most common cause of osteomyelitis is staph aureus. However, diabetics are more susceptible to Gram-negatives and anaerobes. So you do not know what organism to treat, which is why you do the biopsy. * When do you get a SED rate? You get it to follow the response to therapy. You start the patient on 6-weeks of antibiotics after the biopsy, then after 6-7 weeks or so you check a SED rate. You follow the ESR. If the SED rate is down then you can stop, if the SED rate is still up then you continue. * What if the original x-ray were negative, then what is the next step? Answer is MRI. Do not do a bone scan because the red ulceration soft tissue swelling gives too many false positives. MRI has same sensitivity as bone scan but does not come with the false positives. Cellulitis with red draining tissue will mess up the bone scan. If MRI or bone scan is positive, next step is biopsy to determine organism. * If x-ray and MRI negative, then this is just an ulcer, not osteomyelitis. * When do you culture the drainage out of the sinus tract? Answer is never. It will be contaminated and colonized with skin flora. It will not tell you what is in the bone. * Do not culture sinus draining tracts due to false positives. The most accurate test for any infection is culture (few exceptions, like herpes encephalitis with PCR). X-ray is always first with osteomyelitis even if you think it will be negative. Never wait for results of culture to treat. SED rate is useful to determine length of therapy. Some people may need 4-6 months of therapy; this way you do not need to do a bone biopsy every few months, use ESR. --------------------------------------------------------------------------------------------------Septic Arthritis * 72yo female Catholic nun from Brazil with a fever. She has a painful, swollen, red, hot, tender, immobile, effusive knee. What is the best initial diagnostic test? Answer is aspiration. X-ray very rarely shows anything in septic arthritis. X-rays, even in rheumatoid arthritis, have a 6-month lag time. Articulatory cartilage is amazing stuff, takes huge amount of stress and trauma, bacteria cannot penetrate, and considering

it is 97% water. So again, do not order scans here, do an aspiration; tap the knee, arthrocentesis. * Say arthrocentesis shows 62,000 white cells, polys. What is the most accurate diagnostic test for this patient? Answer is culture. When do you wait for results of culture? Never. Take culture and start to treat. What type of septic arthritis does this patient have? You have to order tests, do a Gram stain. * Non-gonococcal septic arthritis, most often staph (40%) and could be strep (30%), 20% gram negative rods, 10% misc. What is the sensitivity of a Gram stain? At least 50-70%. Synovial lining does not have a basement membrane, so bacteria (and antibiotics) pass easily. Culture of synovial fluid is over 90% sensitive. Blood cultures positive in 25-30%. So for non-gonococcal septic arthritis, were going to find it. * Treatment is with two agents to cover Gram-positive and Gram-negatives. Grampositives with something like oxacillin, nafcillin. Gram-negatives with something like third-generation cephalosporin (ceftriaxone), quinolones. There is not specific drug to learn (like ceftriaxone in meningitis), just cover bacteria present. * What about gonococcal septic arthritis? Blood positive in less than 10%, Gram positive in less than 25%, culture positive in only about 50%. Treatment is ceftriaxone. How do we make the diagnosis then? Rectum positive in 10- 20% of patents, urethra positive in 10-20% of patients, pharynx positive in 10-20%, cervix positive in 20-30% of patients. So, if you think there is gonorrhea, culture everything, all these areas. What is going to tell you to do all these cultures? Is there a rash (nonblanching petechial rash) that goes along with neisseria infections? Migratory polyarteritis, polyarthralgia (in multiple locations)? Tenosynovitis? That is how you know. * Many physicians will not even take a sexual history because they assume it is unreliable. Are you sexually active? No, I just lay there. --------------------------------------------------------------------------------------------------Endocarditis * 27yo man, IVU (IV drug use), has fever, and murmur. You wont get asked diagnosis. You look in this guys eyes and see no Roth spots. You look in hands and feet and see no Osler nodes. You look in this guys fingers and see no splinter hemorrhages. No Janeway lesions. Urine has no red cells. What does that mean? Nothing. Less than 10% of patients have any of these signs. All you need to see is fever and a new murmur. * Dr. Osler invented residencies in this country and wrote the first comprehensive medical text, 700 pages, only 40% of disease had treatments. Only single disease with 100% mortality was endocarditis. In 19th century, this is how we think about metastatic disease. These patients dies of post-infectious endocarditis. Now the most common cause of death from endocarditis is CHF. Osler lived 50 years before antibiotics were invented. * What is the next best step in the management of this patient? Blood cultures or echocardiogram? Answer is blood cultures. If blood cultures are positive, then get transthoracic echo. If TTE is negative, then get transesophageal echo (TEE). If TTE is positive, then you know this is endocarditis.

* Sensitivity of TTE is 50-60% sensitive. That is why you order blood cultures first. TTE will miss almost half of endocarditis. TEE is more than 90% sensitive, but that is pretty invasive. * Treatment is started right away, even after blood cultures. When do you answer surgery? What is the most important and urgent indication for surgery in endocarditis? Answer is CHF. No matter how long you give antibiotics, the papillary muscle and chordae tendineae will not jump out and reattach themselves. * To get endocarditis, you need a bacteremia causing procedure (e.g. IV drug use, hemicolectomy, biliary surgery, dental cleaning) and a significant defect (e.g. aortic stenosis, mitral stenosis, VSD). * Prophylaxis for general procedures like dental work is amoxicillin 2grams one hour prior. No dose after the procedure. If patient is penicillin allergic, give clindamycin, azithromycin is alright. Erythromycin will cover the mouth flora, but has side-effects (vomiting, increased motilin), so not used primarily. * Summary: you see fever and murmur, get the blood culture. If negative, get echo and youre done. * Youre going to get your teeth cleaned and have MVP, or pacer in place, CABG, aortic stenosis, mitral stenosis. So you have some heart problem and are getting a bacterial flinging surgery, cystoscopy, hemicolectomy, whatever. Now what? MVP, give prophylaxis if significant regurgitation (murmur). Pacemaker, no prophylaxis needed, even though it is a foreign body. CABG, no prophylaxis needed, coronary arteries are on the outside of the heart. Prophylaxis for endocarditis when something is on the inside. C-section (with valve problem), no prophylaxis, uterus is sterile environment. Cardiac catheterization (with valve problem), no prophylaxis, does not cause bacteremia. IHSS (HOCM) idiopathic hypertrophic subaortic stenosis, needs prophylaxis because it is significant enough to cause sudden death. --------------------------------------------------------------------------------------------------Lyme Disease * A bunch of children are brought to the hospital with rash, fever, and joint pain. The physicians do tests, ANA negative, rheumatoid factor negative, SED rate a little elevated (and SED rate is like Rorschach inkblot test, it doesnt mean anything). Physicians send kids home and say children have juvenile rheumatoid arthritis. Moms look into their pediatrics textbook of medicine and decide to stop back to the doctors office the next day. All the children in an area wont get JRA at the same time. This is how Lyme disease was described, and that is the problem with diagnosis today. It has a high incidence with the wealthy population. Lyme in Connecticut (near Yale, 3rd highest income per capita in world). Second highest incidence of Lyme in Westchester NY which has the 2nd highest per capita income in the world. * There is no malaria vaccine. There is no chlamydia trachomatis vaccine, 3-30 million people go blind annually because they dont have a dollars worth of doxycycline or erythromycin. Lyme, 15000 cases per year and it is very hard to die from. You can get joint problems, cardiac problems, possibly a few AV block cases, and some neurologic problems (e.g. Bells palsy). Mortality on Lyme is negligible, but

can get nasty arthralgia later on. But, there is a Lyme vaccine. 15,000 cases per year and practically no one dies. * The fascination from Lyme comes from parents coming back and back and back until the disease was described. You can get just the rash, or just the neuro, or just any of them. Characteristic about CN VII palsy is that it is bilateral Bell palsy. * Tick ixodes scapularis has to be attached for 24-72 hours. * Diagnosis is clinical manifestation. Serology for IgM and IgG is lousy because it cannot distinguish well between old and current infection. If you have a positive serologic test and no symptoms, you do not have Lyme. Positive blood test with no symptoms means no Lyme; this is not like syphilis with VDRL and RPR (diagnosed with syphilis even if no symptoms). Without clinical manifestations, serology is useless for Lyme. * Treatment is doxycycline or amoxicillin, for minor stuff like rash or Bell palsy or joint problem. If cardiac or neurologic problems, use ceftriaxone. * 26yo woman who gets bitten by a tick on her buttocks. She has no symptoms. What do you do for this patient? Answer is tickectomy, remove the tick. Then what? Do nothing. No need to treat. No need for tick analysis. No need for serology (positive serology wouldnt matter). You do not treat asymptomatic serology. --------------------------------------------------------------------------------------------------Human Immunodeficiency Virus (HIV) * Asymptomatic patient with HIV presents to your office asking what medications they should be taking. Is prophylaxis based on the T-cell count or the viral load? Answer is T-cell count only. Prophylaxis is based on what you are at risk of today; Tcells tell the present. Viral load tells the future, how fast will T-cells drop, how aggressive the diseases is, and when to start anti-retrovirals, HIV drugs, protease inhibitors, reverse transcriptase inhibitors. Its like a train heading toward a cliff. The T-cells tell you how far the train is away from the cliff and the viral load tells you how fast the train is moving toward the cliff. * Less than 200 T-cells and less than 50 T-cells is what you need to know. Normal is around 600-1000 T-cells. You dont feel 400 or even 350. Worry when you get near 200. * 200 T-cell or less, at risk for pneumocystis pneumonia (PCP), prophylaxis with TMP-SMX. Patient may get a rash sometimes. If you get a rash, use dapsone. Third line is atovaquone. Aerosol pentamidine is 4th line and practically an extinct drug. * 50 T-cell or less, at risk for mycobacterium avium intracellular (MAI, MAC), prophylaxis with azithromycin. Rifabutin is a dead drug. * No mortality benefit with antifungal prophylaxis with fluconazole. CMV with oral ganciclovir also not used. * Asymptomatic HIV-positive man comes to your office with 12 lonely T-cells, what should he be started on? Answer is TMP-SMX and azithromycin. * What vaccinations should an HIV positive person be on? Answer is influenza annually and pneumococcal. * When to start HIV medications, based on T-cell count of viral load? CD4 under 350 or viral load over 55000.

* Zidovudine (AZT) causes anemia. Didanosine (DDI), stavudine (D4T), zalcitabine (DDC). Side-effects of DDI and D4T are neuropathy and pancreatitis. Lamivudine (3TC) has such few side-effects that the placebo has more side-effects during testing. Reverse transcriptase inhibitors are -vudine drugs. With these 5 medications, patients were taking pills all the time but still dying anyway. * Protease inhibitors were created de-novo, not by screening thousands of compounds. These are very effective. They kill a virus so we call them -avir. Indinavir, saquinavir, nelfinavir, ritonavir. * Use 2 nucleosides and a protease inhibitor (PI) and viral load goes to undetectable in 80-90% of patients. This means you cannot die from the virus. * About 5% of patients who are HIV positive will never develop AIDS. What determines what length of time you develop AIDS, meaning T-cells from > 500 down to < 200? Answer is viral load. The rate of decay of T-cells is dependent on the viral load. 95-98% of patients who are HIV positive without treatment will die from AIDS. * So protease inhibitors help drop viral load to undetectable in 80-90%. So how much has the death rate from HIV dropped in the United States? Answer is 80-90%. * Protease inhibitors have side-effects of hyperglycemia and hyperlipidemia. Youd rather be alive with a high LDL and HbA1c then dead with normal levels. * Put patient on 2+1 therapy, theyre on it for 3-months, comes back with terrible flank pain and hematuria. Which drug caused this? Answer is indinavir, causes stones. * Once you start HIV medications and you go undetectable, how long do you continue the medications? If you keep patient on medications for 5 years, they are undetectable, stop medications, the virus can come back the next day. So, patient is on medications forever. ------------------------------------------------------------------------------------------------Post-Exposure Prophylaxis * 26yo surgical intern is stuck with an HIV positive needle (needlestick) in the O.R. by her attending physician. What is the next best step in management? Answer is give 2 nucleosides and 1 protease inhibitor for one month. * 33yo man has unprotected sex with an HIV positive girl, who tells him afterwards. What is the next best step in the morning? Answer is give 2+1 for one month. * 27yo woman is sexually assaulted by an unknown man with unknown HIV status? What medications do you give her? Answer is 2+1 for one month. --------------------------------------------------------------------------------------------------HIV Transmission Risk * 32yo woman who is pregnant and is HIV positive. She is at 7 weeks of pregnancy. Her T-cell count is 35, viral load of 600,000. What do you do for this patient? Answer is 2+1 now. You start HIV medication for < 350 T-cell or viral load > 55,000. Do not let her die because she is pregnant. This woman has a 50% six month mortality. * Which HIV medications (nucleosides or PI) are teratogenic? None of them. * Pediatric HIV is virtually non-existent in the U.S., less than 400 children born HIV positive. Average life expectance of an HIV positive person who gets the disease

around the age of 40 is the same as an HIV negative person provided they get the 2+1 therapy. * AZT alone in pregnancy results in about an 8% transmission of HIV. With 2+1 therapy, transmission rate drops even further to about 2%. * In the United States, average life expectancy for a woman is 79 years. In South Africa, it is 32 years, mainly due to the fact that 1/3 of women in prenatal clinics are infected with HIV. Half of those children will be infected and most of those children will be dead by age 5. Have gratitude in your attitude for access to the medical therapy we have. * Risk of transmission is about 1:3000 for female to male (vaginal). Risk is about 1:1000 for male to female (vaginal). Needlestick exposure is about 1:300 episodes/contact. Anal receptive is about 1:100 (1%) per shot, load, event. Mother to child transmission is about 1:20 with meds and about 1:3 or 1:4 without medications. C-section not routinely indicated, only if viral load is not under control. Risk of transmission through breast milk is equal to unprotected sex or equal to the risk from the delivery itself. In Africa, death rate is higher since there is no milk formula available. Oral transmission rates are not really known, it is a difficult thing to study. * Why is male to female transmission higher than female to male? Answer is not vaginal trauma. Answer is not surface area differences. Answer is duration of contact with infected semen for 4-5 hours. Man is only exposed for about, what, 10 seconds? The time duration of the coitus, less than 4-5 hours. * Over 70 years, there are about 5 drugs created for tuberculosis. Over 10 years, 16 HIV medications were approved and new ones coming out all the time. You took a fatal prognosis with HIV, and made it 80-90% not fatal. You took a transmission risk of 50% and brought it down to 2-3%, and that is a great good. --------------------------------------------------------------------------------------------------Kaplan Videos (2001) Dermatology with Dr. Marsha Gordon, MD --------------------------------------------------------------------------------------------------This section is very picture-intensive, so quickly review an image for each disease discussed. --------------------------------------------------------------------------------------------------Bullous (Blistering) Diseases * Pemphigus vegetans is a rare variant of pemphigus vulgaris. Pemphigus vulgaris is an autoimmune disease of the skin. Patients with pemphigus vulgaris have autoantibodies that respond to antigens that are found in the epidermis (top layer) of the skin. These antibodies are found in the intercellular spaces between epidermal cells. * When the antibodies attach to the antigens, the skin can no longer stay intact, and it breaks apart and blisters up. * Pemphigus vulgaris is a blistering disease, but most of the skin will be denuded. The blisters are so superficial that by the time the patient presents to the physician you rarely see intact blisters. If you do see intact blisters, they are very fragile and easily breakable. * This disease affects the skin and the oral mucosa. Erosions in the oral mucosa may occur before skin is involved. The disease was fatal prior to discovering

corticosteroids, now we treat with high dose prednisone and steroid sparing medications like azathioprine. * Many disease present looking similar to pemphigus vulgaris. If you see a slide with blisters and denuded areas, think pemphigus but realize the actual diagnosis is made by biopsy. * Biopsy must show the epidermis breaking apart one cell from the other, and also immunofluorescence can show the antibodies being deposited in little circles around the epidermal cells. * Pemphigus foliaceus and pemphigus erythematosus are more superficial forms of pemphigus. They almost never present with intact blisters. * Bullous pemphigoid is an autoimmune disease of the skin with antibodies reacting against antigens at the dermoepidermal junction zone (DEJZ). They do not react with anything in the epidermis per say, they react at the DEJZ. As a result, the blisters are deeper. * There will be intact, tense, blisters with some denuded/eroded areas. The blisters are tougher, tend to be tense, and often are intact when they present. * Bullous pemphigoid presents in an older population. Pemphigus presents classically around ages 30-40. Bullous pemphigoid is seen in a geriatric population classically, ages around 70-80. This does not generally involve the oral mucosa, a differentiating feature between the two. * Bullous pemphigoid biopsy will show blister with entire epidermis lifting and separating from the dermis. Immunofluorescent will show lighting-up, a linear line, at the DE junction. * Pemphigus and pemphigoid tend to heal well with treatment, steroids. * Cicatricial pemphigoid is a less important variant and does scar even with treatment. It almost exclusively involves the mouth and eyes; so scarred tongue, scars around mouth and blindness from eye involvement. --------------------------------------------------------------------------------------------------Porphyria Cutanea Tarda * All porphyrias relate to abnormalities of heme metabolism. * Porphyria cutanea tarda (PCT) is the classic dermatological porphyria. The metabolites that accumulate are photosensitizing. Skin that is exposed to the sun, classically the dorsum of the hands, develops a photosensitivity. * Clinically we see blisters and erosions. This condition does not heal well; it heals with scarring and heals slowly. * The face also can be involved. The top of the scalp is another location. * Two other skin manifestations besides blisters. First, patients develop hyperpigmentation; they look like they are getting a tan. Second, they develop hypertrichosis, they become hairier. The classic place the hair is seen is at the tops of the cheeks. * Precipitating factors for PCT is alcohol, so middle-aged alcoholic with blisters. Estrogens are a classic precipitant, so women who are on birth control pills or hormone replacement. Iron may be a precipitant, and fungicide hexachlorobenzene may be a precipitant.

* Treatment is to remove precipitating factor and use anti-malarial medication chloroquine. We also treat with blood-letting (phlebotomy) to try to drain off excess iron. It is a slow improvement and may take up to a year. --------------------------------------------------------------------------------------------------Hypersensitivity Reactions * Urticaria is the classic type I hypersensitivity eruption. It is mast-cell induced. Mast-cells have IgE antibodies affixed to them, and when proper allergen comes along, it attaches to IgE antibody on the mast cell and causes release of the mast cell package, including lots of histamine and mediators. * You see vasodilatation, causing hive pinkness. You see leakiness of the vessels, causing hive swelling. * These are immediate reactions, meaning if the patient is allergic to penicillin and you give them penicillin, you will get an urticarial reaction probably within 30 minutes to an hour at the most. * Any individual hive will last up to 24 hours. However, new hives will continue to present over weeks as long as the antigens are present in the system. This is important because other diseases can look like hives. If the urticarial hive lasts less than 24 hours then it is urticaria. * Morbilliform eruption is a faint pink morbilliform (measles-like) eruption. It may be slightly raised. This is believed to be a type IV hypersensitivity eruption, lymphocyte mediated. * If a patient is given an antibiotic and are doomed to develop a morbilliform rash, the first time they are exposed they will not develop a rash from somewhere between 10days to 2weeks. It is believed that this is the time needed to recruit and sensitize the lymphocytes. * The second exposure will result in a reaction within 3-4 days because they have been sensitized. * Morbilliform drug rash often seen with antibiotics. * Spectrum of hypersensitivity reactions begin with erythema multiforme, then to Stevens-Johnson, and finally to toxic epidermal necrolysis (TEN). * Erythema multiforme refers to target-like lesions, classically seen on the palms. However, they can be seen on other parts of the body. Classic example is a bull-shot with a dusky bluish color in the center and a red rim around. * The multiforme means it can take many forms. So there may be a blister in the center with a red rim around. Whatever the formation, it will be target-like, targetoid appearance. Classically involves the palms and soles. * If any mucus membrane involvement, the involvement has to be very minor to be called erythema multiforme. * Erythema multiforme often seen to drugs and also infection (herpes and mycoplasma pneumonia). Specifically with herpes, a recurrent infection, patient can get recurrent erythema multiforme with each outbreak. * Erythema multiforme is self-limiting, thus will go away if you remove the offending agent/disease.

* Stevens-Johnson refers to more than minor mucus membrane involvement. Classically you see hemorrhagic crusts of the lips and oral mucosa. These patients will likely have skin involvement as well. * Stevens-Johnson has a mortality of 5-10% even with appropriate treatment, cortisone (prednisone). * Disease can last 2-3 weeks. With mucosal involvement, patient may not be eating well. So you see fluid and electrolyte abnormalities. With skin involved, patient is prone to infection. * Toxic epidermal necrolysis involves skin that is confluently red that burns or feel tender. Within a short period of time, the skin begins to bubble-up and literally sloughs off. * There can be 100% sloughing of skin, so like a 100% burn. These patients are tremendously ill. May have fever, elevated white count and elevated liver function tests. * TEN prone to electrolyte abnormalities and infection. Mortality is 25-50% even with appropriate treatment. This is almost always caused by a drug, phenytoin, barbiturates, carbamazepine, allopurinol, penicillins and sulfonamides. Think about this if called to a neuro management unit for a major skin disorder. * TEN can look very much like staphylococcal scalded skin (SSS) syndrome, but is treated very differently. Treatment of SSS with antibiotics, while TEN can be caused by antibiotics. * To make a TEN diagnosis, we do a biopsy and send it for frozen section because we want an answer quickly. * On frozen section, we see complete necrosis of the entire epidermis. The entire thickness of the epidermis is lifted and separated from the dermis and is necrotic/dead. In SSS, the break is way high up in the epidermis. * TEN treatment is stop medications, put in laminar flow room to prevent infection, watch fluid and electrolytes, steroids are highly controversial because they mask infection, which is a major cause of death in TEN. Because eye involvement can occur, must have an ophthalmologist involved in this care. * Fixed drug eruption refers to a situation when a patient is exposed to a drug they are allergic to. In one are, they develop either a blister or a red mark. If you take away the drug, it heals. If you give the drug back, in the exact same location, the same reaction occurs. We do not know why this happens, it is just a curiosity. It is benign. * Fixed drug eruption generally heals with a dark mark that can take a long time to fade. --------------------------------------------------------------------------------------------------Bacterial Skin Infections * Tinea pedis (fungal infection of foot) caused by dermatophytes (fungus that attacks the skin). Tinea pedis classically involves the interdigital webs between the toes. There is a blistering form of tinea pedis, bullous tinea pedis, usually involving the soles of the foot. The blisters are intact and if you open and do a KOH prep you will see it teeming with dermatophytes.

* Fungal infection of the toenails is onychomycosis. Tinea cruris is fungal infection of the groin. Classic pattern is annular (round), red, scaly border, extending over time. * Tinea capitus usually seen in kids with a patch of hair loss (alopecia) or thinning. Sometimes the skin is scaly and peeling. Sometimes there will be a few little pustules, pus bumps. Tinea capitus can scar if not treated, giving a permanent scarring alopecia. * Treatment of tinea capitus is oral anti-fungals. The infection can get down into the roots so topicals will not reach. * Tinea corporis can be very subtle with a tiny pink scaly area. Many times you will think this is just eczema, but dont forget it could be a tinea corporis. If you cant tell the difference, do a culture (takes weeks) or a KOH prep. * Honey colored crust, think impetigo. Caused by staph or group A strep (pyogenes). * Certain strains of staph exude a toxin, exfoliatin. This toxin can cause the skin to blister. Say patient has impetigo that is caused by one of these types of staph (bullous impetigo), you will see the honey-colored crust and blisters around. The blisters have purulent material in them and are teeming with staph. * Treatments involve covering for staph and for strep. * Staphylococcal scalded skin syndrome is from staph that exudes exfoliatin toxin, but the main site is distant from the skin (e.g. abdominal abscess). Say in addition the patient is pretty sick, poor kidneys or poor liver, older or very young. Because the patient is debilitated, they cannot break down this toxin and it spread throughout the body. * Exfoliatin toxin causes skin to exfoliate. It may be difficult to distinguish at the bedside from TEN because you see the skin sloughing off. If you culture or Gramstain skin, you wont see anything because the staph is at a distant site and youre getting the result of the toxin. * Here you do a frozen section to differentiate from TEN. Here in SSS syndrome, you see intact epidermis expect for very superficial blistering. Superficial at the granular layer of the skin. * Treatment for SSS syndrome is anti-staph antibiotics. If TEN, then take them off antibiotics. --------------------------------------------------------------------------------------------------Viral Skin Infections * Herpes is a viral infection that classically involves the mouth or genitalia, mostly because those are the common areas of inoculation. It can be seen in other areas including the eye. * Clinically you see groups of blisters on a red base, such as cold sores. The first case of a cold sore for a patient can be extremely angry if they have no immunity to it. Lips can be very swollen. * Herpes comes and goes, often arising in times of stress. Oral herpes can be brought out by sun as well. * Say you see many groups of blisters on a red base, extending down a dermatome. This is shingles from herpes zoster, a reactivation of latent varicella zoster virus (chickenpox). Generally zoster begins with pain. Patient may present to physician

thinking theyre having a heart attack, or kidney stone, or sciatica. A day later, the blisters begin to develop. * If you see a zoster involving the ear, think about Ramsey Hunt, tinnitus, vertigo, CN VIII, facial palsy. * If trigeminal ophthalmic division is involved with herpes you need an ophthalmologist. * Herpes zoster is disseminated if more than 20 blister outside of a dermatome. This means there is hematogenous spread. This patient is not controlling their zoster infection. Admit patient to hospital and treat aggressively. * Patients need a good general workup for any herpes zoster a month after healing because they may be immunosuppressed for a variety of reasons, associated with lymphoproliferative diseases. * Warts caused by human papilloma virus, many types. Not much to say about warts. * Cellulitis usually caused by staph or beta-hemolytic strep. Presents with red, warm, tender plaque. Usually pretty evident. Mark with pen to determine if treatment is working or if cellulitis extending. Often we cannot do a culture so treat for what you think it is (staph or strep). * Necrotizing fasciitis (flesh eating disease) is a medical emergency. Can be caused by a mixed infection or a strep infection. Generally it starts with a cellulitis and quickly the organisms go deep and make it to fascial planes, where they can spread across the planes and cause damage very quickly. Strep can travel extremely fast, from ankle to thigh in a single day. * Treat necrotizing fasciitis with antibiotics and it absolutely requires surgical debridement. There is no way the antibiotic will get into the necrotic tissue at the fascia in time unless the area is completely and widely debrided. * If there is a question, we can do a biopsy and see strep very deep. But we must treat quickly. * Syphilis starts with a chancre (primary), hard, nontender classically, associated with nontender adenopathy. Average time of onset of chancre is 3-weeks after exposure, can be from 10-90 days. The average time when the blood test turns positive is 4-weeks. So if you see the patient and think it is a chancre, you need to do a dark field exam of the chancre looking for spirochetes, treponema pallidum. * Secondary syphilis appears 6-8 weeks after chancre, see oval salmon-colored patches over the body with palms and soles involved. At this point, the blood test should be positive. * HIV-positive patients can quickly develop tertiary syphilis with CNS changes. Some believe you need an LP. --------------------------------------------------------------------------------------------------Other Skin Infections * Scabies is caused by human mite, sarcoptes scabeii. It causes an extremely itchy bumpy area, such as penis and scrotum. Other places classically involved are breast, especially areola, axilla often involved, and the interdigital webs of the hands. * Furrows or linear burrows may be a clue. Only an average of 11 mites on entire body. You need to scrap a spot and look for eggs. Itchiness due to stool.

* Meningococcemia present with a brief upper respiratory infection. Next stage is a fever and mental status changes. If you see fever and rash, think meningococcemia. Petechia are non-blanching purple areas of the skin. The lesions have a smudges look and given time they will become vesicular and necrotic with a slate-gray center. * You may not see signs of meningeal irritation. If can be so acute and fulminant that there isnt enough time. * Do Gram-stain of CSF looking for organisms, increased protein and decreased glucose. * Think about any other bacteremia here also, like staph bacteremia. Not everything that is purpuric is not a major infection. Senile purpura is seen in elderly patients with fragile blood vessels and connective tissue. They can get large purpuric areas with very minor trauma. --------------------------------------------------------------------------------------------------Pigmented Lesions * Pigmented lesions come in all shapes and sizes. Keep in mind symmetry. Is this lesion symmetrical? Yea, look at ABCD, border, color, diameter smaller than pencil eraser, but symmetry matters. Benign pigmented lesions tend to be symmetric. Once a malignant pigmented lesion has occurred, the growth is out of control. * A junction lesion, symmetric, is a benign lesion with the melanocytes found at junction (epidermis/dermis). * Compound nevus, benign, symmetric. Some of the nevocytes are at DE junction and some down into dermis. * Dermal (intradermal) nevus is skin colored, symmetrical, benign. * Halo nevus is a symmetrical halo of lightness around a nevus. This is benign. Lymphocytes are destroying nevus. * Nevus with irregular border, irregular color, this is melanoma. Melanomas start with melanoma in situ, where it is only present at the epidermis. If it presents at this stage without dermis involvement there is 100% cure with proper excision. Melanomas are asymmetrical and irregular. * Nodular melanoma may be more symmetrical than others, but it is still irregular. * Most common melanoma is superficial spreading type, 70%. This has an in situ phase lasing 6months to a year. Next most common is nodular, 15% of melanomas. It appears and immediately invades. Other types are acral melanoma (hand, feet) with high mortality and lentigo malignant type seen on the sun exposed areas of elderly people and has good prognosis. * Stages are I (skin), II (lymph), and III (mets). Prognosis is based on level of depth of invasion. * Greasy stuck-on appearance is seborrheic keratoses, overgrowth of top layer of skin, benign. * Actinic keratoses refer to an area of the skin which is scaly and caused by sun (sun exposed area). They are precancerous, but the majority of patients do not develop cancer. Can turn into squamous cell carcinomas. * 75% of squamous cell carcinomas on the skin are sun-induced. They have a low metastatic rate, around 1%. If we can find them and treat quickly, we can cure the

patient in most cases. The exception is squamous cell carcinoma of the mucosa (lip) where there is a metastatic rate 10-20%. * Basal cell carcinoma has pearly raised border with crust, broken blood vessel on it possibly, most common of all cancers. Generally do not metastasize. Should be removed. * Kaposi sarcoma seen in immunosuppressed population (AIDS, lymphoproliferative, chemotherapy) and elderly (Mediterranean). Sarcoma is a misnomer, probably a neoplasm of epithelial cells likely lymphatic and small blood vessels. Bluish plaque like discoloration. Diagnosis with biopsy. --------------------------------------------------------------------------------------------------Papulosquamous (Scaly) Eruptions * Psoriasis is silvery scale on a red base. Usually involves the knees and elbows, rarely the palms, sometimes the scalp. Can cause nail changes (pits) and generally nail dystrophy (onycholysis). * Eczema (dermatitis) can be asteatotic, meaning eczema simply on the basis of dryness. Skin gets so dry that is simply breaks down. * Atopic (allergic) eczema is usually seen in atopic families. Patients will have asthma, hay fever, and eczema. They have lots of IgE in system, specifically to staph. So they react in an expected way to the staph that is on our skin, with red itchy patches in the flexures. Skin can be thickened from scratching. Ask about family atopy. * Treat by removing things that cause allergy. * Seborrheic eczema is a chronic condition with redness and scaling in classic locations, not well understood. Locations are the nasal labial folds, eyebrows, around ears, scalp. * Stasis dermatitis begins with venous insufficiency. Formed elements in blood cells like hemosiderin leave the dark spots. Protein surrounds the vessels causing fibrin cuffs and preventing oxygen exchange so tissue becomes hypoxic and breaks down. * Contact dermatitis from irritation or from allergic contact dermatitis. In allergic contact dermatitis, you see well demarcated rash only at location of problem (e.g. poison ivy in linear blisters). * Pityriasis rosea looks like secondary syphilis, oval slightly scaly salmon-colored patches over the body. It never involves the palms. It classically begins with a herald patch. Get a VDRL to rule-out syphilis also. * Decubitus ulcers are defined in stages. Stage I is non-blanching redness. Stage II is very superficial breakdown. Stage III is full-thickness skin breakdown but not to fascia. Stage IV means all the way through to bone or muscle. Decubitus ulcers caused by pressure, sheering forces (slipping down in bed), friction, rubbing, moisture from incontinence (urinary or fecal). --------------------------------------------------------------------------------------------------Nail & Hair Disorders * Acute paronychia is an infection, usually staph, in the area around the nail. Seen often in patients who get manicures. Sometimes you see a pus bump, sometimes just red and tender. Generally culture but always cover for staph. Treat quickly because if

swelling is great enough you can impair the blood supply to the distal tip of the finger. So decompress, drain, soak, treat with antibiotics. * Clubbing refers to a distal bulbous enlargement of the finger tip. It is sometimes seen in a familial setting but most often in a patient with chronic lung problems, such as emphysema, bronchiectasis, tuberculosis. * Clubbing can be seen in cyanotic heart disease as well. * Hypertrophic osteoarthropathy looks like clubbing except there is tenderness at the distal fingers. There will be periosteal thickening of the distal phalanx and often gynecomastia. Associated with lung carcinoma. * Alopecia areata is oval or round patches of complete hair loss. Alopecia areata totalis covers all the hair in the head, universalis covers all the hair of the body. It is believed to be an autoimmune type of disease but poorly understood. It can also involve the nails, with nail dystrophy. * Traction alopecia comes from chronic pulling of the hair so much so that the roots are destroyed. Seen in patients who wear tight braids or tight pulling. This is a permanent scarring alopecia. * Telogen effluvium seen after patient goes through tremendous physiologic stress, surgery, delivery of a baby, infection, sometimes many of the hairs will all go into a resting phase (telogen). The body is stressed so it is shutting off non-essential functions like hair growth. When new hairs begin to grow in a month or two, the new hair pushes out the old hairs and there can be an enormous amount of hair loss. These hairs will grow back, but there will be a few months of hair loss. The history makes the diagnosis here. * Androgenetic alopecia is male-pattern or female-pattern baldness. Often begins in 20s-30s and continues throughout life. --------------------------------------------------------------------------------------------------Benign Skin Growths * Gout is caused by chronic elevated uric acid levels. Uric acid begins to deposit in the joint, causing joint pain. Over time it will deposit in skin giving chalky subcutaneous masses called tophi. Tophi is sodium urate crystals deposited in the skin. They are benign. * Acrochordons (skin tags) are benign and not associated with internal malignancy or anything else. Could be a personal or familial tendency. Seen more frequently in obese patients possibly more frequently in diabetics. * Sebaceous cysts divided into epidermoid cyst and pilar (trichilemmal) cyst. It depends on where the cyst originates from, if from dermis it is epidermoid and if hair follicle then pilar cyst. Either way, skin epithelium or hair follicle epithelium invaginates under the skin and makes a pocket. That skin keeps making oil/sebum but is in a pocket. This is completely benign. If there is no punctum connecting it to the surface, all you see is a nodule. * 90% of cysts on scalp are pilar cysts but you really cant tell until you remove them. * Digital mucus cyst are soft, could be a collection of mucopolysaccharides (benign and we drain) or a herniation of a joint sac. Either way, no need to worry.

* Hemangiomas are benign collections of blood vessels. There are two types, capillary and cavernous. Capillary hemangiomas also called strawberry hemangiomas, generally appear early in life and enlarge during first year of life. By the end of the first year they generally stop enlarging and involute. They involute each year such that by the time the child is 9, 90% have completely disappeared. * Cavernous hemangioma appears in infancy and persists, it grows with the patients. It is due to deeper dermal vessels. Also benign. Biopsy of this is difficult and causes lots of bleeding, so dont do it. --------------------------------------------------------------------------------------------------Kaplan Videos (2001) Gastroenterology with Dr. Asher Kornbluth, MD --------------------------------------------------------------------------------------------------Gastroesophageal Reflux Disease (GERD) * Esophagus is a simple structure. Given symptoms, what is the diagnosis or how do you make the diagnosis? * Hypopharynx empties food into the upper esophagus via the upper esophageal sphincter (UES). Food travels down to the stomach at the gastroesophageal junction, where the lower esophageal sphincter (LES) exists. * Esophagus is made of smooth muscle except for the very top where there is some skeletal muscle. Disease such as muscular dystrophies that affect skeletal muscle could cause disturbances in the upper esophagus. * Disease of smooth muscle, like scleroderma, affects the esophagus greatly. * Most of the esophagus is squamous epithelium. The lower part near the GE junction is columnar epithelium. The most common types of cancers of the gastrointestinal tract are adenocarcinomas. There is squamous epithelium at the anus again, so anal cancer would be squamous cell carcinoma. * Does food travel down the esophagus by gravity? No, but gravity does play a role. That is why staying sitting up after eating is a lifestyle modification for GERD. * Food travels down the esophagus by ordered (sequential) peristalsis. So all the areas of the esophagus are at low pressure at rest then higher pressure when moving food. The UES and LES will be at high resting pressure then going to low pressure to allow food to pass. You see these pressure spikes and dips on esophageal manometry. * Primary peristalsis begins when you swallow something. When food hits the upper esophagus you get secondary peristalsis. Primary peristalsis is voluntary and secondary peristalsis is involuntary. * 32yo man comes to the ED for substernal chest pain of two hours duration. He says that he sometimes gets the pain while lying in bed at night. He is otherwise free of symptoms except a non-productive cough the he has had for the past month or so. His physical exam is unremarkable. He is given sublingual nitroglycerin and notes his chest pain worsens. What do you think is the most likely diagnosis? GERD. Remember that common things happen commonly. The pain was probably burning, subxiphoid, radiating up. * Gastroesophageal reflux disease (GERD) is most commonly caused by LES laxity, also known as inappropriate relaxation of the LES at rest. Other causes include

decreased saliva in smokers as saliva buffers acid, loss of peristalsis near the junction, decreased smooth muscle function due to scleroderma. * Symptoms of GERD outside the esophagus include laryngitis, hoarseness, pain with speaking, chronic non-productive cough, nocturnal wheezing or coughing when lying flat. * Pulmonary symptoms of GERD are due to acid making its way all the way up and past the UES, then refluxing into the lungs and causing a chemical irritation of the bronchi. So acid-induced bronchospasm. * Most common causes of chronic cough are: post-nasal drip (e.g. allergy), asthma, GERD. * Nocturnal means it awakes the patient at night; this is a key symptom if associated with any disease. * Peptic stricture is a smooth tapering of the lower esophagus due to chronic GERD. Patient has longstanding GERD and presents with dysphagia. They can swallow liquids and soft foods, but solid foods are difficult (mechanical dysphagia). Over time they will have trouble with soft foods then liquids. * Chronic acid exposure at the lower esophagus, not necessarily correlating to how bad the symptoms are, can lead to Barrett esophagus. Barrett esophagus is metaplasia from squamous to columnar mucosa at the distal esophagus. Histologically it looks like small intestine cells. * Barrett esophagus does not hurt and you cant see it on an x-ray or barium swallow. You can only diagnose it with endoscopy with biopsy. The endoscopist can suspect Barrett because the color of Barrett mucosa is more red than the surrounding mucosa, but redness is non-specific (e.g. esophagitis). * Barrett esophagus can lead to adenocarcinoma, which is why we are about this finding. Most patients with GERD do not get Barrett esophagus and most patients with Barrett esophagus do not get adenocarcinoma. However, Barrett patients should undergo surveillance endoscopy at some interval (e.g. 1-2 years depending on dysplasia grade). * High grade dysplasia management is to confirm the biopsy report with a second expert pathologist because there is a fair amount of inter-observer variation. Once confirmed, do esophageal resection because it is very likely to develop cancer. You may even find adenocarcinoma once the entire esophagus is removed. * Low grade dysplasia management is treat GERD aggressively for a few months then re-biopsy. * Your only chance at cure for any GI tract cancer is resection. A chance to cut is a chance to cure. * Most esophageal cancers are not cured because by the time the patient presents with dysphagia the cancer has already advanced past the wall of the esophagus. * 24h pH monitoring is the gold standard for diagnosing GERD. That does not mean all patients should get this. * Diagnosis of GERD is made by history, then you should start treating the patient. * Gold standard is not endoscopy, barium swallow, or x-ray because those do not show you that acid is coming back up in waves. The test that does do that is 24h pH

monitor via catheter through nose that has pressure transducers along the catheter and a pH probe at the end. Patient wears a pack like a Holter monitor for that time. * Say patient has chest pain, cardiac is ruled out. You do 24h pH monitor and it shows distal to mid esophagus pH is 2-3 during the 24 hour period. That is very suggestive that the symptoms are from GERD. * GERD treatment begins with non-invasive lifestyle modification. Avoid foods like caffeine, chocolate, alcohol or fatty foods because they open the LES. Avoid cigarettes smoking because it dilates the LES and decreases esophageal mucus. Tell patient not to lie down within an hour or so of eating. * GERD treatment after lifestyle is medication. Single best drugs are proton pump inhibitors (PPI) like omeprazole, pantoprazole, lansoprazole, esomeprazole, rabeprazole. These drugs inhibit the parietal cell from secreting acid. Theoretically the patient will stop making all their acid and become achlorhydric. So even if you LES is wide open and you reflux all day, the reflux is neutral (non-acidic) so causes no major damage. * GERD medication for mild symptoms would be antacids if sporadic like magnesium hydroxide or calcium carbonate. Antacids do not eliminate the acid, they buffer it. Side effect of magnesium hydroxide antacids is diarrhea. The magnesium is not absorbed and causes osmotic diarrhea. * Mnemonic: MG, Makes you Go. Aluminum gives you minimum stool (constipation). * GERD medication for mild symptoms with longer relief is the H2-receptor antagonists. Examples are cimetidine, ranitidine, famotidine, nizatidine. Nearly all of these are available over the counter because they are generally safe and are effective for most people. PPIs are also available over the counter more recently. * Histamine binds to parietal cells and stimulates the release of acid. H2-blocking reduces but does not eliminate acid because parietal cells are stimulated by other things. Whatever stimulates the parietal cell, the final pathway to acid secretion is via the hydrogen-potassium ATPase pump. That pump is blocked by PPIs. * Cimetidine can cause gynecomastia in men, some confusion in the elderly, and inhibits CYP450 so be careful with drugs like warfarin and theophylline. * Metoclopramide inhibits dopamine. Dopamine at the LES causes relaxation. So inhibition will result in contraction, thus metoclopramide can tighten the LES. More so, it increases gastric motility and emptying so acid is put further downstream. Why dont we use metoclopramide as primary treatment? Because dopamine receptors are also found in the brain. Lack of dopamine causes Parkinson-like symptoms, such as tardive dyskinesia. * Cisapride is no longer available. It tightens the LES by simulating cholinergic receptors. Acetylcholine keeps LES shut at rest. Cisapride was very effective, but the major side effect is torsades de pointes due to prolonged QT. The drug was pulled due to a fair number of sudden deaths, which is a bad price to pay for heartburn. Drugs like cimetidine or azithromycin could potentiate cisapride, increasing the chance of polymorphic ventricular tachycardia. * If you do endoscopy and see erosive esophagitis, go straight to the PPI medications.

* Say you tried lifestyle modifications and then went to H2-blockers and then PPIs, added metoclopramide and the patient still has symptoms. You do a 24h pH monitor and the pH is still 2 or 3. Or another example say the patient is symptomatic and no longer wants to take medications. The next step is surgery, a Nissen fundoplication. * Nissen fundoplication is usually done laparoscopically. In this surgery, the fundus is wrapped around the LES so that the LES has a sleeve tightening it. If the wrap is too tight, the patient will have dysphagia. If not tight enough, the patient still has reflux. Surgery should be considered after no relief from maximal medical therapy. Dysphagia * 32yo woman with no past medical history comes to your office for evaluation of difficulty swallowing foods. She has had the problem for over a year and has trouble with solid foods. She has occasional difficulty swallowing liquids. The symptoms have not worsened at all over this period and the physical examination is unremarkable. What is the next step in the evaluation of this patient? * Dysphagia means difficulty swallowing. Odynophagia means pain with swallowing. * If there is progressive narrowing then suspect mechanical dysphagia. Maybe there is a cancer near the esophagus or there is a stricture. Motility dysphagia has symptoms that do not depend on the size of the food. * You need esophageal manometry to diagnose the particular kind of motility problem. Say you do a manometry and see flat-line with no spikes and the LES does not relax. This is achalasia. * With achalasia, there is invasion of the neural plexus that causes contraction. So there are no peristaltic waves in the esophagus. You also see inappropriate contraction of the LES that does not relax. When you do an upper GI series (i.e. barium swallow), you see a birds beak esophagus with barium. The beak is the contracted LES. You cannot make the diagnosis of achalasia without an esophageal motility study. * With most dysphagia, think of an x-ray (e.g. barium swallow) as being the first step. * Treatment of achalasia is usually being pneumatic (balloon) dilation at the LES to tear the muscle fibers. Another option is to inject the area near the LES with botulism toxin because that poisons the acetylcholine, but this does not work very well long-term. Definitive procedure is a myotemy; the surgeon cuts the muscle fibers of the LES so it is permanently opened. Can be done laparoscopically. * Achalasia is rare and even less common is pseudo-achalasia where you see the birds beak but in reality the causes a gastric carcinoma wrapping around the LES. So also do an endoscopy with biopsy when you see achalasia. * 32yo woman with motility dysphagia. Esophageal manometry shows flat-line of no peristalsis but the LES is also flat-line (low pressure). This is classic for scleroderma. In scleroderma, the esophageal smooth muscle is replaced with collagen and LES replaced with collagen. The LES is open so they get reflux, but there is also no motility to wash the acid back down so the reflux can be very severe. * Limited systemic sclerosis is also known as CREST syndrome: calcinosis, Raynaud phenomenon, esophageal dysmotility, sclerodactyly (thick tight skin), telangiectases.

* There is no good treatment here other than treating reflux as aggressively as possible. Metoclopramide will not help tighten the LES because the LES is now replaced with collagen. Give large dose PPIs. * Nitrates and calcium channel blockers are smooth muscle relaxants, so they would relax the LES. So a patient with chest pain who got worse with nitroglycerine we would suspect GERD once cardiac is ruled-out. * 30yo man with mid-chest pain has had several negative cardiac workups. He continues to have the pain. The cause is likely esophageal motility problem. The most common cause is reflux. There are some others that cause chest pain because the esophagus goes into tight spasm. * Manometry of diffuse esophageal spasm would show diffuse disordered peristalsis. Barium swallow will slow a corkscrew esophagus as there is diffuse non-ordered peristalsis. The term nutcracker esophagus is used because the manometry peristaltic wave pressure can be so high it is said they can crack a nut. * Treatment of diffuse esophageal spasm is nitrates or calcium channel blockers to relax the smooth muscle. * Nobody died of a little bit of esophageal spasm. Work hard to rule-out cardiac causes of chest pain before you jump to the motility work-up. * 40yo man presents to the ED after going to dinner. He complains of severe chest pain and feeling that food is stuck in the mid-chest. He has had no prior episodes except nine months ago he had a single similar episode that gradually resolved over several hours. Since then he has had no heartburn or weight loss. * Episodic mechanical dysphagia (large bolus of food) should suggest a Schatzki ring, also known as steakhouse syndrome. The Schatzki ring is always found just above the LES. The caliber or width of the ring is variable and in general it is soft mucosa, so not a fibrous piece of tissue. Thus, most things get through until you hit the critical diameter like a large piece of steak that does not get through. Barium x-ray would show a ring or band near the LES. If the history is classis though, you can jump to the endoscopy to directly visualize the ring. * Treatment via endoscopy would be removal of stuck food or carefully pushing the food through the soft ring. Pushing the food can break the ring too. Other options include pneumatic dilation. Schatzki ring thought to be due to chronic acid exposure, but the history is not progressive it is episodic. * 75yo man comes to see you because he thinks he has bad breath. He notices people tend to keep their distance from him because of this. This past weekend a most disturbing event occurred while he was watching a football game. He coughed up the chicken teriyaki that he ate two days earlier. He claims to brush his teeth every night. Physical exam is normal. What is the next step in the evaluation of this patient? * Can this be achalasia? Yes it can be. But most likely Zenker diverticulum, more commonly seen in an older patient and associated with coughing and gurgling as soon as they start eating then spit it back up. * Zenker diverticulum most commonly at upper esophagus. Food can sit there for some time but typical history is coughing food back up at the time of eating. * Diagnostic test is barium esophagram showing pocket.

* There is no medical treatment for Zenker diverticulum, treatment is surgical diverticulectomy. This often occurs in the setting of a tight UES, so they also get a surgical cricopharyngotomy (cricopharyngeal myotomy) of the UES. Esophageal Cancer * Plummer-Vinson syndrome signs include upper esophageal web, iron deficiency anemia, and angular cheilosis. It is a risk factor for esophageal cancer; squamous carcinoma since we are at the top of the esophagus. Barium x-ray would show an upper esophageal ring. Think Plummer-Vinson when you see a low Hct and low MCV with upper esophageal issues. * 62yo man comes for evaluation of progressive difficulty swallowing solids and recently semi-solids for four months. He has had a 20lb weight loss. Think cancer until proven otherwise. He has a medical history of significant reflux esophagitis for 15yrs and a 40pack-year smoking history. We dont know if this patient has squamous cell carcinoma or adenocarcinoma, but it doesnt really matter because surgery is the only hope for a cure. On physical exam, you find a 1.5cm left supraclavicular lymph node (Virchow node) but otherwise exam is normal. * Most patients with esophageal carcinoma will give a history of smoking and/or alcohol use. Squamous cancers along with head/neck cancers are associated with chronic alcohol use and smoking. * Virchow node is not common, meaning of all those that have gastrointestinal cancer only a few will have the Virchow node. But, if you find the Virchow node you assume gastrointestinal cancer, almost always adenocarcinoma. The gut lymphatics drain through the left supraclavicular node. * Nodules in or around the umbilicus are also highly suggestive of gastrointestinal cancer. These peri-umbilical nodes are called Sister Mary Joseph nodules. The story is William Mayo (Mayo clinic) was a surgeon at the beginning of the 20th century. All the Mayo nurses at the time were nuns and Sister Mary Joseph did the surgery prep for Dr. Mayos patients. Once in a while she would feel the nodules in the umbilicus. With these think of a gut cancer, the most common being colon cancer in the United States. * So 62yo guy with dysphagia, major weight loss, Virchow node. He has esophageal cancer; no diagnostic test needed to tell us this. Of course you do the diagnostic tests though prior to surgery. So what tests do you do? There is some controversy, but go with barium esophagram first. The barium swallow is done first to help determine if you can get an endoscope through since there is a risk of perforation. The cancer can be anywhere in the esophagus and it usually takes an irregular ulcerated pattern, may show an apple-core lesion on barium swallow. Then do the endoscope with biopsy. Biopsy important near the LES to distinguish squamous carcinoma from adenocarcinoma, but treatment is the same (surgery). * Some evidence that squamous carcinoma responds to chemotherapy and radiation, but that is adjunctive therapy. * Primary treatment for any esophageal cancer is surgery.* Caustic ingestion especially with lye can lead to strictures. Over time the strictures can degenerate into squamous cell carcinoma.

Esophagitis * The most common cause of esophagitis is GERD. Next most common are infectious agents. * Immunocompromise increases risk for infectious agents, so HIV, chemotherapy, rarely steroids. * Common symptom of esophagitis is odynophagia, can be just dysphagia though. * Esophagitis in HIV patient think endoscopy with biopsy. You wont see infectious agents on barium x-ray. * With the exception of the HIV patient, if the patient has esophageal symptoms and thrush on their tongue it is fair to say the patient has esophageal thrush (candidiasis). Rather than put the patient through endoscopy you can treat them with fluconazole. If symptoms relieve in 7 days, you know the patient had candidiasis. * Say the patient has no thrush or had thrush but still has esophageal symptoms a week later. What is the next best test? Endoscopy. There are several types of infectious ulcers that can cause symptoms. * Thrush looks the same in the esophagus, white cottage cheese-like plaques. You can scrape the plaques off and put them on a slide (KOH prep) to see budding forms and hyphae. Treat with fluconazole. * Ulcer seen in esophagus and you want to determine what kind. There is no endoscopic determination that can be made visually to determine what kind of virus. Types that occur are CMV, HSV, and HIV-caused ulcer (least common) all of which are indistinguishable from each other. However, candida is usually in the upper third of the esophagus with small nodules. CMV usually shows severe inflammation with large ulcers. HSV usually shows small discrete ulcers without plaques. * Biopsy will show cytomegalic cells and/or owls eye inclusions (CMV) or ground glass cells and/or Cowdry type A bodies (HSV). Virology can show which grows out but viral cultures are unreliable. * Treatment for CMV esophagitis is ganciclovir or foscarnet. * Treatment for HSV esophagitis is acyclovir, valacyclovir, or famciclovir. * Treatment for HIV-induced ulcer (diagnosis of exclusion) is prednisone. Wait, prednisone for an immunocompromised patient with an infection? Yes, in this particular incidence. It is not from the HIV as the initial insult but due to the inflammation that the HIV causes. Give prednisone for a week and if symptoms disappear you have confirmatory evidence that it was HIV-induced ulcers. * Medical student goes out to celebrate passing USMLE Step 2 and drinks too much then begins to vomit up the steak that they ate. They keep vomiting and bile starts to come up (stomach empty). They keep vomiting and red fluid starts coming up (blood). This is a Mallory-Weiss tear, a longitudinal mucosal tear (not full thickness). * Mallory-Weiss diagnosis is made by endoscopy seeing the lacerations. Endoscopist can do various techniques to stop the bleeding at that point if required. * Most Mallory-Weiss tears resolve on their own, some need endoscopic treatment.

* Boerhaave syndrome is also caused by retching and is much rarer. It is a complete horizontal full-thickness tear of the esophagus casing esophageal separation. This is life-threatening. Pernicious Anemia * Chief role of the stomach is to grind food as only several millimeters can get through the pyrosis. * Water and bicarbonate secreted in the stomach, which are the natural defenses against HCl. Stomach makes pepsin, a protease, so it plays a minor role in digestion of proteins. You can remove the stomach and do just fine with proteins though because the pancreas creates plenty of proteases. Stomach also secretes gastrin, a stimulant from antral cells. Gastrin is secreted and travels through body in bloodstream, arriving at parietal cells in the body of the stomach and stimulating the hydrogen-potassium ATPase pump whereby hydrogen gets secreted into the lumen of the stomach. Parietal cell also secrets intrinsic factor for vitamin B12 absorption. * Histamine stimulates the parietal cell and acetylcholine as well. * We have H2 blockers but no real drugs in clinical practice to block gastrin or acetylcholine at the parietal cell. Anti-cholinergic drugs have poor side effects like dryness and urinary retention. * Acetylcholine comes from nerves so a surgical vagotomy would eliminate one stimulus for acid secretion. The Gcells in the antrum secrete gastrin so removing the antrum (antrectomy) would eliminate another stimulus for acid secretion. Vagotomy and antrectomy are not common anymore due to the prevalence of effective ulcer mediations. * The benefit of PPIs is that no matter what the stimulation to the parietal cell (histamine, acetylcholine, gastrin) it does not matter because the whole cell output is blocked. * Intrinsic factor from parietal cells is essential for absorption of dietary B12. Intrinsic factor is not needed for subcutaneous B12 injections because that does not absorb via the GI tract. Intrinsic factor links with B12 and get absorbed in the terminal ileum. * Pernicious anemia is due to loss of parietal cells and thus loss of intrinsic factor, leading to megaloblastic anemia. Youd lose parietal cells with a sub-total gastrectomy or patient has a large gastric tumor that gets removed. * Autoimmune disease (more common in women) can cause megaloblastic anemia later in life via autoimmune destruction of the intrinsic factor; look for anti-parietal cell antibodies and anti-intrinsic factor antibodies. * Biopsy of patient with pernicious anemia would show atrophic gastritis. * Neutrophils in pernicious anemia will be hypersegmented. Could be folate or B12. * Intrinsic factor from parietal cells goes back and shuts down gastrin, a negative feedback loop. So with no acid, the gastrin goes high to 1000 range. Normal is < 50 blood gastrin, patient on PPI might be 80-100. * Gastrinoma is a tumor comprised of gastrin cells, usually outside the stomach, would also show sky high blood gastrin levels. The gastrin-secreting cells are autonomous (not shut off by acid section), Zollinger-Ellison syndrome.

* There it lots of overlap between gastric and duodenal ulcer symptoms. Trying to distinguish the type of ulcer based on getting better or worse with food, with or without nausea, is not very helpful. Ulcers * Typical symptoms of any ulcer is epigastric pain, relation to food (might be improved or worsened), gnawing pain, boring pain (meaning it feels like it is going to bore through you). * Pain that is better with eating and then gets worse, think peptic ulcer disease. Classic symptom is nocturnal epigastric pain that wakes someone from sleep. * The vast majority of community acquired ulcers, in non-ICU patients who are not taking steroids or drinking alcohol, is infection with Helicobacter pylori. * With H. pylori negative, ulcers are most likely due to NSAIDs. You do endoscopy and see ulcers, gastric or duodenal, then stop NSAIDs and try to make the ulcers better. * NSAIDs come in COX1 and COX2. Non-specific cyclooxygenase (COX) inhibition is seen with older NSAIDs like ibuprofen, naproxen, and asprin. COX-1 metabolites, namely prostaglandins and prostacyclin, are important in the cytoprotection of the stomach. They maintain good blood flow to the stomach and tight gap junctions. * COX-2 metabolites are leukotrienes. So being more specific with COX2 blocking will help relieve aches and pains but not block the prostaglandins that are important for gastric protection. COX-2 inhibitors include celecoxib and acetaminophen. Although there is come controversy on acetaminophen as it does not exhibit antiinflammatory properties, and some label it as COX-3 (but COX-3 does not have antipyretic properties). COX-2 specific medications have been association with vascular events, which was the cause of rofecoxib being pulled off the market in 2004. Mnemonic for -coxib drugs to remember COX2 is -cox and b for 2nd letter in alphabet. * If a patient has rheumatoid arthritis and ulcers, youd want to go with the COX -2 inhibitors. * Misoprostol is an analog of prostaglandin. It is specifically designed to defeat the problem of NSAIDs. Side effects are diarrhea and abdominal cramping. If patient has NSAID-induced ulcer, we use PPIs. If asked what drug will reconstitute prostaglandin activity, then pick misoprostol. * Never give misoprostol to pregnant women or even women of childbearing years, this is a black box warning. Prostaglandins increase intestinal motility and uterine motility. Misoprostol is part of a home abortion kit. Methotrexate is used to kill the fetus and misoprostol is used to evacuate the fetus. In countries where abortion is legal, women can buy these medications to abort early in pregnancy (less efficacious later in pregnancy). * H. pylori is a Gram-negative organism that lives in the antrum of the stomach in the mucus bicarbonate layer. It is not an invasive organism. It secretes urease, creates an alkaline environment around itself to protect from HCl, and has a number of actions leading to ulcer production (e.g. cytotoxic, stimulates meal-induced gastrin).

* If you treat an ulcer but not H. pylori, there is over a 90% chance that the patient will develop another ulcer. So every patient with an ulcer should be tested for H. pylori; do endoscopy with biopsy of the antrum and the pathologist will find H. pylori on the surface of the antrum. * CLO test is taking biopsy tissue from antrum and putting it in yellow gel. Urease will turn the yellow gel to pink. Called CLO because H. pylori was once named campylobacter-like organism. * Non-invasive tests for H. pylori include breath tests to measure for exhaled urease, which is very accurate. Say you had a positive biopsy or breath test and you treat the patient. To determine if you treated the patient accurately you can check a repeat breath test or stool antigen test. Least accurate test is blood IgG looking for antibody, does not differentiate current from past infection, but least expensive test. * Treatment for H. pylori has many regimens, remember you never use a single drug because resistance becomes problematic very quickly. Always use at least two antibiotics, sometimes three. * Bactericidal antibiotics for H. pylori include metronidazole, clarithromycin, azithromycin, amoxicillin and tetracycline. Bismuth also has bactericidal effects. Proton pump inhibitors are useful too. * Mnemonic: MTB, make tummy better, metronidazole, tetracycline, bismuth. * Another regimen is MOC, metronidazole, omeprazole, clarithromycin. * Ulcer complications include pain, bleeding (could ulcerate into a vessel), penetrating (pancreatitis as ulcer bores into back), perforation (surgical emergency), gastric outlet obstruction symptoms (chronic scarring, repeated ulcers). * Perforation would present as an acute abdomen, peritoneal signs, pneumoperitoneum on x-ray and free air under the diaphragm. Obstructing lesion would show scarring and inflammation from the ulcer on endoscopy, biopsy to determine if benign cause versus malignant cause like adenocarcinoma. * NSAID-induced ulcers treated with PPIs, problem-specific treatment are misoprostol; use COX-2 in future. * Antacids may be used in addition to the other medications for ulcer symptom relief. * Most important thing to ask in a patient with an ulcer is does this patient have Helicobacter. Treat patient with multiple antibiotic regimen for about 14 days. Gastrinoma (Zollinger-Ellison Syndrome) * 42yo woman comes to your office with complaints of diarrhea for six months. She has stopped all dairy and milk products for the past few months because another physician told her that her symptoms were caused by a lactase deficiency. There has been no improvement in symptoms. The diarrhea occurs throughout the day and she has not noticed blood or pus with the stools. Her past medical history includes peptic ulcer disease for many years that has been relatively resistant to medical treatment. She takes maximum doses of omeprazole daily along with famotidine and still has symptoms. Physical exam is unrevealing. Lab results are normal except for mild hypercalcemia. What is your next step in evaluation of this patient. * Think gastrinoma when you see diarrhea with multiple ulcers, bad history of ulcers, or other ulcer problems.

* The patient still has ulcers even at maximal PPI therapy. What would cause this? An autonomous constant drive to secrete gastrin, a gastrinoma. * Gastrinoma tumors, often microscopic, are usually found in the head of the pancreas or region of the common bile duct and usually not found in the stomach. * Test to order is a serum gastrin level. Wouldnt omeprazole increase serum gastrin level? Yes, but not to the 100s range with a PPI. With gastrinoma there is no feedback inhibition so serum gastrin is in the hundreds or thousands. * Secretin stimulation test is done by measuring baseline fasting gastrin level after stopping omeprazole, then you inject secretin via IV. In normal people, secretin inhibits gastrin production so you see a fall in serum gastrin. Secretin is released by the small bowel to cut down acid production by causing water and bicarbonate production. If you give secretin IV and gastrin level increases, think Zollinger-Ellison syndrome. * Once high serum gastrin is confirmed by secretin stimulation test, do imagings study like a CT scan looking for the tumor. If you can find the tumor, take it out. Some of the tumors can be microscopic and you can have metastatic ZollingerEllison syndrome rarely. * What if you cant take the tumor out? Do we have a drug that reduces gastrin? Not currently. We do have a drug that suppresses acid so use that; massive doses of PPIs. * The diarrhea is due to the acid spilling out into the pancreatic area, which inactivates the pancreatic enzymes and there is ineffective digestion. * Why hypercalcemia? MEN-1 syndrome associated with parathyroid tumor, causing hypercalcemia. Gastroparesis * The majority of gastroparesis cases are seen in diabetic patients, usually Type 1 diabetes. * Look for other signs of neuropathy like autonomic dysfunction (orthostatic). * Signs of gastroparesis are feeling full very quickly after eating, they feel gassy and bloated, feel progressively uncomfortable throughout the day and may ultimately need to vomit to relieve the symptoms. * There is damage to the neural innervation of the stomach, which is important for peristalsis. * This can be particularly difficult problems for diabetics because they time their insulin doses with eating. The goal of timing is to match when the carbohydrates are digested in the small bowel mucosa. If the food just stays in your stomach for 8 hours, 2 hours later when the regular insulin works the patient has a period of hypoglycemia. Then when the insulin wears off, the food hits the small bowel and the patient gets hyperglycemia. * Treatment includes metoclopramide to improve gastric motility. Side-effect is dopamine inhibition and therefore Parkinson-like symptoms like bradykinesia. * Cisapride no longer used due to increased QT interval leading to torsades de pointes.

* Erythromycin is the next option because it stimulates motilin receptors. The motilin receptor in the stomach responsible for peristalsis gets activated because erythromycin simulates the motilin hormone. * Test is nuclear gastric emptying scan. Patient eats a meal mixed with nuclear chemical. Gamma counter put over patient to see how long food sits in the stomach. You have to do some test, whether x-ray or endoscopy, to confirm you are not dealing with a mechanical obstruction. You would get a prolonged gastric emptying test with gastroparesis or a mechanical obstruction like a tumor pushing up against the gastric outlet. * H. pylori is present in patients with gastric ulcers and duodenal ulcers, associated with gastric adenocarcinoma as well as gastric lymphoma. 95% of cancers in the stomach are adenocarcinoma; about 5% are gastric lymphomas. * Mucosa associated lymphoid tissue (MALT) is a localized lymphoma, there is always an association with Helicobacter. These patients can sometimes be cured with plain H. pylori treatment without chemotherapy, radiation, or surgery. * Patient gets endoscopy and lymphoid tissue is found in the stomach (MALT). Full body workup for lymphoma is negative, no organomegaly, blood counts are good. What do you do? Answer is treat H. pylori, which is always present in MALT. Inflammatory Bowel Disease (IBD) * Inflammatory bowel disease (IBD) covers Crohn disease and ulcerative colitis (UC). * Irritable bowel syndrome (IBS) is far more common than any other bowel condition, is called a syndrome because there is nothing we can point to that looks inflamed or abnormal, usually presents as a younger women (teens, 20s, 30s), symptoms include combination of crampy abdominal pain associated with altered bowel habits (diarrhea, constipation, or more commonly alternating diarrhea/constipation). * IBS thought to be due to altered motility in the bowel, maybe altered sensitivity to pain sensation in the intestines. * IBS treatment is to increase dietary fibers, look for bowel irritants like food and stress. May occur with other syndromes that have vague symptoms like fibromyalgia or chronic fatigue syndrome. * Treatment for IBS is symptomatic as well, so anti-diarrheal agents if needed, stool softeners or laxatives as needed, anti-spasmotics for cramping pain. * By definition, IBS is not associated with any inflammatory signs or symptoms. So you have ruled out inflammatory diseases when you diagnose IBS. The nervous stomach people get before big exams is sort of an irritable bowel symptom. * In IBS, the patient will not have focal findings, no peritoneal signs. More of a vague non-specific exam. No fevers, no night sweats, no weight loss, not guaiac positive (unless they have hemorrhoids from straining if constipated), no mucus or pus in stool, no nocturnal diarrhea, no anemia of chronic disease, no elevated sedimentation rate, etc. * Ulcerative colitis starts at the distal rectum right above the anus in the columnar mucosa, the anus is not involved, there is symmetric circumferential continuous disease for some portion of the colon. If just the rectum, it is proctitis. If rectum and

sigmoid, protosigmoiditis. Disease to the splenic flexure, left sided colitis. Entire colon, pancolitis. * Job of the colon is to absorb water, it is a stool packaging factor. About a liter a day leaves the ileum and enters the cecum, so colon absorbs about 85% of the water giving you 150mL of liquid in the stool. If colon is ulcerated and inflamed, it cannot do its job so you get diarrhea and blood. * UC occurs over weeks to months, bloody diarrhea. Bloody diarrhea for a few days is more likely to be infectious colitis. Older patient for a few days it might be ischemic colitis causing bloody diarrhea. * Crohn disease is also a chronic disease but unlike UC, Crohn is transmural through the full thickness of the bowel wall. UC affects the mucosal layer only. * Crohn disease can affect the colon but specifically there will be terminal ileum involvement. So look for terminal ileum inflammation, terminal ileitis. * Crohn disease symptoms are abdominal pain usually RLQ, diarrhea, some weight loss. Diarrhea can be bloody but more typically is non-bloody diarrhea. * Acute terminal ileitis associated with Yersinia enterocolitica. Yersinia is siderophilic (iron-loving) so patients with hereditary hemochromatosis are more susceptible. Y. enterocolitica is the most common contaminant of stored blood and can live at lower temperatures (e.g. refrigerator). Treatment is antibiotics. * Any ethnic group can have Crohn disease, higher incidence in Jewish particularly of Eastern European origin, specifically Ashkenazi Jews. Many patients with Crohn disease will have a relative with the disease. * UC never involves the ileum and is always continuous, showing lead pipe pattern. Crohn occurs in patches, skip lesions, showing cobblestone pattern. * Test for UC can be done on the first visit via a flexible sigmoidoscopy. You want to see the whole colon so do a colonoscopy. Rule out infectious etiology before labeling them with this life-long disease. Biopsy will show mucosal inflammation, negative stool studies will help rule out infectious agents. * Treatment of UC is sulfasalazine and steroids. Sulfasalazine is sulfa connected to 5ASA. Although 5-ASA looks like aspirin, it has none of those effects (no antiinflammatory outside the gut, no anti-platelet action). Another name for 5-ASA is mesalamine or mesalazine. Side effects of sulfasalazine tend to come from the sulfa portion, whether it is sulfa intolerance with headaches, nausea, vomiting, or rare hemolytic anemia, or rare photosensitivity. * Mesalamine can be given as an enema or suppository as well. * Sulfasalazine, but not mesalamine, affects the sperm count and sperm motility. 32yo woman visits due to problems with fertility. Her medical history is negative but he has ulcerative colitis. Sulfasalazine may be the cause. After stopping the medication sperm counts will return to normal, so a reversible side effect. * Say patient is not improving with sulfasalazine or mesalamine, next step is steroids (oral or rectal). If that fails, admit for IV steroids. If that fails, aggressive doctors may use cyclosporin. Once medical therapy fails, no matter how much of the colon is affected by UC, next step is total proctocolectomy; remove the entire colon.

* Crohn disease is treated similarly. However, the sulfasalazine link of sulfa to 5-ASA is cleaved at the distal ileum making the 5-ASA (mesalamine) available at the proximal colon. So if the patient has a lot of ileum involved and maybe jejunum, use mesalamine alone because it is generally released higher up in the intestinal tract. * Antibiotics often work in Crohn disease although it is not part of the official regimen and Crohn is not considered an infectious disease. Next step is steroids after mesalamine and antibiotics. * Perianal fistulas can occur in Crohn disease. Inflammation can occur in anal crypts and burrow through the soft tissues, emerging to the perianal area or buttocks draining mucus, pus, and/or stool. The specific antibiotic for perianal fistula is metronidazole. * If severe fistula disease not better with metronidazole or severe Crohn disease not getting better with medical treatment, give IV infliximab. Infliximab also approved in rheumatoid arthritis. It is a monoclonal antibody (-mab) against tumor necrosis factor (TNF-alpha). Mnemonic: infliximab inflicts pain on TNF. Infliximab not needed every day, maybe once every few weeks depending on patient. Side effects include a lupus-like drug reaction with antihistone antibodies. * Failed medical therapy in Crohn should go to surgery. Unlike in UC, you try to take out as little bowel as possible with Crohn disease. Far more commonly in Crohn is a localized ileal resection. Surgery also for complications of Crohn like progressive small bowel obstruction or perforation or bad fistulizing disease with abscesses. * IBD, particularly UC in the colon, increases the risk of cancer. Similar to Barrett esophagus, after 10 years of UC disease in the colon the patient requires colonoscopy surveillance to see if the patient is developing dysplasia. Low grade dysplasia is treated by treating UC and seeing if it goes away. High grade dysplasia is confirmed by a second pathologist and treated like cancer, remove the entire colon. If cancer, remove entire colon; this is different than regular adenocarcinoma where you do a segmental resection. With UC cancer you remove the entire colon. * Diagnostic test for Crohn disease is upper GI series with small bowel follow through. Do an upper GI series with barium then give extra barium to get to distal ileum. Look for narrowing, ulceration, cobblestone appearance (more intense ulceration). * Extra-intestinal manifestations of IBD include arthritis (e.g. reactive arthritis of small joints), eye involvement (uveitis, iritis), mouth ulcers (aphthous ulcers), skin involvement (erythema nodosum typically on lower extremity, pyoderma gangrenosum). These usually correlate with the underlying irritable bowel disease. * Extra-intestinal manifestations of IBD that do not follow the condition of the underlying bowel disease include ankylosing spondylitis (bamboo spine, straight/flat back), sacroiliitis, and primary sclerosing cholangitis (sclerosis/fibrosis of the bile duct, young men typically present with jaundice and itching). Infectious Colitis * Infectious diarrhea occurs in enterotoxic and enteroinvasive. * Enterotoxic (E. coli, Vibrio cholera) watery diarrhea without mucus or pus or blood or fever. Treat with fluids, ETEC E. coli can respond to antibiotics like TMP-SMX or

quinolones. ETEC is the most common cause of travelers diarrhea. Salmonella can be enterotoxic in one form. * Enteroinvasive invade the mucosa (shigella, entamoeba histolytica, campylobacter) bloody diarrhea with fecal leukocytes and fever. Shigella dysentery has typical blood and pus in stool. Entamoeba is a parasite typically in the ileum or colon, acute onset, probably in a traveler or people having anal intercourse. For entamoeba, do rectal parasite testing looking for ova or protozoa. * TMP-SMX works for most bacteria and quinolones work for nearly all causes of enterotoxic or enteroinvasive bacterial diarrhea. * Yersinia enterocolitica can mimic Crohn disease or appendicitis. * Giardia is a parasite that lives in upper intestine (duodenum, proximal jejunum), typically traveling, maybe hiking or drinking from a stream, Saint Petersburg Russia. Symptoms are belching, gas, nausea, vomiting, diarrhea, foul smelling stools. Treatment of giardia is metronidazole. Side effect of metronidazole is upper GI discomfort (nausea), headaches, metallic taste, numbness/tingling due to peripheral neuropathy, disulfiram-like reaction with alcohol causing bad nausea/vomiting (rare but serious). * Campylobacter jejuni is one of the most common causes of enteritis, treat with quinolones or erythromycin. * All of these diagnoses are made by stool studies. * Patient has blood diarrhea, looks ill, serum creatinine is 2.5, hematocrit is at 19%, hemoglobin is 6 grams. This is hemolytic uremic syndrome associated with E. coli O:157 (EHEC). Secretory & Osmotic Diarrhea * Secretory diarrhea means there is some secretory drive for the intestine to put out lots of water. Enterotoxic E. coli or cholera are secretory forms of diarrhea. * Secretagogue is something that causes secretion, such as a toxin, gastrin, VIPoma (vasoactive intestinal peptide), calcitonin. * Young female patient has supposed secretory diarrhea. Youve done all the studies and hormone assays but found nothing. Think factitious diarrhea, patient is taking laxatives. Patient may have multiple operations that they dont have good reason for, like cholecystectomy for vague symptoms, appendectomy, 3 exploratory laparotomy operations. Patient may have connection to the medical field (works in drug store, paramedic, nurse). * Osmotic diarrhea means there is an osmolar substance that is not absorbed in the small intestine, so the small intestine tries to dilute it by adding water into the lumen of the small bowel. * Most common osmotic agent is lactose, milk sugar. Enterocytes have villi and microvilli with lactase enzyme. With lactase deficiency, gut tries to dilute nonabsorbable lactose. Bacteria can ferment the lactose so patient can get a lot of gas and discomfort as well. * Stool Osmolar Gap is a calculation you will probably only see on the boards, even if youre a gastroenterologist.

* Measure liquid stool sodium and potassium (if you can trick the lab tech into placing it into the machine where normally urine or serum goes) youd get say Na+ 130 and K+ 7. To get stool osmolality 2*(Na + K) so 274 here. * Now measure osmolality directly for the stool. Say it is 275, close to 274. So besides the Na+ and K+ there is nothing else that has osmolar properties in the stool. So not likely osmotic diarrhea, probably secretory. * Say you measure 2*(Na + K) as 274 and osmolarity is 350, so gap of 76mosm of something that does not belong there, so this is an osmotic diarrhea. * In clinical practice, you can differentiate osmotic from secretory diarrhea from clinical presentation and initial laboratory testing. Pseudomembranous Colitis * 59yo construction worker is admitted to the hospital for treatment of cellulitis that developed after dropping a steel beam on his leg. He is treated for 7 days with IV cefazolin with dramatic improvement in his symptoms. He returns home and does fine until one week later when he has 6 watery green bowel movements a day and a low grade fever. Exam shows vague mile left mid-abdominal pain. What is your next step in the evaluation of this patient? * Top of the differential list should be pseudomembranous colitis (C. difficile colitis). Normally C. difficile does not grow in the gut but when you wipe out the normal flora with antibiotics the C. diff can start to flourish. * C. difficile causes a secretory diarrhea, usually non-bloody, colonic (left sided abdominal tenderness), spectrum from well with mild diarrhea to very toxicappearing with high fevers. * Pseudomembranous colitis associated with a very high white count (e.g. WBC of 30-50,000). * Ways to get C. difficile are being in a hospitalized/nursing home setting, receiving antibiotics, or being exposed to diarrhea (nosocomial infection). Alcohol-based hand sanitizers do not remove C. difficile so hand-washing with soap and running water is required, and even that is not perfect because the spores are so sticky. * Direct way to make the diagnosis would be to put a sigmoidoscope in the patient and look for pseudomembranes, which are yellow exudative plaques on the colon. This appearance does not look like anything else so you can tell the patient right there that they have pseudomembranous colitis. * Everyone with pseudomembranes on exam has C. difficile, but not all patients with C. difficile make pseudomembranes. In a negative sigmoidoscopy, do a toxin assay. Add stool to plate, if cytotoxic then we know it is coming from C. difficile. If very sick patient, go for sigmoidoscopy. If not very sick, can wait for toxin assay. You may get called to the ICU for a very sick patient who has been on many antibiotics, they dont want to wait for a few days they want to know now so get your sigmoidoscope and get moving. * Treatment is metronidazole first-line, given oral. C. diff lives in the lumen of the gut so you want to clear it from the lumen of the gut. IV metronidazole gets into blood stream but not really to lumen of the gut. Next step is vancomycin, PO (oral) is very effective. Vancomycin is more expensive than metronidazole. There are also strains

of VRE (vancomycin-resistant enterococcus) superbugs so metronidazole is firstline. There are also strains of vancomycin-resistant staphylococcus. * Most C. difficile patients do not have sky-high white counts, but if it is present it points you to C. difficile. * If patient cannot tolerate metronidazole (e.g. terrible neuropathy) use vancomycin. * Antibiotic-associated colitis (non-pseudomembranous) is diarrhea associated with the change in flora that occurs with antibiotic use. Stop antibiotics in these cases. * Causes of C. difficile colitis includes clindamycin (most likely). Also cephalosporins or penicillins because we use those medications more than clindamycin. * Patient do not develop shigella, salmonella, campylobacter in the hospital. Patient in the hospital is likely nosocomial or medications, so look at medication list for antibiotics. Medication causes include magnesium containing drugs, quinidine, colchicine, gold salts for rheumatoid disease, antibiotics as a side-effect, etc. Ischemic Colitis * 72yo man complains of LLQ pain for the past few days. He has had diarrhea for the past 5 days, stools bloody over last 48 hours, and has developed a temperature. History is significant for peripheral vascular disease and a myocardial infarction. Patient is on isosorbide dinitrate, propranolol, aspirin, diltiazem, glipizide, metformin. Exam shows some tenderness, rectal exam reveals bloody stool. What is the most likely diagnosis? Answer is ischemic colitis. What vessel is most likely involved? Inferior mesenteric artery (IMA) serving distal transverse colon to midrectum, where the hypogastric or internal iliac artery takes over. Where is the watershed area? At the splenic flexure, where the superior mesenteric artery (SMA) stops sending branches and IMA starts sending branches, also in midrectum where IMA stops giving branches and hypogastric starts giving branches. * Ischemic colitis should be at the top of your list in an elderly patient developing acute bloody diarrhea. Other thing on the list is infectious colitis. * Treatment is usually medical for IMA ischemia, patients generally get better. * Young patient with acute onset of diarrhea is most likely infectious. Less likely is ischemia, patient would have to be hypercoagulable since they likely do not have atherosclerotic disease. History might be use of oral contraceptives, or pulmonary embolism at one point, or other hypercoagulable event with no risk factors. * Three major gut branches off the aorta are the celiac, SMA, and IMA. Celiac feeds gastric, splenic, hepatic, superior pancreaticoduodenal. SMA feeds intestinal, ileocolic, right colic, middle colic. * Losing the SMA would infarct the entire small intestine. You would probably die from the gangrene and if you did survive youd have to be on IV feedings forever. * If IMA is lost, you see the localized or segmental colitis in the distal colon and sigmoid, negative stool studies. * SMA ischemia risk factors include atherosclerotic heart disease, bad CHF with poor flow, hypercoagulability (less likely), irregular heart rhythm that puts patient at risk for thrombi (e.g. atrial fibrillation), digoxin use. * SMA ischemia symptoms are centrally located abdominal pain that is out of proportion to physical exam. The patient is writhing around as if they have a

perforation, but abdominal exam is soft without guarding. This is the preperforation pre-gangrene stage. * If SMA ischemia is suspected, do an abdominal aortic angiogram. If ischemia is present and severe, go straight to surgery. * Patient may have fever, leukocytosis, acidosis (lactic acidosis from poor perfusion). * Ischemic bowel is a useless description since SMA and IMA ischemia are very different presentations. IMA has bloody diarrhea, elderly patient with atherosclerotic disease, get better with medical management. SMA has usually non-bloody diarrhea, horrible central abdominal pain, needs angiogram then surgery. Malabsorption Syndromes * Lots of diseases can cause malabsorption, but here were talking about malabsorption as the presenting symptom. * The small intestine does the absorption, so the malabsorbed material will be from the small intestine. * Symptoms include weight loss, diarrhea, increased food consumption, nonspecific pain, greasy or fatty looking stool, malodorous stool (due to fat). * Malabsorption is an osmotic diarrhea. * Most things absorbed past the ligament of Treitz. Things absorbed only at the terminal ileum are B12 linked to intrinsic factor and bile salts. We need grams of bile salts to absorb fats and 2/3 of those are reabsorbed in the ileum up to the liver (enterohepatic circulation). * Protein malabsorption signs are hypoalbuminemia and peripheral edema. * Fat soluble vitamins are in the attic: A, D, E, K. * Vitamin A deficiency will show disturbances in night vision and hyperkeratosis. Vitamin D deficiency will show hypocalcemia, osteopenia/osteoporosis with no reason (e.g. healthy young man, premenopausal woman). Vitamin E deficiency (rare) will show nystagmus and ataxia. Vitamin K deficiency will show problems with clotting factors II, VII, IX, and X. Vitamin K deficiency measured by protime (PT) or INR, factor VII has the shortest half-life * Three things if not absorbed in proximal ileum are not well compensated for are folate, iron, calcium. * Mnemonic is FIC for folate, iron, and calcium. * If proximal small bowel is affected alone, like celiac sprue, patient will have signs of FIC deficiency like megaloblastic anemia (folate deficiency) or hypochromic microcytic anemia (iron deficiency). Patient might have both so the MCV might be normal but the width spread would be high (high RDW or RBC distribution width). Patient may show signs of calcium deficiency like osteopenia or tetany (Chvostek or Trousseau sign). * Anisocytosis is the term used for mixed RBC widths (i.e. high RDW). * Celiac sprue is the most common cause of chronic malabsorption. Other major cause is tropical sprue. Less common (very rare) is Whipple disease. * Celiac sprue typically presents in childhood with malabsorption. Presents again as an adult, patient may say they had to watch their diet as a kid and get food from special bakeries, but now feels sick again (honeymoon period).

* Pathophysiology of celiac sprue is autoimmune disease with destruction of villi showing villous atrophy with blunting or complete loss of villi showing crypt hyperplasia. There will be inflammatory cell infiltration such as lymphocytes, plasma cells. * Celiac sprue antibodies are against villi, look for anti-endomysial (most sensitive, specific), also known as antitransglutaminase (tGT). Also anti-gliadin antibodies seen in sprue, older test and less specific. Gliadin is a property of wheat, barley, and rye. Some patients cant tolerate oats either. * Treatment is follow non-gliadin diet. This prevent the malabsorption and the longterm complication including intestinal lymphoma and intestinal adenocarcinoma. * Definitive diagnosis is based on small bowel biopsy. After removing gliadin from diet the villi grow back. * Tropical sprue affect patients in the tropics (Caribbean, India, Asia) who comes back and now has malabsorptive symptoms. It looks like celiac sprue if you were to do a small bowel biopsy, no villi. But patient does not get better on gliadin-free diet and no anti-villi antibodies. * Treatment is broad antibiotics like metronidazole, tetracycline, fluoroquinolone. * Tropical sprue affects distal small bowel, where B12 is absorbed. So these patients will have B12 deficiency showing megaloblastic anemia. Celiac sprue patients also get megaloblastic anemia but that is due to folate. * A viral gastroenteritis can wipe out the villi for a few days but they come back in a week or so. So the diagnosis of sprue is not made from villous atrophy alone, you need the clinical features too. * Testing for malabsorption in general includes the 72h fecal fat collection, but it is rare that you get one of these completed in the hospital because the collection can will stink up the entire room by the third day. More likely to get this done at home. Tell patient to eat a diet with a lot of fat (100g/day) and collect the stinking steatorrhea stool and go to the lab, where it gets weighted for large volume then measured for fat. Even on a high fat diet, you should have less than 7g/day of fat in the stool. Say you see 50g/day then the patient is malabsorbing fat. * Major enzyme classes all come from pancreas: amylase to digest carbohydrates, proteolytic enzymes like trypsin, elastase, chemotropism, and fat digesting enzymes like lipase. * Most common disease causing insufficient pancreatic enzymes is chronic alcoholic pancreatitis. This is exocrine insufficiency of the pancreas. The patient would put lots of fat into the 72h stool collection. X-ray plain film can show calcifications across the midline where the pancreas sits; this is pathognomonic for chronic pancreatitis. Small bowel absorption in chronic pancreatitis would be normal. * Test for small bowel absorption is the D-xylose test. D-xylose is a sweet tasting sugar that does not appear in the regular diet. 50g of D-xylose powder is added to water and patient drinks it, blood measured 2h later. If normal absorption, there will be D-xylose above a certain level in bloodstream. If no absorption, no D-xylose in blood. Could also do a 5h urine collection (D-xylose in urine if normal, no D-xylose in urine if malabsorption).

* D-xylose depends on nothing except an intact small bowel. There is no need for active transport or breakdown by digestive enzymes. * Whipple disease presents as malabsorption symptoms plus other organ involvement. Example would be CNS involvement (dementia), skin involvement (hyperpigmentation), lymphadenopathy, cardiac involvement, joint involvement. * Whipple disease diagnosis confirmed on small bowel biopsy showing PAS-positive macrophages. Tropheryma whipplei bacteria can be seen on electron microscopy. * Treatment for Whipple disease is broad-spectrum antibiotics like metronidazole, tetracycline, fluoroquinolones. Diverticulosis & Diverticulitis * Diverticulosis can occur anywhere in the colon but usually it occurs in the sigmoid because that is the area of greatest pressure generation by the muscles of the large intestine (LaPlace law, pressure against pipe wall). * Most people over the age of 60 have diverticulosis but they are asymptomatic in general. If you do a colonoscopy and see the diverticuli just leave them alone as long as patient is asymptomatic. * Diverticular spasm due to contraction against hard stools can cause pain, LLQ postprandial pain that resolves after bowel movements. Treat with high fiber diets to make stools bulkier giving some symptomatic relief. * Diverticular bleeding can occur in the dome of the diverticuli. Patient presents with brisk arterial bleed; the diverticuli dont bleed to give iron deficiency anemia or guaiac positive stool, they present with sudden painless bright red blood per rectum (BRBPR, hematochezia). * Diverticuli can rupture or perforate, spilling fecal contents out. This causes diverticulitis, inflammation. If the rupture is well contained you can see bowel wall thickening on CT scan. If not well contained you can see abscess in the wall of the colon (intramural abscess) or more severe pericolonic abscess. If bad perforation, patient gets fecal peritonitis and will be very sick. * Diverticulitis is usually in an older patient, LLQ pain of relatively acute onset, LLQ findings on an exam even looking like a left-sided appendicitis (appendix is just a diverticulum on the right side), tenderness, localized rebound, fever, leukocytosis, and no bleeding. * Diverticulitis is a localized perforation with abscess and no bleeding. Do not confuse this with diverticular bleeding presenting with hematochezia, no fever, no leukocytosis. * If patient not too sick, we treat with IV antibiotics effective against anaerobes and gram-negatives like ampicillin, metronidazole, and third-generation cephalosporin. Fluoroquinolones used as well. Gentamycin not used much. * If diffuse peritonitis, start IV antibiotics and go to surgery to clean up mess and resect area that is perforated. * Diverticulitis diagnostic test is CT scan. Diverticulosis can be sigmoidoscopy, but it might be difficult to see when blood is coming at you. Rule-out hemorrhoids then maybe CT scan if cannot do sigmoidoscopy.

* Do not do endoscopy or barium/air enema with suspected diverticulitis because you can worsen the perforation. Constipation & Colorectal Cancer * Constipation is defined as less than three bowel movements in a week. Constipation is a broad workup however, it depends on the patients normal daily bowel movements. Better measurement would be too few bowel movements and bowel movements that are too hard. * Regular constipation can be treated with increased fiber and laxatives. Here we are discussing diseases that present with constipation as the primary symptom. * Older patient with acute change in bowel habits (constipation) think colon cancer until proven otherwise. * Hirschsprung disease is rare in children and even more rare in adults. Constipation in children is most commonly caused by regular constipation. With severe constipation though, worry about Hirschsprung. * Hirschsprung disease is a congenital loss of the myenteric plexus and the internal anal sphincter fails to relax. The baby is squeezing to empty their bowel and over time the more proximal bowel gets dilated. * Diagnostic test for Hirschsprung is barium enema showing dilatation of colon above tight contracted sphincter. Diagnosis is confirmed with full-thickness biopsy of rectum showing loss of myenteric plexus. Treatment is surgery. * Chagas disease (Trypanosoma cruzi) can cause megacolon and achalasia pattern due to loss of myenteric plexus from acetylcholine fibers. Chagas can also cause cardiomyopathy and myocarditis. * Older patient with blood in the stool, change in bowel habits (constipation), thin caliber stools (pencil thin), abdominal pain, distention (if well advanced), iron deficiency anemia (might be first symptom). * Any older person with iron deficiency has colon cancer until proven otherwise. Do most of those patients actually have colon cancer? No, but you cannot miss this diagnosis. * Test of choice if you suspect colorectal cancer is colonoscopy. You can remove polyps and biopsy any mass. * Risk factors for sporadic colorectal cancer include Western diet (low fiber, high red meat), family history of colorectal cancer, adenomatous polyps, ulcerative colitis, extensive Crohn disease, hereditary polyposis syndromes (familial polyposis coli, autosomal dominant), non-polyposis familial cancer syndromes (Lynch syndrome, pancreatic cancer, breast cancer, ovarian cancer). * If family history is strong with adenocarcinomas, think about hereditary nonpolyposis cancer syndromes. Specific genetic defect, autosomal dominant. Patients develop colorectal neoplasms at a younger age and with a higher likelihood. These patients get early colorectal cancer screening. * Peutz-Jegher is non-adenomatous polyps, they are hamartomas, hyperpigmentation around lip corners. * Juvenile polyposis syndromes are also hamartomas, not adenomas, generally resected.

* Gardner syndrome is large bowel cancer with non-colonic involvement, soft tissue tumors, desmoid tumors, osteomas of the skull, bone tumors, tumors of papilla of Vader. Early screening if family history. * Most common cause of cancer in women is lung cancer. Breast cancer is most common in non-smoking women and second most common in all women. Colorectal cancer is second most common in non-smoking women and third most common in all women. * Most common cause of cancer is men is lung cancer. Prostate cancer is next, third is colorectal cancer. * The only preventable cancer of those top three is colorectal cancer. In lung cancer, we find the cancer not the precancer. With mammography we hope to detect early cancer, but it is still cancer. PSA or prostate biopsies are not pre-cursor lesions, BPH is not a pre-cursor. In colorectal cancer we are finding a truly premalignant condition that we can remove and eliminate the cancer risk. * The premalignant condition for colon cancer is the adenomatous polyp. We could eliminate colorectal cancer (except familial forms) if we removed every adenomatous polyp in this country. 90% or more of colorectal cancer is sporadic where the only history is polyps and/or first degree relative with polyps or colorectal cancer. * Screening tests can change over time. Even now your screening could depend on what organizations recommendations you follow. * Everyone should have an annual digital rectal examination beginning at age 40. Occult blood with guaiac testing should occur annual beginning at age 50. At the least, everyone should get a barium enema and flexible sigmoidoscopy at age 50. Most recommendations say colonoscopy at age 50 for everyone because it is more sensitive and allows you to manage the polyps that you find. * Around 30% of Americans have adenomatous polyps after the age of 50. * Sigmoidoscopy with barium enema is less expensive and less likely to have a complication such as a perforation. Colonoscopy involves IV sedation so there is risk there too. * Negative colonoscopy at age 50, patient can wait 10 years until next colonoscopy. * Virtual colonoscopy is a CT scan of the colon and is not standard of practice currently. * Polyp seen on colonoscopy can be sessile (broad base on wall) or pedunculated (on a stalk). Tubular is a histologic description; do not call it tubular because it looks like it has a tube. * If polyp is hyperplastic, it is not premalignant. * If polyp is adenoma, pathologist will give you additional information. They will say if it is tubular histology or villous histology. Villous histology has a higher potential to become malignant. * If adenoma polyp is found, patient has to come back to make sure no new polyps have grown back. Interval is controversial, say 3-5 years if small polyp. * If biopsy of a polyp (polyp not removed) and it is adenoma then you need to go back and complete the polypectomy. Example would be a general surgeon or non-

gastroenterologist doing a sigmoidoscopy and doing a biopsy of a polyp instead of polypectomy. Then you would have to do a complete colonoscopy to clear colon. * The better we get with colon cancer screening and polyp removal, the less likely Americans are to die from colon cancer. A couple decades from now colon cancer may be deeper down the list instead of the third most common cancer in men and women. * Malignancy found on colonoscopy, patient needs to go to surgery and have cancer removed with clean margins on each side and management of blood vessels, anastomosis of colon ends together, then think about whether the patient needs chemotherapy or not. * Chemotherapy is never curative for colon cancer, you have to remove the cancer first before chemo. * Duke A cancer is limited to the mucosa, patient cured almost for sure with surgical resection. * Duke D cancer is distant metastasis, goes locally to lymph nodes then liver (not lungs or bones without going through liver first as a rule). * Duke B cancer goes into layers of the bowel wall but not through it, past mucosal layer. Layers are mucosa, submucosa, the muscularis mucosa. B1 means not full thickness, B2 means full thickness but not through. * Duke C cancer does not go through full thickness but has lymph nodes. C1 means not full thickness and with lymph nodes, C2 means full thickness with lymph nodes. * When do we give chemo? Duke A patient is cured so not needed. Duke D patient is too late. Chemo should be given for Duke C as the cancer is localized but not systemic. Duke B is controversial; B1 no chemo, B2 maybe. * Duke C is the staging that has increased survival with chemotherapy. * Chemotherapy drugs for colorectal neoplasm are 5-fluorouracil (5-FU) and leucovorin (folinic acid) or levamisole. Leucovorin is used as adjuvant therapy working synergistically with 5-FU. Patients can tolerate these medications so useful for outpatient chemotherapy from 6 months to a year. * Radiation is useful for advanced rectal cancers, either pre-op or post-op to improve long term survival. * Anal cancer is squamous cell, particularly seen in homosexual men with HIV. HPV is the precursor to cervical cancer in women. HPV warts (condyloma accuminata) are precursors to anal cancer. Squamous mucosa does respond to chemotherapy and radiation. Early anal cancers can be cured without surgery using chemotherapy and radiation instead. Rectal cancer on the other hand needs surgery and radiation, maybe chemotherapy depending on the stage of rectal cancer. Gastrointestinal Bleeding * Upper GI bleeding means the bleeding is originating from proximal to the ligament of Treitz, which is the junction of the duodenum and jejunum. Most upper GI bleeds are near the esophagus, stomach, or duodenal bulb. Beyond the duodenal bulb acid is neutralized so you do not get ulcers there. * Cause of upper GI bleeds includes esophageal varicose veins, Mallory-Weiss tears and ulcer disease.

* Varicose veins is due to portal hypertension from liver disease, such as alcoholic liver disease or hepatitis C induced cirrhosis. The veins are collateral circulation that become engorged; hemorrhoids too. * Presentation can be coffee ground emesis (blood), hematemesis, melena (acid degrading blood, black tarry foul smelling stool), and even hematochezia since blood is a GI irritant and would have rapid GI transit. * Other causes of dark or black stool are patients taking iron supplements and patients taking oral bismuth. * Any cause of upper GI bleed patient will become tachycardic and have orthostatic hypotension with volume loss then hypotensive and shock-like symptoms with further blood loss. * First step in management of active GI bleeding is start large bore IV and give volume, send blood for type and cross, get blood bank ready if needed. * Test of choice is upper GI endoscopy. Upper GI barium series has no role in the management of active upper GI bleeding. Endoscopy allows you to make the immediate diagnosis and even treat the source. * Medical therapy is not as useful for controlling GI bleeding. IV PPIs might help. Exception is variceal bleeding. * Management of choice is endoscopic hemostasis. * If esophageal varices, endoscopist can inject a sclerosing agent or place rubber bands over veins (banding). * Medical management for esophageal varices specifically is octreotide. Octreotide is a synthetic analog of somatostatin and reduces portal pressures. Octreotide is 8 peptides, given as IV bolus then hourly infusion typically for three days if the patient has esophageal variceal bleeding. * Lower GI bleeding is defined as distal to the ligament of Treitz, but very few things bleed in the small intestine. * Exception is children or young adults with negative colon testing and painless blood emanating through ileocecal valve, you should suspect Meckel diverticulum. Gastric mucosa arrives in the diverticulum and acid production can cause ulceration of the diverticulum causing bleeding. Bleeding events may be episodic. * Diagnosis of Meckel diverticulum done with a bleeding scan that labels gastric mucosa, called Meckel scan. You would see gastric cells light-up in the small bowel. Treatment is resection, can be done laparoscopically. * Most common cause of mild (small amount) painless rectal bleeding is hemorrhoids or fissures. A local external exam or short-scope anoscopy can show hemorrhoids or fissures that are bleeding. * Hematochezia or maroon stool (not BRBPR, no melena, in between) is likely colonic bleeding. In elderly, next most common is diverticulosis then arteriovenous malformations (AVMs, angiodysplasia was previous name). AVMs more commonly seen in patients with aortic stenosis (controversial idea). * Diverticulosis usually found in sigmoid. AVMs usually found in ascending right side of colon; painless bleeding.

* If major bleeding, next step is stabilization. Start large bore IV(s), give fluid, type and cross blood. * A clue that rectal blood is from an upper GI bleed is an increased BUN/Cr ratio. BUN/Cr around 40:1 because small intestine absorbs some of the blood and breaks it down to urea, so BUN raises out of proportion to creatinine. Blood is not absorbed in the colon so BUN/Cr would not increase with a straight colonic bleed. * You can do a sigmoidoscopy first in a patient that is actively bleeding because you do not necessarily need to prep the patient (bowel prep). You can see anorectal bleeding or on occasion see diverticular bleeding; so you see the bleeding diverticuli then go higher and see no blood, just formed brown stool. That tells you that the blood is occurring from a distal area and since there is lots of blood in the region of the diverticuli you assume this is a diverticular bleed. * Management is to prep the patient for a colonoscopy (bowel prep) then do a colonoscopy as bleeding can occur all the way to the right colon. Sigmoidoscopy can be done immediately, but might not be diagnostic. * The colonoscopy can show the location of the bleed, or during the prep the bleeding could have stopped. Many people have diverticuli that do not bleed, so if you see them do not assume that was the cause. * Say you do flexible sigmoidoscopy and it is non-diagnostic. You do colonoscopy and it is non-diagnostic. Next step if the patient is actively bleeding is a bleeding scan where nuclear label is mixed with the patients red blood cells, injected back into the patient, then looking for the bleeding source; if the cells are leaking out somewhere they will accumulate in that area. * A red blood cell scan in the abdomen should show you the vasculature normally, the aorta and internal iliacs. You should not see a blush or collection on the right side, which would imply right colon, typicaly AVM. Say the patient his diverticuli along the whole colon and you see lots of blood on colonoscopy, then the bleeding scan can help localize the problem area. * Most diverticular bleeds stop spontaneously and do not require surgery. If the patient continues to bleed, they require a resection. Hopefully the bleeding scan pinpoints the area for localized resection. If the patient stops bleeding from diverticular disease and then restarts, we generally send the patient to surgery. * If the bleeding scan is non-diagnostic and the patient continues to bleed from diverticular disease, the surgeon is stuck and has to do a sub-total colectomy taking out the whole colon down to the rectum. * Angiography is rarely used. It is done if the nuclear scan cannot be done or the nuclear scan is not diagnostic. An angiogram requires active bleeding to be positive. It requires more active bleeding than a nuclear scan, which is why the nuclear scan is done first. Also nuclear scan is less invasive and can be repeated over several hours. Another use of angiography would be an active bleed where you are worried about the patient sitting in nuclear medicine for a while as they might bleed too much. * During angiography, physician can inject the blood vessel locally with the angiogram catheter or maybe put a coil in to stop the bleeding.

Pancreatitis * Acute pancreatitis presents as abdominal pain, usually epigastric with/without radiation to back, associated with nausea and vomiting. Exam shows tenderness in mid-epigastric region, sometimes diffuse tenderness. * Nearly all cases of acute pancreatitis are caused by alcohol or gallstones. Alcohol is a direct toxin to the pancreas causing inflammation. * Blood test abnormalities are described in Ranson prognostic criteria, which was originally developed just for alcoholic pancreatitis. Severe pancreatitis is likely if score 3+. Mortality 100% if score 7 or 8. * Mnemonic (admission): GA LAW, glucose, age, LDH, AST, WBC count. * Mnemonic (48 hours): C HOBBS, calcium, hematocrit, O2, BUN, base deficit, sequestration of fluid > 6L. * Initial labs should include amylase and lipase, lipase is more specific. Initial testing can include CT scan. * Treatment is to put the pancreas at rest. NPO because food stimulates the pancreas, IV hydration, correction of hypochloremic hypokalemic metabolic acidosis, NG tube if need to decompress the stomach (if vomiting). * Gallstone pancreatitis is usually associated with a history of biliary colic (crampy RUQ pain after meals). A stone would need to block either the common bile duct and/or the pancreatic duct (choledocholithiasis). All the proteolytic enzymes cannot leave the pancreas so you get autodigestion of the pancreas by the enzymes. * Treatment of gallstone pancreatitis that does not resolve on its own is endoscopic retrograde cholangiopancreatography (ERCP) with stone removal. Bile and even pus may pour out and patient gets better. * Common bile duct obstruction with cholangitis should be treated via ERCP stone extraction. Cholangitis is classically with Charcot triad of RUQ pain, fever, and jaundice. Bilirubin may climb to 5 or 6 here. * Treatment for cholangitis should also include antibiotics. * Inflamed swollen boggy pancreas is called a phlegmon (spreading diffuse inflammatory process with formation of suppurative/purulent exudate or pus). Disruption of the duct with severe pancreatitis will release fluid that forms into a cyst called a pseudocyst (pseudo because not lined with true epithelial cells). Pseudocyst can cause obstruction, pain, fistulization into the lung causing pleural effusion (send fluid for amylase), and localized pancreatic abscess occurring in a few days. * If patient is not getting better after a few days of NPO and IV fluids for pancreatitis, do a CT scan to rule out pancreatic abscess. If pancreatic abscess, they need surgical debridement of the abscess or even radiology-guided needle aspiration. For ICU patients, you may need to do follow-up CT scans and drain any new cysts/abscesses. * Gallstone pancreatitis never leads to chronic pancreatitis because patient either gets better or goes for stone extraction with ERCP. Chronic pancreatitis is usually associated with alcohol use, exocrine pancreatic insufficiency. Calcium becomes deposited through duct system of the pancreas and can be seen on plain film indicating chronic calcifying pancreatitis.

* Chronic pancreatitis can present with exocrine insufficiency as maldigestion and we treat them with pancreatic enzyme replacement. They can present with endocrine insufficiency as diabetes with insulin dependence from beta cell destruction, so insulin dependent diabetes but not Type I, it is due to chronic pancreatitis. Other important symptom of chronic pancreatitis is chronic abdominal pain, which is problematic because in general they are alcohol addicts and you run the risk of making them narcotic addicts from iatrogenic prescribing. If the pain is very severe, patient can get an operation called a Puestow procedure. * Puestow procedure (longitudinal pancreaticojejunostomy) can be useful to prevent narcotic addition or to help a patient who is already addicted to prescription narcotics. Procedure filets open the pancreas then sew a loop of jejunum into the bottom of the pancreas, now pancreatic enzymes drain right into the jejunum preventing irritation of the pancreas from trapped enzymes. Liver Disease * Liver is an important synthetic site for many compounds such as albumin and coagulation factors. It is also an important detoxification organ and nutrition absorption organ. Bilirubin is also metabolized here. * The SMA feeds the small intestine and the SMV, after taking in intestinal nutrients, meets with splenic vein into the portal vein and through liver. * RBCs last about 120 days and get trapped in spleen in the reticuloendothelial system as they break down. Heme part of RBCs is broken down to biliverdin then to bilirubin. Bilirubin bound to albumin travels via portal system to liver hepatocytes where it is conjugated to glucuronic acid giving bilirubin glucuronide. That bilirubin is the one that ends up in your bloodstream making you yellow or into kidneys making urine tea or cola colored. Albumin bound bilirubin coming out of spleen is not water soluble and is not urinated, thus high unconjugated bilirubin will not darken the urine. * The colors of a bruise are caused by the phagocytosis and sequential degradation of hemoglobin to biliverdin to bilirubin to hemosiderin, with hemoglobin itself producing a red-blue color, biliverdin producing a green color, bilirubin producing a yellow color, and hemosiderin producing a golden-brown color. * Total bilirubin is direct + indirect bilirubin. Indirect is unconjugated bilirubin. Direct is conjugated bilirubin. * Say total bilirubin is 5.0 and indirect is 3.8. Then you know most of the bilirubin is unconjugated. Say total bilirubin is 10 and indirect is 9, then you know this is not a liver problem (spleen problem, hemolysis).

* Gilbert (pronounced jheel-BAYR), Crigler-Najjar, and Dubin-Johnson syndrome are problems with indirect - 64 bilirubin. The only one you will probably see in clinical practice is Gilbert syndrome. Gilbert syndrome is a problem with the uptake and conjugation due to deficiency of glucuronyltransferase. Chief problem with Gilbert syndrome is you see the bilirubin (2-4, almost all indirect) and worry about liver problems. Patient is asymptomatic but may get mild jaundice (e.g. scleral icterus) at times of stress. Reticulocyte count is normal, no signs of hemolysis. Tell patient about the syndrome to help prevent medical testing in the future (ultrasounds, biopsies). * Crigler-Najjar and Dubin-Johnson syndrome are very rare and are problems secreting bilirubin from the hepatocyte, there will be a high indirect bilirubin. Crigler-Najjar presents at birth and requires major medical and surgical therapy (phototherapy 12h/day, liver transplant, transfusions). DubinJohnson prognosis is much better with most patients living normal lives, post-mortem liver may look black or dark-pink. * Blockage of the common bile duct, such as intrahepatic stricturing or stone, would lead to jaundice and pruritus. Bile contains water, electrolytes, bilirubin, and bile salts. Bile salts derive from cholesterol synthesis and are needed for fat digestion. Bile salts are also conjugated and hydroxylated, separate process from bilirubin. With a mechanical blockage of the bile duct we have conjugated bilirubin backing up into the bloodstream making you jaundice. The bile salts in the bile give you the pruritus. * Autoimmune disease against the microscopic bile ductules is primary biliary cirrhosis (PBC). This is almost always seen in middle-aged women (40s-60s). Patient presents with pruritus as the earliest symptom. ALP is the first blood test to become elevated. Then the patient becomes jaundiced (elevated conjugated bilirubin). Eventually patient will get cirrhosis from this. Autoimmune marker is anti-mitochondrial antibody is suggestive. Ultimate diagnosis rests in liver biopsy, seeing damage to microscopic bile ducts. * PBC treatments do not work very well and patients will get cirrhosis at some point. Management at that point is liver transplantation. * Younger man presenting with pruritus, increased ALP, increased bilirubin. Disease is primary sclerosing

cholangitis (PSC). Do not confuse PSC with PBC. PSC is associated with ulcerative colitis. This is sclerosing of the larger macroscopic bile ducts, so biopsy not need but a picture of the ducts is needed. Test of choice is ERCP with dye injection to visualize biliary tree. Classic is stricturing and narrowing of the biliary tree, showing beads on a string pattern. As this disease progresses the patient will need liver transplantation. Antibody that is positive but not too useful in diagnosis is ANCA. * Hemochromatosis, Wilson disease, and alpha1-antitrypsin deficiency are hereditary liver disease. They all have another organ involvement to suggest them. All can progress to cirrhosis and end stage liver disease. * Hemochromatosis is hyper-absorption of iron in the intestine and deposits of iron in various organs. Organs are liver, pancreas, skin (bronze diabetes), heart (cardiomyopathy). There is likely a family history of liver disease. Screening test is high serum iron to total iron binding capacity ratio (Fe/TIBC) up to 90% even and a high serum ferritin (lots of blood iron). Diagnosis is via liver biopsy and stains for iron on that biopsy (Prussian blue). * Treatment of hemochromatosis is weekly or monthly phlebotomy to make the patient borderline iron deficient thus bone marrow is hungry for iron and pulls it out of the organs. If patient has progressed to cirrhosis and portal hypertension, they will need liver transplantation which is curative. * Wilson disease is a problem with excess copper deposition in the liver due to poor excretion from GI tract in bile. Family history of liver disease. Organs include eye (Kayser-Fleischer ring), CNS (choreoathetosis, dementia), liver disease. Labs show high serum copper, high copper in 24h urine, low levels of ceruloplasmin (copper carrying protein). Diagnosis confirmed by liver biopsy staining for copper (Timm silver stain). * Treatment of Wilson disease is penicillamine, which is copper chelation from bloodstream. Other drugs include zinc and triene, but know penicillamine. Liver transplantation is curative. * Alpha1-antitrypsin deficiency leads to unopposed action of trypsin which is a proteolytic enzyme. Trypsin originates in the liver and is inactivated by alph1-antitrypsin. Organs affected are liver and lung. Suspect in patients who have emphysema and are non-smokers or if patient has relatives with precocious emphysema. Blood test is alpha1-antitrypsin level. Diagnosis is via liver biopsy.

* Treatment of alpha1-antitrypsin is limited, liver transplantation if needed and for cure. Gene for alpha1-antitrypsin is located in liver so new liver corrects the problem. Viral Hepatitis * Hepatitis A can be asymptomatic and IgM (active infection) positive. Typical symptoms of hepatitis are jaundice, fatigue, low grade fevers, anorexia, RUQ tenderness. Hepatitis A is fecal oral, can be transmitted via seafood. Past major cases involved strawberries and green onions. Example scenario would be a nursing home or kindergarten outbreak where kids and teachers get jaundice, malaise, fevers. Test to do is hepatitis A IgM. * Hepatitis A does not become chronic so patients cannot get cirrhotic or get a hepatoma. Past exposure can be seen with IgG antibody. Incubation period is a couple of weeks. Most cases are mild. * Hepatitis A vaccine exists, get before you travel to endemic areas or get because universally recommended. - 65 * Hepatitis B patients are generally symptomatic, fevers, jaundice, fatigue, RUQ pain. Active infection serology is hepatitis B surface antigen positive (HBsAg). Hopefully you become immune if you body creates the appropriate antibodies, hepatitis B surface antibody (HBsAb) and hepatitis B core antibody (HBcAb). If after six month of HBsAg positive without HBsAb, you are labeled as chronic hepatitis B. * There is a hepatitis B vaccine. True hepatitis infection in the past would be HBsAb and HBcAb. Getting the vaccine only would be HBsAb without HBcAb (surface without core). Anyone exposed to blood or sexual contact should be vaccinated so health care workers, prostitutes, IV drug abusers, etc. * Hepatitis B transmitted via blood or sexual contact. Hepatitis B can become chronic and thus develop cirrhosis and hepatoma (hepatocellular carcinoma). * Medical treatment for hepatitis B includes antiviral drugs lamivudine. Also others like adefovir, tenofovir, telbivudine, and entecavir. Interferon can be used as well. * Hepatitis C presents with chronic fatigue and chronic transaminase elevation, it usually does not present acutely. Transaminases increased are SGOT (AST) and SGPT (ALT) from 50 up to 200 max. Test is PCR for hepatitis C where we can measure quantitatively how much hepatitis C there is. * Hepatitis C transmitted via blood usually, pre-1989 via transfusion or needlestick injury or dialysis worker or IV

drug abuser. There is no vaccine for hepatitis C, a major risk for health care workers. Every case of hepatitis C becomes chronic, eventually cirrhosis and hepatoma (hepatocellular carcinoma). * Medical treatment for hepatitis C is pretty good, alpha-interferon with ribavirin. No need to treat everyone, do liver biopsy to assess activity. If minimal activity then might not need treatment. If progressed to cirrhosis then it is too late to treat. Treatment used for moderate hepatitis C prior to cirrhosis progression, give alpha-interferon subcutaneously and ribavirin. * Hepatitis D is a co-infection with hepatitis B, typically seen in patients with a high titer of hepatitis B like patients who continually use IV drugs. Suspect hepatitis D in severe cases of hepatitis B, or chronic hepatitis B that has become newly activated. There is not much we can do about hepatitis D. * Hepatitis E has a high mortality in pregnant women. It is fecal-oral and not common in the United States. It is seen in epidemics. -----------------------------------------------------------------------------------------------------------------------------------------Kaplan Videos (2001) Endocrinology with Dr. Charles Faselis, MD -----------------------------------------------------------------------------------------------------------------------------------------Pituitary Disease: Hyperpituitarism * Pituitary gland is on the sella turcica surrounded by the sphenoid bone and around that area is the optic chiasm. So pituitary disease commonly affects visual field, specifically bitemporal hemianopsia. * Pituitary is divided into posterior and adenohypophysis, secreting hormones (ACTH, TSH, etc) and is affected directly by other hormones in the hypothalamus. Usually adenohypophysis secretes hormones that affect organs in the periphery. Stages of control are hypothalamus, pituitary, and end organs. Posterior pituitary secrets oxytocin and ADH, which are produced by the hypothalamus and stored in the pituitary. * 32yo woman comes to your office because she has noticed a milk-like discharge from her breast for the past four weeks. Examination reveals galactorrhea but is otherwise normal (no visual field cuts or amenorrhea). * With galactorrhea, think prolactinoma because these are the most common active pituitary tumors. They secrete autonomous prolactin. Prolactin is under continuous inhibition from dopamine. When dopamine is inhibited or if autonomous adenoma will you get prolactinemia.

* In women, the prolactinoma is usually a microadenoma. Thus there is minimal pressure phenomenon. They usually presents with amenorrhea and galactorrhea +/- hemianopsia. * In men, the prolactinoma is usually a macroadenoma. They rarely present with gynecomastia or galactorrhea. They usually present with visual field cuts, headaches, and pressure phenomenon. * With galactorrhea presentation, do not jump to adenoma as the cause. Look for other causes like dopamine inhibitors like haloperidol (depletes dopamine) and primary hypothyroidism, which has low T3 and low T4. The low T3 and T4 go back as positive feedback to pituitary to increase TSH and also goes into hypothalamus to increase thyroid releasing hormone (TRH). In a normal individual hormone, TRH activates TSH release and that activates T3/T4 release from thyroid. With primary hypothyroidism, TSH is up and TRH is up. TRH at high doses seems to be an activator of prolactin. Check for TSH; TRH is not tested for because it is expensive and has to be sent out. * 32yo woman comes in with galactorrhea. She is on no dopamine depleting medications and has a normal TSH and T4. What is the next test to order? Answer is prolactin level. * If prolactin level is elevated, do an MRI of the pituitary to look for the lesion. * Treatment of prolactinomas is bromocriptine, which is a dopamine activator and thus a prolactin inhibitor. Decreasing production of prolactin from a prolactinoma would shrink the size of the tumor. - 66 * Treatment of prolactinomas that are macroadenomas (usually men) may need surgery or radiotherapy. * In rare scenarios, prolactinomas occur in older women. In this case, you are not worried about pregnancy so you can watch the patient and not treat if there is no pressure phenomenon. * Acromegaly with pituitary adenoma is caused by a macroadenoma 75% of the time. Acromegaly is due to hypersecretion of growth hormone. Growth hormone mainly works through the liver by secreting insulin-like growth factor (ILGF, somatomedins), which has an effect on tissue and bone causing it to grow. * Acromegaly symptoms are enlarged head, larger feet, shoe size, ring size, tongue grows, jaw grows, and organs grow such as the heart which leads to congestive heart failure and is the most common cause of death.

* Before considering a MRI of the pituitary (80-90% of cases) in acromegaly, you have to document that the patient has autonomous hypersecretion of growth factor, done by a glucose tolerance test. * Glucose causes the release of insulin and counter-regulatory hormones like growth hormone, glucagon, and cortisol. Insulin puts glucose into cells and lowers blood glucose. Counter-regulatory hormones keeps glucose levels high in the blood. If glucose is high, normal response is increased insulin and decreased counter-regulatory hormones. Screening test for acromegaly is to give the patient glucose then measure growth hormone after an hour, normal response is decrease in growth hormone. If after glucose load, growth hormone is elevated, this is an inappropriate response and we say the patient likely has acromegaly due to autonomous hormone release. * 80% of acromegaly patients have elevated glucose and about 30% of them have diabetes mellitus. * We could also measure ILGF levels, which would always be elevated in acromegaly patients. Then do the MRI looking for the tumor in the pituitary. * Treatment for acromegaly is octreotide (somatostatin). Somatostatin is a statin of somatic activity. ILGF are somatomedins, or mediators of somatic activity. Next treatment option is bromocriptine, which shrinks the size of the tumor in acromegaly for unknown reasons. Surgery and radiotherapy are also options. Pituitary Disease: Hypopituitarism * Hypopituitarism is loss of pituitary hormones over time. The first hormones to be lost are the ones you need the least, and those are FSH, LH, and growth hormone. You can live without these. Then TSH is lost, which is a bit more important. Lastly you lose the most important, ACTH which affects the adrenals. * Screening test for hypopituitarism is the insulin-tolerance test. Give patient 5U of insulin to cause hypoglycemia. Appropriate response is no insulin release and elevated counter-regulatory hormones. So you expect high glucagon, high GH, high cortisol. You measure growth hormone an hour later and expect it to be elevated. You do not just measure GH because it changes throughout the day so you have to provoke it. If GH is not high an hour after 5U insulin given, probably hypopituitarism. You can check the cortisol level at the same time to see if ACTH is missing too. You can also check the levels of FSH and LH in the blood.

* Pituitary apoplexy is a medical emergency. Patients usually have a pre-existing pituitary adenoma that was not treated well or not treated at all. The adenoma increases in size and the patient bleeds into the adenoma then virtually blows out their pituitary. History is patient with pituitary adenoma comes in with stupor, nuchal rigidity, vomiting, and headaches. * Treatment of pituitary apoplexy is medical stabilization, high-dose corticosteroids, evaluate electrolytes, administer appropriate hormones, neurosurgical consult for transsphenoidal decompression of the tumor. * Sheehan syndrome usually occurs after a prolonged and labored pregnancy. The first sign of Sheehan syndrome is the inability to lactate. This can occur even 5 years after pregnancy. If it looks like Sheehan syndrome, do the insulin tolerance test to make sure it is not hypopituitarism. * Treatment of Sheehan syndrome is to replace hormones they have lost. * In children, any space occupying lesion can cause hypopituitarism. The most common in children is a craniopharyngioma. In adults, it would be stroke syndromes, cancers, granulomas. * Patients with empty sella syndrome usually have normal endocrine function. In empty sella syndrome, the patient is missing the diaphragmatic surfaces holding the pituitary in. The meninges come into the sella and push out the pituitary. So the pituitary is pushed off to the side and squished between the meninges, thus cannot be easily visualized on CT or MRI. These patients had no endocrine issue but got a CT or MRI for another procedure and were found to have an empty sella. You cannot have empty sella syndrome with normal endocrine function. Pituitary Disease: Posterior Pituitary * Diabetes insipidus (DI) occurs when patients are missing the major water-retaining hormone in their body, antidiuretic hormone (ADH). This is needed to concentrate your urine, else you lose free water. * DI patients complain of polyuria and polydipsia. It may occur because the patient is missing ADH, called central diabetes insipidus (CDI). CDI can occur with any space occupying lesion or anything affecting the central nervous system, namely stroke syndromes, tumors in the brain, granulomas in the brain, and trauma to the head. * Nephrogenic diabetes insipidus (NDI) occurs when ADH is in high levels, but the ADH cannot work because the

- 67 receptors in the kidney do not function. The causes of NDI are lithium and demeclocycline. These drugs can make the ADH receptors in the kidney unresponsive to ADH. * Insipid means devoid of characteristic, so diabetes insipidus has dilute urine. * Test for DI is the water deprivation test, where plasma osmolality is plotted against urine osmolality. In a normal individual will get dehydrated and Posm will increase. At the same time, the patient does not make urine to conserve free water. Normal is increased Posm and increased Uosm (urine concentrates). A patient with DI cannot do this, so as they are getting more dehydrated they will keep putting out dilute urine. Posm will increase, serum Na+ is increasing, but they are putting out dilute urine (Uosm low/not changing). At this point, you cannot differentiate CDI from NDI, so you give the patient vasopressin (ADH). If the patient has no ADH (CDI), you will start to see a normalization of Uosm (Uosm increasing and concentrating). If you give ADH and there is no change in Uosm, this can only be NDI since the kidney receptors are not responding to ADH. * Treatment for CDI is to give vasopressin (ADH) or give intranasal desmopressin. Desmopressin is like ADH but only different by one amino acid, however you do not have the pressor effects of ADH. * Treatment for NDI is hydrochlorothiazide or amiloride. The diuretics get rid of sodium and help retain water. This sounds like a paradox because the patient is getting rid of lots of urine, but the aim is to retain water. The mechanism for thiazide diuretics is to decrease distal tubular sodium reabsorption, which increases urinary excretion, which decreases extracellular volume, which increases proximal sodium and water reabsorption, which decreases distal delivery of sodium and water, and finally decreases urine output. * Patients presenting with polyuria and polydipsia either have diabetes mellitus (check serum and urine glucose) then think about DI, but also think about psychogenic polydipsia. * Psychogenic polydipsia is characterized by large intakes of water and large outputs of urine. These patients do not wake up in the middle of the night to go to the bathroom. This disease runs in families. Water deprivation test will show almost normal response. When these patients drink lots of water the wash out the medulla of the kidney and lose concentrating capabilities.

* Patients with DI usually have access to water, so they can take in 20L and put out 20L. It is when they lose access to water that they get very dehydrated. Common scenario is a patient on lithium for many years who goes into the hospital for surgery (or another scenario where water access is lost) and now they become very dehydrated showing with severe hypernatremia. * Syndrome of inappropriate ADH (SIADH) secretion is when patients produce small amounts of concentrated urine. These patients hold on to too much free water, which dilutes the sodium and causes hyponatremia. * SIADH can occur with CNS processes like tumors, strokes, granulomas, infections, trauma. * SIADH can occur with lung processes like abscess, COPD, tuberculosis. * SIADH can occur with medications like vinblastine and vincristine. * Treatment of SIADH is to treat the primary disease, then SIADH usually resolves. Sometimes the sodium is so low we have to treat the SIADH. Treatment of choice is water restriction. Treatment could also include drugs that cause NDI like lithium and demeclocycline, demeclocycline is probably preferred due to less side effects and less drug-drug interactions. * In severe hyponatremia, hypertonic saline can be used. Do not replace too quickly due to risk of central pontine myelinolysis. Correcting hypernatremia too quickly with free water can result in cerebral edema and seizures. Thyroid Function Testing * Thyroid gland produces T4 as the main secretory product. * Do not assume enlargement or nodules of the thyroid mean hyperthyroidism. This is why we need to objectively assess the thyroid by looking at thyroid hormones, T4, T3, and TSH (indirect measurement). * The best test for screening for thyroid disease is thyroid stimulating hormone (TSH) as it is the most sensitive and specific test. The current TSH testing can diagnose hypothyroidism or hyperthyroidism with a 95% confidence interval, so they are very good tests. * If the TSH is normal, in general that patient is euthyroid. Forget about the T4 and forget about the T3. * Thyroid levels of control are hypothalamic function (TRH), pituitary (TSH), and T4/T3 from thyroid. The thyroid produces mostly T4, T4 in the periphery gets converted to T3. * T3 is a more potent hormone with a better effect. Because T3 is more potent, it has a stronger feedback. To

suppress TRH and TSH, you would give T3 hormone. * When your thyroid puts out T3 + T4, nearly all of it is bound to albumin. This is called thyroid binding globulin and it is an inactive form, so does nothing to TSH or TRH for feedback nor does it have metabolic effects. The free T4 is less than 0.1% of the total, it is metabolically active, and has the feedback effect to TSH and TRH. To see how the thyroid is really working, you want free T3 and free T4, not the total because you only care about free T4. * Free T4 use to be a difficult test, so people measured total T4 then got RT3U (resin T3 uptake) to indirectly - 68 measure free T4 index. Resin T3 uptake does not measure T3, it is an indirect measure of T4. Again, this test is not done anymore so look for a free T4 test. * Since 99.9% of thyroid hormones are bound to albumin, any changes in albumin and protein levels in the body will fluctuate the total T3 and total T4 levels. Example would be pregnant patient or patient taking estrogen, they have increased proteins so total T4 will increase. Is this patient hyperthyroid? No. Free T4 in this patient would be normal and TSH would be normal. So be careful when you see a total T4 or total T3 instead of free T4 or free T3. Example patient with nephrotic syndrome or cirrhosis, total protein level is down. Total T3/T4 would be down in this patient but it is not hypothyroid. Free T4 and TSH would be normal in this patient. * With fluctuations in protein the body will keep the free T3 and free T4 stable. * Radioactive iodine uptake scan (RAI-uptake) is a way to directly measure function o the thyroid. Radioactive iodine is given to the patient, wait 24h, then scan the thyroid, repeat in 48h to see what the uptake is. Normal is 1040% uptake just as a note. Anything under this would be hypothyroidism, anything over this is hyperthyroidism. * Primary hyperthyroidism: TSH low, free T4 elevated, free T3 elevated, RAI-uptake scan high. Example is Graves disease, toxic nodular goiter. * Thyroiditis: TSH low, free T4 elevated, free T3 elevated, RAI-uptake scan is low. Example is subacute thyroiditis. Thyroid hormone is being spilled into the periphery due to destroyed thyroid cells but uptake is low because thyroid is injured. This patient should not be treated with antithyroid medication, treatment is to wait.

* Secondary hypothyroidism: TSH low, free T4 low, free T4 low, RAI-uptake scan is low. Example is hypopituitarism. Likely to have abnormal growth hormone, LH, and FSH because usually not just one pituitary hormone missing. * Pregnancy or protein-increasing medication: TSH normal, free T4 normal, free T3 normal, RAI-uptake scan normal, elevated total T4. * Exogenous thyroid medication use (e.g. weight loss attempt): TSH low, free T4 elevated, free T3 elevated, RAIuptake scan low or normal. Thyroid hormones use to be in weight reduction medication a few decades ago even in this country. * Anti-microsomal antibodies sometimes associated with Hashimoto disease. * Thyroid globulin immunoglobulin sometimes associated with Graves disease. * A patient is intubated in the ICU and the patients FT4 was high or low. Someone thought this patient might have a thyroid problem, but TSH came back normal. Since this is an ICU patient, they likely have a lot of other comorbid conditions. This patient has sick euthyroid syndrome. There is no need for endocrine management here. Hyperthyroidism * Graves disease is the most common cause of hyperthyroidism. The cause is an immune process, antibodies that affect the TSH thyroid receptors, antibodies act like TSH to rev-up the thyroid. The thyroid enlargement is diffuse and is the only time in thyroid disease where you get proptosis and exophthalmos. * Other Graves signs include dermopathy, palpitations, tachycardia, intolerance to heat, weight loss, diarrhea, menstrual irregularities, tremor, bruit over thyroid, sometimes swelling of the feet (pretibial myxedema). * Graves laboratory findings would be low TSH, increased FT4, and elevated RAIuptake if you felt you needed it. * Management in the acute phase is to relieve symptoms and reduce peripheral thyroid hormone. Anti-thyroid medications are propylthiouracil (PTU) and methimazole to decrease thyroid hormone level. Symptomatic relief includes beta-blocker therapy (propranolol, atenolol) until patient is out of acute phase meaning they feel better and become euthyroid. * Management after acute phase is definitive treatment with radioactive iodine to kill the thyroid. Within months or years the patient will become hypothyroid, so you have to follow this patient every six months to check thyroid

hormones. When they become hypothyroid, then give thyroid medication (synthroid or any other T4 form). * Complication of PTU and methimazole is agranulocytosis. If patient is on this medication and calls you saying they feel horrible and are running a high fever. Tell them to stop the medication and have their white cells checked. It is reversible agranulocytosis as long as you catch it. * If agranulocytosis occurs with PTU then you cannot give the patient methimazole, they will get agranulocytosis with that as well. * Proptosis and exophthalmos commonly gets worse when you treat Graves disease. Ophthalmopathy is treated separately as patient may not be able to close their eyes and may get eye ulcers. So they may need surgery, or orbit radiation, or steroids. * In pregnant patient with Graves disease the only anti-thyroid medication we can use is PTU. We can also use propranolol. Both of these cross the placenta and have side effects. Propranolol can cause small for dates. So we can only use these at low doses. Give them a little bit of PTU and propranolol to cool down the thyroid, then take the patient to surgery in the second trimester. Radioactive iodine has not been shown to cause cancer, but you definitely - 69 do not want to give it to a pregnant patient. * Toxic nodular goiter is more common in adults and older patients. These patients usually had a non-toxic goiter prior. Toxic goiter means the thyroid goiter produces thyroid hormone. Non-toxic goiter means nothing is being produced from the goiter. * Toxic nodular goiter exam will not show a diffuse enlargement. You will find nodules, maybe one, maybe many. * Presentation occurs mostly in elderly patients and they have mainly cardiac manifestation such as atrial fibrillation, tachycardia, congestive heart failure. * Both hyperthyroidism and hypothyroidism cause proximal muscle weakness. Thyroid Storm * Thyroid storm definition is a hyperthyroid patient with delirium, fever, hypotension, CHF. This is high output cardiac failure, treated with propranolol. * Thyroid storm usually in patients with Graves disease and precipitated by a condition such as infection or surgery.

* Treatment is propranolol even if they have hypotension, PTU, and iodide. Iodide is the cornerstone of thyroid storm treatment. Thyroid will either output T3/T4 or grab iodide but not generally both. It would rather grab onto iodide, so giving the patient iodide will down-regulate the production of T3/T4. * Iodide buys you some time. If you do not manage the thyroid storm patient, a couple days later they will have more iodide so they can produce much more T3/T4, starting the process again. * Treatment for thyroid storm also includes fluids, cooling blankets, and steroids (hydrocortisone). In all severe thyroid problems there is dysregulation of adrenal steroid production. Hypothyroidism * Most common form of hypothyroidism is Hashimoto thyroiditis. This is an immune process where there is production of antibodies and lymphocytes that cause chronic inflammation of the thyroid. The thyroid eventually dies out and the patient becomes hypothyroid. * Iatrogenic hypothyroidism is also common in this country because of how we treat Graves disease. * Symptoms of hypothyroidism include weight gain, cold intolerance, constipation, cholesterol increases due to lack of metabolic hormones, a type of dementia can form, dry skin, course hair, periorbital edema. * Elderly patient develops dementia, has constipation, and all of a sudden developed high cholesterol. Exam shows decreased deep tendon reflexes specifically the relaxation phase. Patient is a little puffy and edematous. * Lab profile is high TSH, low FT4, low FT3. * Treatment is thyroid hormone replacement usually in the form of T4. Since these are metabolic hormones, you want to give this slowly in the elderly patient who may have coronary heart disease. * A normal adult mainstay does is 100mcg/day. For elderly patient, start at 25mcg/day then go to 50mcg/day then consider going up if needed. Elderly patient may develop chest pain and may even get a myocardial infarction. * Myxedema coma is a severe form of hypothyroidism. These patients have altered mental status, are hypothermic and are sometimes found out in the cold (cold or infection precipitated the untreated hypothyroidism). * Treatment of myxedema coma is high dose of thyroid hormone, like T4 at 300mcg. This is a very dangerous dose but it is the only way to treat these patients. We also give the patient a more potent hormone, T3. Also give fluids,

warming blankets, and steroids because of hydrocortisone dysregulation. Remember high doses of T4 given. Thyroiditis * Thyroiditis is an inflammatory condition of the thyroid gland and is relatively common. Inflammatory process can be acute where there is intense inflammation of the thyroid. This causes spilling of T4 and T3, so these patients usually present with hyperthyroid symptoms. After a week or month, these patients will become euthyroid. * This is subacute thyroiditis, also called De Quervain thyroiditis or giant cell thyroiditis. Subacute thyroiditis is most likely due to a viral infection. Palpation of the thyroid results in pain. * Labs for subacute thyroiditis show low TSH with high FT4/FT3, which looks like Graves disease. Order the RAIuptake test, which will be low. * Because subacute thyroiditis is an inflammatory process, acute inflammatory markers will be elevated. ESR will be elevated always. * Treatment is pain medication such as NSAIDs, aspirin. * Post-partum thyroiditis is subacute thyroiditis after delivery, same process, transient, conservative management. * Chronic low-grade inflammation of the thyroid will eventually lead to fibrosis and destruction. These patients will usually present with hypothyroid symptoms as the thyroid has been destroyed. * This is Hashimoto thyroiditis. Biopsy here will show lymphocytic infiltrate, fibrosis and scarring of the thyroid. * Hashimoto associated with other autoimmune diseases such as pernicious anemia. * Can have antimicrosomal antibodies. * Treatment is the same as all hypothyroidism, replace thyroid hormone. - 70 Thyroid Cancer * Papillary cancer of the thyroid is the most common, about 70-80% of all primary thyroid cancer cases. * Papillary cancer is benign, usually grows focally and metastases to the lymph nodes around the thyroid, is slow growing, associated with radiation to the head and neck area. * Treatment of papillary cancer allows for 95% of patient to live 5 years, so great prognosis. 10yr survival 80-90%. * Treatment of papillary cancer is surgery then give patient T3 or high doses of T4 because cancers of the thyroid are probably sensitive to TSH so you want to suppress TSH production as much as possible. * Iodine scan is a way to monitor these cancers, they can look for metastatic disease.

* Follicular cancer of the thyroid occurs in older individuals and usually has hematogenous spread. * Patient presents with some kind of thyroid cancer and metastatic disease to distal visceral organs. Think that this is likely follicular cancer. * Treatment of follicular cancer is the same as papillary cancer, surgery then T3 or high dose T4. * Medullary cancer of the thyroid arises from the parafollicular cells (C cells, calcitonin producing cells). It is associated with some familial forms, one form being solitary medullary cancer. * You notice in the family that patients have had solitary medullary cancer (not multiple endocrine neoplasia). * Medullary cancer also associated with MEN-II (MEN-IIa), which also has pheochromocytoma and parathyroid hyperplasia. Also associated with MEN-III (MEN-IIb), which also has pheochromocytoma and neuromas. * Anaplastic cancer of the thyroid is deadly, occurs in older individuals, does not metastasize but it grows focally, no treatment available. Less than 5% of patient live more than a couple of years. * Patient usually comes to the office complaining of a thyroid lump or you find one on routine exam. What do you do with the thyroid nodule? Every time you have a thyroid nodule the first test and only test to do is a fine needle aspiration (FNA). * Risk factor for thyroid cancer if you find a lump is radiotherapy to the head and neck (presumed to have cancer until proven otherwise). They may just say the patient had Hodgkin disease stage IA several years ago, so you would need to know that the treatment for that is radiation therapy (XRT) only to the head and neck. * Other risk factors for thyroid cancer are male gender, older age, associated lymphadenopathy, recent growth of nodule, thyroid scan showing cold nodule. * Always do a fine needle aspiration on a thyroid nodule. But, the nodule has to be non-active meaning no hyperthyroidism. * Protocol for thyroid nodule found is to get TSH to see if functioning. If TSH low the patient has some form of hyperthyroidism so you treat as hyperthyroidism (toxic nodular goiter). If TSH normal, then you know the patient has a non-functioning thyroid nodule and thus a chance for cancer. Next step in management is fine needle aspiration, do not order a scan of any kind or an ultrasound. If thyroid cancer found on pathology of FNA, then do

surgery. If FNA is benign, follow patient but no management needed now. * Exception is follicular cells found on FNA. The difference between a benign follicular adenoma and follicular cancer is the architecture. So you cannot tell from only an FNA. If follicular elements are found on FNA, then do thyroid surgery because that is the only way you can tell if this is a cancer or just a benign tumor. * If a scan is done and a warm or hot nodule is found, the next step is not waiting. You have to do the FNA. Parathyroid Disease * As a patients calcium goes down, parathyroid hormone (PTH) increases. This prevents hypocalcemia to the point of tetany and death. PTH stimulates osteoclasts to break down bone and release calcium into serum. * PTH also affects the kidney to reabsorb calcium and the intestine to absorb more calcium through vitamin D. * When PTH activates the kidney it wastes phosphate (PO4-). Mnemonic: PTH, phosphate trashing hormone. * With high levels of PTH secretion, the calcium may be high or normal but the phosphate will always be low. This is because you do not know where the patient started with calcium. Say they have primary hyperparathyroidism with an autonomous adenoma secreting PTH but what if the patient has vitamin D deficiency or renal failure? So calcium in this scenario would be about normal or low if you do not have enough bone mass. Phosphate will always be low. * When talking about PTH affect, look at PO4-, Ca++ is not a sensitive index of parathyroid hormone. * Vitamin D increases calcium and increases phosphate. * Calcitonin tones down calcium, decreases calcium by reducing bone resorption. We can use salmon calcitonin to decrease calcium levels in hypercalcemia. Hyperparathyroidism * Primary hyperparathyroidism is the most common cause of hypercalcemia and is the most common cause of benign or mild hypercalcemia. * Hypercalcemia found on routine screening 50% of the time it is primary hyperparathyroidism. * There is an autonomous adenoma that is producing PTH, activating osteoclasts, releasing calcium. 80% of cases - 71 there is a single adenoma. In 15-20% of cases there is four gland hyperplasia or parathyroid cancer.

* Four gland hyperplasia requires removing all four glands, putting one in the arm, then watching the serum calcium go up and down. * Fibrosa cystica (brown tumors) develops in the bone due to the constant osteoclast activation resulting in cyst formation. These are breakdown of bone. * Primary hyperparathyroidism associated with MEN-I (the 3 Ps): pancreatic tumors, pituitary tumors, and parathyroid tumors. Patient has hypercalcemia with severe peptic ulcer disease and diarrhea, this may be due to a gastrin producing pancreatic tumor (Zollinger-Ellison) with parathyroid tumor. Lab would be low phosphate. * Treatment is surgery for definitive treatment, it should be done in all patients with symptomatic hyperparathyroidism such as kidney stones, osteitis fibrosa cystica. * Older individual with mild asymptomatic hyperparathyroidism may be watched. * Mild hypercalcemia would be bone pain, abdominal pain, polyuria, polydipsia. Severe hypercalcemia occurs with > 15mg/dL and is a medical emergency. Treat hypercalcemia with fluids (normal saline) primarily. Fluids is the cornerstone of treatment for hypercalcemia. If calcium is still high, then calcitonin (immediate action, wears off after 48hrs) or bisphosphonates (long acting) like pamidronate. * Hypercalcemia associated with cancer is usually severe, so that is a case when you could give calcitonin at the same time as a bisphosphonate. * Furosemide and diuretics can be seen in hypercalcemia protocols because furosemide gets rid of calcium. On the other hand, hydrochlorothiazide retains calcium. Furosemide does not do much to the calcium level; it is given because the mainstay of treatment is fluids and we do not want to cause fluid overload in an elderly patient. Do not write for IVF and furosemide at the same time here, only give furosemide if evidence of overload. * With one gland hyperplasia, the other three glands shrink due to negative feedback with calcium. After surgery, you have three glands that are not working well. So patient goes from hypercalcemia to hypocalcemia, this is called hungry bone syndrome. Hypocalcemia after surgery for primary hyperparathyroidism is due to calcium going to bone and nothing left to stabilize the calcium. Watch these patients closely in a telemetry unit and monitor calcium levels for a couple of days. Treatment here is IV calcium with ECG monitoring for QT abnormalities.

* Hypocalcemia prolongs the QT interval. Hypercalcemia causes shortened QT. Hypercalcemia of Malignancy * Hypercalcemia of malignancy can occur through multiple metastases to bones, such as multiple myeloma. The other mechanism is through production of PTH-like molecule, which activates the osteoclast. Example is squamous cell lung cancer or esophageal cancer. Squamous cell cancer does not metastasize much, it grows focally. * Most blood assays only check for parathyroid hormone that is intrinsic. So PTHlike substance may not be picked up by parathyroid hormone levels. * To distinguish in a hypercalcemia patients between primary hyperparathyroidism and hypercalcemia of malignancy, you order PTH level. PTH level would be elevated only in primary hyperparathyroidism. * Familial hypocalciuric hypercalcemia is a benign disorder. These patients hardly ever have symptoms. Treatment is usually not necessary, giving diuretics if calcium is high or if kidney stones occur uncommon. * Hypercalcemia of immobilization is also benign. This would be a nursing home patient who is in bed most of the time. They will have an elevated calcium level and it is not likely cancer or hyperparathyroidism. Treatment is just to monitor the patient, no need for medications or further testing. * Sarcoidosis can cause hypercalcemia. It is a chronic granulomatous disease, where granulomas occur mainly in the lung but can occur anywhere. Diffuse granulomas seem to activate vitamin D, so hypervitaminosis D. This is the cause of the hypercalcemia (and more phosphorus too). * Most of vitamin D is absorbed in the summertime due to sunlight activation. So sarcoid patients develop hypervitaminosis D in the summertime or when they go to sunny areas. Hypocalcemia * Whenever you see hypocalcemia, make sure to account for low albumin states. When we measure calcium in serum we get total calcium and 40% of this is ionized, so majority is protein bound. Ionized calcium is the part that affects neuromuscular irritability. * A patient with nephrotic syndrome would have a lower total calcium, but their ionized calcium would remain the same. For every drop in albumin by a single point there is a drop in calcium of 0.8. * Normal albumin is 4mg/dL. Say nephrotic patient is 2mg/dL albumin and total calcium level is 6.5. What is this

patients real total calcium? Answer is (4mg/dL - 2mg/dL) = 2 * 0.8 factor = +1.6. So 6.5 + 1.6 = 8.1 total calcium. * With hypocalcemia, think about hypoparathyroidism. A patient who got exploratory surgery around the neck area could have had damage to parathyroid gland circulation. With vitamin D deficiency and in some stages of chronic renal failure the parathyroid hormone becomes ineffective. - 72 * Any drop in magnesium levels affects parathyroid hormone. Magnesium is needed to secrete PTH and is needed for PTH to act on the osteoclast and the kidney. * Symptoms of latent tetany include Chovstek sign where you tap the facial nerve and see an ipsilateral facial twitch, or the Trousseau sign where you put a cuff on the patients arm then inflate the cuff and leave it there for a minute or so until you see tetany (can be rather dramatic). Patient could get laryngospasm. * Renal failure or hypoparathyroidism would cause low calcium and high phosphate. * Vitamin D deficiency (secondary hyperparathyroidism) would cause low calcium and low phosphate. * Treatment of hypocalcemia is oral calcium +/- vitamin D if no symptoms and IV calcium if symptomatic. * Secondary hyperparathyroidism is caused by hypocalcemia, so PTH increase and PO4- decreases. Calcium may normalize in the beginning. It is seen in vitamin D deficiency, Rickets, renal insufficiency. * With secondary hyperparathyroidism, the parathyroid glands get activated to constantly dump PTH. After many years, they become autonomous. This leads to tertiary hyperparathyroidism. * Renal failure patient can get secondary hyperparathyroidism due to lack of calcium. After time, the parathyroid glands start acting autonomously which leads to tertiary hyperparathyroidism. Now you cannot suppress the calcium and you have to remove the parathyroid glands, parathyroidectomy. Diabetes Mellitus * Diabetes is a disease of carbohydrate metabolism. Type I diabetes is usually seen in young individuals, patient is usually slender and no family history of diabetes. This results from an inflammatory process of the pancreas beta cells causing them to get destroyed and leading to insulin deficiency. * Type I diabetics are insulin dependent. Ketosis can occur because there is no insulin, so these patients can break

down lipids as their main form of energy which produces ketone bodies and diabetic ketoacidosis. * Identical twins with Type I diabetes has only about a 50% concordance rate. Some people mistakenly think Type I DM is congenital because patients present as children, but remember there is not much family link here. * Type II diabetics are overweight, usually 30-40+ years of age, it runs in the family unlike Type I. * Identical twins with Type II diabetes has a 100% concordance rate. * Type II diabetics are insulin resistant, but non-insulin dependent diabetes mellitus (NIDDM). Resistance to insulin decreases with weight loss. * Type II diabetics can develop hyperosmolar hyperglycemia nonketonic syndrome (HHNS). These patients may have glucose levels in the 1000s, but no acidosis; They do have severe dehydration. * We can only reverse the microvascular complications of diabetes with tight glycemic control, not the macrovascular complications. The types of microvascular complications are the eye (retinopathy), brain (neuropathy), kidney (nephropathy). You cannot reverse the peripheral vascular disease or cardiovascular complications. * A Type I diabetic who presents at your office has not had the disease for very long. You cannot live without insulin for a long time. Maybe a couple of weeks. Presentation is usually diabetic ketoacidosis. These patients do hot have any of the microvascular complications; they develop these at least 5 years later. * A Type II diabetic presenting at your office will have complications already because they have had the disease for at least 5 years or more. It is difficult to study type II DM because patients have had the disease for a long time but do not know it. This patient would need to go to the ophthalmologist right away to look for retinopathy, you would need to look that day for proteinuria, and decide if they need an ACE-I. * Hyperosmolar hyperglycemia nonketonic syndrome (HHNS) is a state of high sugars, no acidosis, severe dehydration, usually older individuals. This has a worse prognosis than diabetic ketoacidosis (DKA) because it occurs in older individuals with comorbid conditions. HHNS can progress to hyperosmolar coma. * DKA usually occurs in young patients with no other comorbid diseases. * If patient presents in HHNS or DKA, you diagnose them with diabetes. Most patients present complaining of polydipsia and polyuria. You check their glucose and find it to be elevated.

* Definition of diabetes is fasting glucose > 126 for two consecutive visits. You do not have to treat the patient right away, but they do have diabetes. * Initial management is diet modification. If that does not work you do oral hypoglycemics (e.g. metformin). If that does not work you use insulin, but thats probably years later. * Diabetic retinopathy is the most common cause of blindness in middle aged people. Background retinopathy is associated with retinal hemorrhages and cotton wool spots on fundoscopic exam. It usually does not cause blindness. Proliferative retinopathy is neovascularization, the new vessels are prone to bursting and bleeding which leads to blindness. These patients need laser treatment to the eyes or they will go blind. For background retinopathy, you just have to control the patients sugar. * Studies show that even excellent primary care physicians miss proliferative retinopathy. So patient with DM must - 73 go to the ophthalmologist at least once a year. * Mononeuropathy in DM would be a patient with foot drop, wrist drop, or cranial nerve abnormality. Central cranial nerve that is most affected in diabetes is CN III (ptosis). The longer nerve CN VI gets affected in many lesions but in diabetes for reasons unknown CN III is the most commonly affected. * Peripheral neuropathy in DM is the stocking and glove distribution (loss of sensation). * Autonomic neuropathy in DM is associated with gastroparesis, patient feels bloated, has diarrhea, feels full after eating very little. Autonomic neuropathy also causes erectile dysfunction and hypotension. * Nephropathy in DM causes nephrotic range proteinuria > 3g/day but not nephrotic syndrome. If you put the patient on ACE-I, you will decrease the amount of protein in the kidney. So the treatment of proteinuria or microproteinuria in a DM patient is ACE-inhibitor (e.g. lisinopril). Any patient with hypertension and diabetes, first line drug is ACE-I. * To stabilize any of these complications (ophthalmopathy, neuropathy, nephropathy) do tight glycemic control. Meaning get to normal blood sugar which is < 125. * Patients may have complications but show up at your office with normal sugars. To measure glucose control over

time, look at the glycosylated hemoglobin (HgA1c). This looks at sugar over 2-4 months. * A normal blood sugar (< 125) over time would correlated to an HgA1c of < 7 or even < 6.5. * Do not diagnose diabetes with an HgA1c level, it is just a monitor of disease activity. * HgA1c of 7.5 correlates to sugars between 150 and 175. * HgA1c of 8+ correlates to sugars running above 200. It does not matter what the patient tells you about their control or their blood sugar at the office visit. The bottom line is an HgA1c over 8 means they have been poorly controlled for the past few months. * Treatment of Type I DM is insulin. * To cover a patient throughout the day, give some rapid insulin and some intermediate acting insulin. So patient may get regular insulin then NPH insulin in the morning, followed by regular insulin and NPH at night. * Determination of insulin units is based on what they used in the past 24 hours. Break that down into taking most of the insulin in the morning and about 1/3 at night. If patient used 100U over the last 24h prior to hospital discharge, you give 66 units at the next AM and 33 units at next PM. In the AM, use 2/3 NPH and 1/3 of regular (so of the 66, 44 NPH and 22 regular insulin). In the PM, use 2/3 NPH and 1/3 of regular or maybe 1/2 of each. Yes, this is random. Called law of 2/3rds. Just understand that a type I diabetic will need coverage throughout the day with short-acting insulin and intermediate-acting insulin. * Regular has a peak action of 2-4 hours, so that will kick in around lunch time when given in the AM. NPH onset is around 6-8 hours so it will kick in later in the afternoon. Say patient comes in saying their noon blood sugar check is around 300. Which insulin will you increase to cover this (NPH, regular, AM v PM)? Increase AM regular insulin. Say patient has blood sugar just before dinner is around 300. Now what do you increase? Increase AM NPH insulin. Say glucose before bedtime is in the 300s. Now you increase the PM regular insulin. * Type II DM can get a little NPH once a day or so. Type I DM need insulin twice a day. * Exception is patient with high sugar in the morning. This can be true high sugar because insulin you gave the day before has worn off, this is called the dawn effect. To fix this, give the patient AM regular and NPH. Then give them regular before dinner and NPH before bedtime to carry over until the morning. The problem is the second scenario

with morning hyperglycemia, the somogyi effect or somogyi phenomenon. Somogyi effect is due to rebound from too much insulin use. During the night you see this example, midnight sugar of 120, 3am sugar of 70 (hypoglycemia), 5am sugar is 90, 8am sugar is 300. Patient will say they are not sleeping well or they are having bad nightmares, but they do not wake up from the hypoglycemia. What happens is the insulin wears off at night leading to hypoglycemia, then the counter-regulatory hormones kick in and by the time the patient wakes up they are hyperglycemic. If you increased the insulin in this patient at night, you could put them in a coma. * With morning hypoglycemia, be worried about somogyi effect and do not increase the night insulin. First, decrease the amount of insulin at night until you can figure out if this is the dawn effect or this is the somogyi effect. When you decrease the night insulin, the patient with somogyi effect will have normal sugars in the AM but the patient with dawn effect will have even higher sugars. Another option is to have the patient wake up at 3am a couple of nights to check their blood sugars. * Patient with diabetes usually presents because diabetes is precipitated by something, like stress or a cardiac event. Type II diabetic patient may have infection or cardiac event. Type I diabetic patient usually have an infection. * Type I diabetes occurs due to an inflammatory process. So this is assumed to be from a viral infection (e.g. coxsackievirus) in predisposed individuals. * Type I DM, there is a decrease in functional pancreatic mass after insulitis event. The mass decreases until a diabetic threshold is reached, then they develop clinical diabetes. If patient gets an infection, their insulin needs get - 74 higher and their pancreatic mass is stressed. They come with clinical evidence of diabetes. Treatment of the infection (e.g. pneumonia, UTI) and diabetes is started, then they call you two weeks later because their sugar level is normal. That is because they have residual pancreatic mass and non-clinical diabetes. This is called the honeymoon period. * Treatment during the honeymoon period is to give the patient low dose insulin. By treating this patient you are protecting the residual functional pancreatic mass and not stressing it.

* Type II DM treatment starts with diet, then oral hypoglycemics, then insulin likely needed 10 years later. Oral Hypoglycemic Agents * Sulfonylurea have been around for many years. They act on the beta cells to produce insulin. They also sensitize the peripheral cells to the effects of insulin. * Sulfonylureas include glyburide, tolbutamide, glipizide, chlorpropamide. * Glipizide and glyburide are given once a day or maybe twice a day. All the sulfonylurea drugs are metabolized by the kidney except tolbutamide. So in a patient with renal failure (common in DM) choose tolbutamide. Tolbutamide is not used often because dosing is three times a day. * Most sulfonylureas have a prolonged effect, so in an overdose you should anticipate hypoglycemia for 24 hours and admit to the hospital. If patient takes too much regular insulin, you can watch and give sugar then discharge. * Metformin is a biguanides and does not affect the beta cells. Metformin acts on the liver to stop neoglycogenesis, so it stops the liver from making sugar. It also blocks, a little, glucose uptake from the gut. It also facilitates the movement of glucose into muscle. Major action is stopping glucogenesis in the liver. * Metformin is a good drug, patients have less complications and do better on this medication. Complications of metformin include GI upset and diarrhea (most common), lactic acidosis (reason to discontinue medication) which occurs more commonly in patients with renal insufficiency * When sending a diabetic patient for a radiologic study that needs IV contrast dye, stop the metformin at least 24h ahead of time. If you cause a little renal failure with the dye you could increase the lactic acidosis side-effect seen with metformin. * Acarbose and miglitol are alpha-glucosidase inhibitors that are rarely used because of the severity of their sideeffects (flatulence and bloating). * Rosiglitazone, pioglitazone, and troglitazone are thiazolidinediones (TZDs) which improve glucose use by cells. They are not used due to their side-effect profiles. * Type 2 DM (obese): metformin first, then +/- sulfonylurea, then +/- exenatide or insulin or glitazone. * Type 2 DM (non-obese): sulfonylurea or metformin first, then +/- metformin or sulfonylurea, then +/- exenatide or insulin or glitazone. * Type 2 DM (elderly): low dose secretagogue, then switch to simple insulin regimen.

* Type 2 DM (Asians): glitazone, then metformin, then sulfonylurea or insulin or exenatide. Hypoglycemia * Hypoglycemia is not just low sugar in the blood, there must also be symptoms. * Whipple triad: low sugar, symptoms consistent with hypoglycemia, improvement after glucose given. * Normal response of the body to hypoglycemia is to drop insulin down and increase counter-regulatory hormones like cortisol, growth hormone, and glucagon. * Hypoglycemia with DM cause occur due to exercise because there is enhanced glucose uptake by muscles without insulin. Patient is taking regular + NPH in the morning and evening. They start working out in the evening before dinner, but notice they feel dizzy and their sugars run in the 50s. Tell the patient to keep exercising and then decrease the morning NPH insulin. * DM patient develops new onset renal failure, not insulin has a longer half-life and patient develops hypoglycemia. So check creatinine to rule out renal insufficiency if DM patient develops hypoglycemia. * Hypoglycemia in a non-diabetic is initially managed by looking at the body response. If you measure insulin and it is zero, this is an appropriate response to not eating. Example would be rounding in the AM post-call and someone starts to get wobbly. Treatment is to eat, frequent meals. If you measure insulin during hypoglycemia and it is normal or high, this is an inappropriate response. * Hypoglycemia with high insulin is due to a rare pancreatic insulinoma, which is an autonomous insulin-secreting tumor, or more commonly the patient is taking exogenous insulin. The usual profile of the exogenous insulin taker is someone in the medical profession or lives with someone who is diabetic. * Endogenous insulin is secreted with C-peptide at an almost 1:1 ratio. If the patient has an insulinoma, they will have high insulin and high C-peptide because the only way you can secrete endogenous insulin is to co-secrete with C-peptide. Patient taking exogenous insulin will have high insulin level but low Cpeptide. Another complication that can occur with exogenous insulin takers is development of insulin antibodies. - 75 * It is important to differentiate exogenous insulin taking from insulinoma. An insulinoma may be very difficult to

locate, to the point where surgery could involve cutting off pieces of the pancreas until the sugar normalizes. It may not be a tumor, it could be dispersed cells in the pancreas. * What if a patient took exogenous sulfonylurea medications? These act on the beta cells to increase insulin and make sugar plummet. C-peptide would be elevated as well, looking just like an insulinoma. If there is clinical suspicious for exogenous sulfonylurea use (patient in medical profession or diabetics in family) then check the urine for sulfonylurea levels. Cushing Syndrome * Excess exogenous cortisol does a lot of things but it depends on how long you are taking the cortisol for. Yes it can cause a buffalo hump, purple striae, diabetes because it is a counter-regulatory hormone, hypertension due to mineralocorticoid effect. * Likely the most common cause of Cushing syndrome is iatrogenic. Example would be steroid-dependent asthma or collagen vascular disease. Next most common is autonomous ACTH adenoma in the pituitary. * Autonomous adenoma in the pituitary secreting ACTH can cause Cushing syndrome. It is also called Cushing disease because it was in the original patient described (moon facies, buffalo hump, purple striae). This is a benign disease in that it occurs over a long time (e.g. 10 years) before it is diagnosed. * Small cell (oat cell) cancer in the lung can cause ectopic ACTH secretion leading to Cushing syndrome. This is a malignant disease, due to very high levels of ACTH. This patient does not have enough time to develop the typical Cushing appearance. These patients look thin, wasted, cachectic because of the lung cancer. Because of the huge ACTH secretion, they will be very hyperpigmented. ACTH is similar to melanocyte stimulating hormone, in fact they are co-secreted. This autonomous secretion cannot be suppressed no matter what you do. * So if you see a thin person with hyperpigmentation, do not assume Addison disease. Check that they do not have a form of Cushing syndrome. * Adrenal adenoma or adrenal hyperplasia or adrenal cancer will cause a secretion of cortisol. The ACTH is suppressed, thus these patients do not have hyperpigmentation. This form is autonomous so does not get suppressed. * Diagnosis of Cushing syndrome should not involve scanning the patient. You will likely find nodules in the lung

that are not cancerous, nodules in the pituitary that mean nothing, nodules in the adrenals that mean nothing. * First question is to ask if the patient has Cushing syndrome. After diagnosis, then localize. First screening test is the overnight dexamethasone suppression test. It is a good screening test because it is done easily and all patients with some form of Cushing syndrome will have an abnormal test (very sensitive, but not specific). If the test is normal, you can exclude Cushing. It is like the sedimentation rate in temporal arteritis, if the sed rate is normal then the patient does not have temporal arteritis. * If dexamethasone suppression test is abnormal, it could be Cushing or something else. Obese patients will have abnormal tests. Patients with stress, psychosis, even normal people can have abnormal tests. * Dexamethasone is a potent type of cortisol. When you measure hydrocortisone in the morning, you do not pick dexamethasone up on the assay. Overnight corticotropin releasing hormone (CRH) causes ACTH to increase cortisol, which peaks in the morning. If you give a patient dexamethasone at night, normal response is to see that enough cortisol is present and not release CRH and ACTH. So a normal response is suppressed AM cortisol. Abnormal response is to give 1mg dexamethasone at night then patient wakes up with high cortisol. * Follow-up test is urine 24h free cortisol. If cortisol is normal, no Cushing. If elevated free cortisol, this patient has some form of Cushing syndrome. Now localize the cause (pituitary, lung, or adrenal). * High-dose dexamethasone suppression test is used for localization. It is never used as a screening test because you miss all of the semi-autonomous causes (e.g. pituitary). * High-dose dexamethasone suppression test is normal (cortisol suppressed) if pituitary adenoma. Next step is get a CT scan or MRI of the pituitary and get neurosurgeon involved. Surgery is curative. * High-dose dexamethasone suppression test is abnormal (cortisol elevated still) now you are stuck. It could be either lung or adrenal. The difference you know is that patients with lung Cushing have high ACTH with hyperpigmentation and adrenal Cushing do not have high ACTH. * High-dose dexamethasone suppression test is abnormal, now check ACTH and if elevated assume lung cancer. Next step would be CT scan of the chest. This is likely unresectable. If ACTH is low, then adrenal lesion is likely so get CT scan of MRI of abdomen and adrenals. There is a possibility of excision here.

Hyperaldosteronism * Conn disease (rare) is caused by an adenoma in the mineralocorticoid apparatus, causing high levels of aldosterone. The main function of aldosterone is to keep salt in vessels, maintain vascular tone, and deals with bicarbonate, hydrogen ions, and potassium. * Autonomous aldosterone is secreted and aldosterone holds onto sodium (high Na+), causing hypertension. At the - 76 same time, potassium is wasted (low K+) and hydrogen ions are wasted (metabolic alkalosis). So these patients will complain of weakness. The low hydrogen ions leaves bicarbonate unopposed. The increased sodium increases intravascular volume (high BP) and this feeds back to renin apparatus which shuts off and tries to tell the aldosterone to stop, but it wont stop because it is an autonomous adenoma. * Primary hyperaldosteronism (Conn syndrome) is a rare form of secondary hypertension. Think about this as a cause if a hypertensive patient complains of weakness and headaches. Removal of the tumor is curative of the disease and treats the blood pressure as well. It is most common in young adult men. It is also called reninindependent aldosteronism. * You would think the patient with increased intravascular volume would have edema, but they do not. There is an unknown escape mechanism where the excess water gets wasted. * The most common form of hyperaldosteronism is secondary aldosteronism. It starts with decreased intravascular volume due to hypotension, diarrhea, vomiting, over-diuresis, edematous states (CHF, cirrhosis), etc. The depleted intravascular volume goes back to the renin apparatus. The renin apparatus read this and cranks up the renin, which cranks up the aldosterone. The aldosterone increases sodium in an attempt to get intravascular volume to normal. But, it will not get to normal especially in the edematous states. * In this process, potassium and hydrogen ions are lost leading to hypokalemic metabolic alkalosis. Also known as renin-dependent aldosteronism. * If a CHF patient is on furosemide and gets hypokalemia with metabolic alkalosis, you know you are causing secondary aldosteronism. So you are over-diuresing that patient. Treatment is to cut the diuretic dose.

* Young woman presents with hypokalemia, metabolic alkalosis, no hypertension, no edematous states. Think about secondary aldosteronism from surreptitious vomiting or use of diuretics to lose weight. * Gitelman syndrome is a rare inherited defect in the distal convoluted tubule of the kidneys. It causes the kidneys to pass sodium, magnesium, chloride, and potassium into the urine, rather than allowing it to be resorbed into the bloodstream. Bartter syndrome is a rare inherited defect in the thick ascending limb of the loop of Henle. These are both forms of secondary aldosteronism. * Primary and secondary aldosteronism both have muscle weakness, metabolic alkalosis, hypokalemia. The only difference, is that primary aldosteronism has mainly diastolic hypertension and secondary aldosteronism does not have hypertension. Edema is not seen in primary aldosteronism. Edema is seen in secondary aldosteronism if they are in the edematous states and not seen in other states. Congenital Adrenal Hyperplasia * Cholesterol is converted to progesterone, which is converted to aldosterone, cortisol, and sex hormones. Patients with congenital adrenal hyperplasia are missing enzymes to convert progesterone to aldosterone or cortisol. Thus everything gets shifted to the sex hormone pathway. * 21-hydroxylase deficiency accounts for 90% of congenital adrenal hyperplasia cases and is the only one that could be diagnosed in an adult. There is a block in the conversion of progesterone to aldosterone, causing an increase in 17-hydroxyprogesterone. * All females born with 21-hydroxylase deficiency have ambiguous genitalia (e.g. cliteromegaly), evidence of virilization. Thus, females are recognized early. Males have normal genitalia and can be missed. These patients have salt-losing nephropathy due to lack of aldosterone. Signs include hypotension and dehydration. * Non-classic form of 21-hydroxylase deficiency can occur; patient produces some aldosterone and only has symptoms during stress. This occurs in 1 of 100 cases of 21-hydroxylase deficiency, very rare. * Treatment is to give mineralocorticoids like hydrocortisone and fludrocortisone because hydrocortisone does not have enough mineralocorticoid action. Adrenal Insufficiency

* Most cases of adrenal insufficiency are due to tuberculosis worldwide. In the United States, most cases are due to idiopathic or autoimmune. Infections with MAI or CMV in HIV patients can cause adrenal insufficiency. * Adrenal insufficiency causes increased ACTH so there is hyperpigmentation, loss of mineralocorticoids causes low blood pressure, and elevation is eosinophils in serum for unknown reason. * Electrolyte problems are hyperkalemia, metabolic acidosis, hyponatremia with hypotension. * Diagnostic testing is to activate the adrenals by giving ACTH, the rapid ACTH test. Measure cortisol levels at zero, 1hr, 2hrs. If the adrenal is normal, you should see gradually increasing cortisol levels. Even if the cause is in the pituitary, the rapid ACTH test will likely be abnormal so you can get diagnosis there too. * Exception for rapid ACTH test is sick patient (stressed), then you can get random cortisol levels. This is probably the only time you can get random cortisol levels, and some physicians even question this. * Rapid ACTH test is negative (no increase in cortisol with ACTH given). Treatment is to replace missing hormones, give hydrocortisone 10-20mg/day plus fludrocortisone sometimes for mineralocorticoid action. Give - 77 enough to maintain the blood pressure and electrolytes at normal. * Adrenal insufficiency patient going for surgery or gets a febrile illness, then increase the maintenance hydrocortisone dose. If febrile illness, increase hydrocortisone two times dose. If major surgery, increase maybe up to ten times dose. Severe illness like pneumonia, increase to 75-200mg/day hydrocortisone. Pheochromocytoma * Pheochromocytoma is a form of secondary hypertension (also primary aldosteronism and renal artery stenosis). * Secondary hypertension is less than 5% of all hypertensive patients, but surgical correction of the underlying disease cures the disease and the hypertension. * Pheochromocytoma arises from the adrenal medulla causing increased levels of epinephrine and norepinephrine. * Rule of 10% for pheochromocytomas: 10% of patients are bilateral, 10% are malignant, 10% in children, 10% extra-adrenal, 10% re-occur.

* Most patients (60%) have stable chronic hypertension with pheochromocytoma, thus not the classic presentation. Consider pheochromocytoma when patient is young, you are using a lot of medication to control their blood pressure, and there is orthostasis despite the hypertension. E.g. patient is 200/110 laying down, 150/90 sitting up. These patients are volume depleted for unknown reasons. * Diagnostic test is urine free catecholamines. Older test is VMA (vanillylmandelic acid) and metanephrines. * Only after you find urine free catecholamines do you proceed to the abdominal CT/MRI. If you take random people off the street you will find 15-20% have some non-functional adenoma of the adrenal glands; these are called incidentalomas. Do not send an incidentaloma to surgery. * Treatment of pheochromocytoma is surgery. But if you go in and start playing with pheochromocytoma you will get a large norepinephrine and epinephrine surge. Patient could have complications or even die from this. * Treatment is first suppress epi/norepi by giving alpha-blockade. Medications include phenoxybenzamine and phentolamine (reversible). Do not use any other drug except alpha blocker as the first choice or else you leave the alpha receptors unopposed and the patient will have a hypertensive emergency. * Patient presents with high blood pressure and you start them on a medication like HCTZ or beta-blocker. The patients blood pressure goes up. This would be expected in pheochromocytoma since alpha receptors are open. * Treatment is surgical excision after blood pressure normalized with alpha blockade. * Some patients complain of symptoms like palpitations even with alpha blockade. Only if the alpha blockade is adequately controlling the blood pressure could you add a beta-blocker to control symptoms. * Treatment before surgery also includes IV fluids because they are volume depleted. * A patient gets a CT scan for suspected appendicitis. A tumor is found in the adrenal gland. The patients blood pressure is normal. What do you do next? This is likely an incidentaloma because BP is normal, do nothing. Hypogonadism * Kallmann syndrome is isolated gonadotropin deficiency, hypogonadotropic hypogonadism. This is due to a decrease in the release of LHRH. Key feature is anosmia (lack of smell). * LHRH causes the release of FSH and LH, which makes testosterone. * These patients present with small testes, eunuchoid features (large waist), and anosmia.

* Treatment is synthetic LHRH to normalize testosterone. * Klinefelter syndrome is the most common cause of hypogonadism. This occurs in about 1 in 400-500 births. * Chromosomes are XXY or XXXY, XXXY has more abnormalities. * Patients have normal LHRH, high FSH and LH, but the receptors for testosterone production are deficient. * Treatment is testosterone. * Klinefelter syndrome patients have a 20x greater risk of breast cancer. -----------------------------------------------------------------------------------------------------------------------------------------Kaplan Videos (2001) Hematology/Oncology with Dr. Conrad Fischer, MD -----------------------------------------------------------------------------------------------------------------------------------------Anemia * Anemia, regardless of etiology, is associated with the symptoms of fatigue, malaise, shortness of breath, lightheadedness, confusion. Symptoms are most dependent on the severity of the anemia. * Hct > 30-35 is usually asymptomatic. Hct 25-30 symptoms are fatigue, malaise. * Hct 20-25 symptoms are shortness of breath on exertion (dyspnea) due to lack of oxygen carrying capacity. * Hct 15-20 symptoms are lightheadedness, confusion, syncope. Symptoms are dependent on the patient too. * Young healthy patient might not die until Hct of 10-12. Take an older patient with vascular stenosis, they may die at an Hct of 24 or become symptomatic at 28. * Hct < 15-20 is associated with death. Cause of death is myocardial infarction, not high output cardiac failure. * High output cardiac failure takes months to develop and is not common in the United States since the precursor - 78 causes are usually managed. * Stroke is also not the cause of death in anemia. You can live without your brain functioning but cannot live without your heart functioning. Yep, you can live by being stupid and in love but not intelligent with no love. * Left ventricle cannot distinguish between anemia (decreased oxygen delivery), coronary artery stenosis (decreased oxygen delivery), and carbon monoxide poisoning (decreased oxygen delivery). * Hematocrit (Hct) is found to be low. What is the next step? Answer is look at cell size (MCV).

* Mean corpuscular volume (MCV) tells you the size of the RBCs, which helps you determine the etiology. If low it is microcytic, high is macrocytic, or normocytic. * Microcytic anemia (TICS): thalassemia, iron deficiency anemia, chronic disease anemia, sideroblastic. * Macrocytic anemia: B12 deficiency, folate deficiency, liver disease (liver cannot metabolize fats/lipids and they stick to the RBCs), medications (e.g. alcohol, methotrexate, zidovudine, mercaptopurine). * Alcohol directly suppresses bone marrow, thus is the most common drug cause of macrocytic anemia. Alcohol can also cause dementia, cardiomyopathy, peripheral neuropathy, myopathy. * Normocytic anemia: hemolysis, blood loss. Microcytic Anemia * Iron deficiency anemia is usually from blood loss. You only need 1mg/day of iron when not menstruating, 23mg/day when menstruating, 5-6mg/day when pregnant. Maximum absorbable is 34mg/day. * Thalassemia major requires a unit of cells every 2-4 weeks, say you get hemochromatosis from iron overload with iron deposits in organs (liver, heart, pancreas, skin, testicles). We cannot treat this form of hemochromatosis (secondary to thalassemia major) with phlebotomy. You could treat hereditary hemochromatosis or polycythemia vera with phlebotomy. * Unit packed RBCs has 1mg/mL of iron. So one little mL contains all the iron you need for a day. There are about 5mL in a teaspoon and 15mL in a tablespoon. Unit packed cells of 350mL is enough iron for a year. * Treatment of iron overload in thalassemia major is deferoxamine. * Most normal iron loss is through the GI tract. Half the volume of stool is cells, so you will continue to have bowel movements even if you do not eat. There is no real mechanism to get rid of excess iron. * Next step in management is iron studies. These will all look hypochromic and microcytic on peripheral smear. * Ferritin is the storage form of iron. Total iron binding capacity (TIBC) is unoccupied sites. * Iron deficiency anemia: low ferritin (most specific), high TIBC. Most sensitive test is bone marrow looking for stainable iron stores; but this is not the first test. RDW will be increased because the newer cells are smaller. Reticulocytes are usually larger, but the new cells will be more deficient than the old cells, iron running out.

* Anemia of chronic disease: high ferritin, low TIBC. * Sideroblastic anemia: high iron (only microcytic anemia with a high circulating iron level). Associated with lead poisoning. Hereditary deficiency in pyridoxine metabolisms, so treatment with vitamin B6 can help them. Myelodysplasia (MDS) is premalignant for leukemia (AML). * Dysplasia (e.g. colon polyps, cervix) is premalignant. Hyperplasia (e.g. fibrocystic breasts, benign prostatic hypertrophy) is not premalignant. * To have AML, you need 30% blasts. Refractory anemia, refractory anemia with ringed sideroblasts, chronic myelomonocytic anemia, and refractory anemia with excessive blasts are all the same disease; these are myelodysplasia with a certain percentage of blasts. * Thalassemia anemia: normal iron studies. Electrophoresis tells you what type of thalassemia. Hg A is two alpha chains plus two beta chains. Hg A2 is two alpha chains plus two delta chains. Hg F is two alpha chains plus two gamma chains. So in alpha thalassemia there are no beta chains, thus increased A2 and F. Single gene deletion patients are normal so not likely to be seen in clinic. Four gene deletion patients are dead. Two gene deletion is mild anemia and three gene deletion is moderate anemia. Who cares though, it comes down to hypochromic microcytic anemia with normal iron studies. * Latest and greatest treatment for iron deficiency anemia is iron. * Latest and greatest treatment for anemia of chronic disease is to treat the chronic disease. * Treatment of thalassemia minor (thalassemia trait) is do nothing, genetic counseling. Genetic counseling does not mean do all the karyotype testing, it just means tell the patient to avoid having sex with people who has the same genetic problems that they have (e.g. family planning). * Treatment for thalassemia major is blood transfusion every few weeks. Macrocytic Anemia * 72yo alcoholic man is brought to you by his son because the man is having memory loss and some numbness and tingling of the feet. Hct is 28, MCV is 110 (normal 80-100). What do you do first to establish a diagnosis? Answer is peripheral smear. If the smear is normal, these symptoms are from alcohol. If the smear is abnormal showing - 79 megaloblastic cells, then you know it is due to folate or B12.

* Only B12 and folate can make cells megaloblastic (hypersegmented). Alcohol, liver disease, and drugs can make cells macrocytic (large cells) but not megaloblastic. * Hematologically, there is no difference between folate deficiency and B12 deficiency. If you see hypersegmented polys, you have to order both the folate and B12 levels. * B12 deficiency has neurologic problems, folate deficiency does not. But you do not have to have one before the other. So B12 deficiency can occur without neurologic problems. Neurologic problems are memory loss, peripheral neuropathy, decreased vibratory sensation, subacute combined degeneration of the cord which accounts for the vibratory sensation problems and looks like tabes dorsalis. Posterior columns also contain large two-point discrimination pain fibers. In reality, you can get any neurologic issue that you see on a neuro exam. Peripheral is the most common. * Methylmalonic acid used with equivocal B12 level, it goes up in B12 deficiency. Schilling test is only done when you know you have B12 deficiency and you want to know the cause (bacterial overgrowth blind loop syndrome, most common pernicious anemia, rare Diphyllobothrium latum fish tapeworm). * Treatment of folate deficiency iswait for itfolate. Treatment of B12 deficiency is B12. * Folate treatment can fix the hematological problem of B12 deficiency, but not the neuro problem. Hemolysis * Causes of hemolysis include sickle cell disease, autoimmune, glucose 6 phosphate dehydrogenase (G6PD), hereditary spherocytosis, paroxysmal nocturnal hemoglobinuria (PNH). * All forms of hemolysis have normal MCV (80 to 100). * All present with high LDH but this is nonspecific and increased in anything that causes cell destruction, tissue destruction (muscle, brain, blood, lung). Sign nonspecific tissue damage, good sensitivity but poor specificity. * Reticulocyte counts will be high. Hemolysis is the only one that can be acute. You cannot get thalassemia or iron deficiency in the acute setting. Microcytic or macrocytic all have to be chronic. If you have an acute situation, look for normocytic anemia. * Hyperuricemia can only occur in cells that have the purines and pyrimidines of nuclei. You could see that in tumor lysis, rhabdomyolysis sometime, myeloma, leukemia, lymphoma. But, not hemolysis because no nuclei.

* Increased bilirubin can be seen in hemolysis. Some will have a low haptoglobin if the cells are destroyed in the vasculature because haptoglobin takes freely released hemoglobin to the spleen for recycling. * Sickle cell in 1 of 400 African Americans and 8% have the trait. 47yo man comes to the Emergency Department with a history of sickle cell and complains of terrible pain in the chest, back, and thighs. He is asking for narcotics to treat his, as he states, painful sickle crisis. His temperature is 98.6F, RR of 12, O2 Sat 100% on room air. You tell him you think he is not in pain. He says there is no test to disprove pain and that you cannot prove he is not feeling horrible pain. You note no lower extremity ulcers (a complication of sickle cell), not history of osteomyelitis (most common staph aureus, salmonella only happens in sickle cell), no gallstone history (bilirubin), no aseptic necrosis of the femoral head. How should you manage this patient? * Most accurate or specific test is electrophoresis for sickle cell. CBC, reticulocyte count, and bilirubin do not tell you if the patient has sickle cell. Peripheral smear is what you do for this patient. The smear can tell the difference between sickle cell trait (heterozygous) and disease (homozygous), sickled cells occur in the disease. Patients with sickle cell trait will have a normal smear. * We know that hypoxia, dehydration, acidosis, and infection predispose sickle cell patients to sickling. We also know that patients can sickle without any of those things. * Treat the above 47yo patient with oxygen, fluids, and pain medications. * Give sickle cell patients antibiotics without obvious signs of infection if they simply have a fever or leukocytosis. The low threshold for antibiotics is because these patients are immunocompromised due to function asplenia. These patients can die from pneumococcal or hemophilus sepsis very quickly. * Any patient with neutrophil count < 500 and a fever should get antibiotics (neutropenic fever). * Say you gave the 47yo patient oxygen, fluids, pain medication, and antibiotics. A few hours later he is worse with blurry vision because the sickle cells have reached his eyes, CNS disturbances, and priapism due to infarction by sickle cells of the prostatic plexus of veins that drain the penis. How do you treat this patient now? Treatment is exchange transfusion. * Hydroxyurea is used for prevention of sickle cell crisis, like leukotriene antagonists for asthma prevention.

* Heterozygote sickle cell patients are about 40-50% sickle hemoglobin and 50-60% normal hemoglobin. So for exchange transfusion you get rid of half of their (homozygote) blood and replace it with normal blood, which basically turns them into a heterozygote. * 21yo patient asks about testing because his father gets treated for sickle cell. The patient has never had symptoms. - 80 Test of choice here is electrophoresis. In asymptomatic patient, a CBC and peripheral smear will not be helpful. Testing shows sickle cell trait. Patient asks what that means for him. Answer is renal disturbance such as hematuria, isosthenuria, kidney infections. Also some genetic counseling. Less common worries would be getting sickle cell while climbing Mount Everest, but that could happen. * What is the normal drop in hematocrit with a sickle cell crisis? Answer is none. If there is a drop in hematocrit with sickle cell crisis think about aplastic crisis from parvovirus B19 or folic acid deficiency (no storage form). * Parvovirus freezes the bone marrow, so you can lose 20-30% of your hematocrit via reticulocytes over the next few days. * Autoimmune hemolysis should be considered as the cause when you have signs of hemolysis and a history of autoimmune disease, like SLE, lymphoma. Half the time it is idiopathic. * Drugs that cause autoimmune hemolysis penicillins, cephalosporins, sulfa drugs, rifampin. Mnemonic: It is like becoming allergic to your own blood, think of the drugs people are generally allergic to. The same drugs cause allergic interstitial nephritis. * Sodium metabisulfite test is a screening test for sickle trait that is sometimes used. It is an old test, where metabisulfite is added to blood, it binds up the oxygen and causes localized hypoxia to provoke sickling. * Homozygous sickle cell patients can have sickle cells even without crisis. So the smear can tell you if someone has the disease but cannot help you differentiate normal from crisis. * Patients with sickle cell disease are protected from malaria because when the falciparum enters the cell it acts as an oxidative stress so the cell commits suicide and hemolyzes. The malarial also does not recognize the cell as well because it is lacking certain receptors.

* Coombs test differentiates autoimmune hemolysis from other forms of hemolysis. Autoimmune is the only one that is Coombs test positive. * Treatment of bad autoimmune problems is steroids. If it keeps recurring, do splenectomy. * G6PD deficiency associated with Mediterranean descent, primiquin use, fava beans, dapsone use, sulfa drug use. Like all other forms of hemolysis, this is an acute drop in cells that is not blood loss. Most common oxidative stress is infection. * Acutely, measuring a G6PD level will show a normal level. This is because the most deficient cells are destroyed leaving the normal cells behind. Wait for a couple of months then do a G6PD level. * Oxidized iron in a RBC (precipitated oxidized hemoglobin) is Heinz bodies. They can be used to tell you that you have G6PD deficiency. This cell travels to the spleen and the spleen takes little bites of the iron, leaving bite cells. * G6PD deficiency is the only hemolysis cause that has an enlarged spleen because it is chronic. Splenomegaly takes time, it does not happen right away. * Treatment is to remove sources of oxidant stress, dont eat fava beans, avoid certain drugs. * Spherocytosis is diagnosed with an osmotic fragility test. Autoimmune hemolysis has spherocytes as well on peripheral smear. The antibodies attach to the RBC and take out little bites, eventually converting into a sphere. Spherocytes are the opposite of a target cell, which has a compact hemoglobin and a big floppy membrane. * Osmotic fragility test swells the cells a little bit causing lysis. * The problem is not that you have spherocytes, they carry oxygen and CO2 just fine. The problem is when the spherocyte meets the spleen and cannot get through hemolysis occurs. Thus, the problem is not the cell it is the spleen. Splenectomy does not remove the spherocytes, it just removes the cause of hemolysis. * Paroxysmal nocturnal hemoglobinuria (PNH) presents with just that complaint. Most people do not get up to urinate at night, so they see the hemoglobinuria in the morning. Thrombosis of large vessels (DVTs, portal hypertension, portal vein thrombosis) can occur but we do not know why. * PNH responds to steroids for unknown reasons, so that is the treatment. Leukemia * Forms: Acute myelogenous leukemia (AML), acute lymphocytic leukemia (ALL), chronic myelogenous leukemia

(CML), and chronic lymphocytic leukemia (CLL). * Acute forms are immature cells, pancytopenia, kills you quick (as short as 6-12 weeks). Chronic forms are more mature cells, found on routine screening, kills you slower (6-12 years even without therapy). * There is no known cause for chronic leukemias. We know acute leukemia can be caused by chloramphenicol, certain drugs, radiation, benzene use, etc. Diagnosis cannot be made on history. * Presentation for AML and ALL is based on the severity. Anemia gives malaise, low or dysfunctional white cells gives high fever and infection, thrombocytopenia gives bleeding. * Causes of pancytopenia include cirrhosis and portal hypertension causing hypersplenism as the spleen can sequester all three cell lines, aplastic anemia, bone marrow depressing drugs (most common alcohol, methotrexate, carbamazepine, chloramphenicol), radiation, any primary or metastatic cancer, autoimmune (SLE), nutritional - 81 deficiency (B12 and folate), marrow-invading infections (TB, fungus, virus). * The difference between all those causes and acute leukemia is blasts. * The 7-8 different types of acute leukemia (M0-M7) cannot be distinguished from history and initial tests, and they have the same initial therapy. They all present the same so no real need to know them unless you are an oncologist. * Exception is M3, pro-myelocytic, which causes DIC. The others are indistinguishable with initial tests. * M3 also has Auer rods, myeloperoxidase. cALLa seen in ALL (common ALL antigen). * Patient at clinic for routine exam and found to have high white count. This is how you pick up CML and CLL. * An enlarged spleen is most characteristic of CML. * Side Note: 25% of the population feels chronic fatigue at any given time, a little tired. * Patient found to have 62,000 white cells. How do you distinguish CML from CLL? * CLL cell on smear does not look different from normal lymphocyte. CML cell on smear does not look different from normal neutrophil. Yes you may see some smudge cells with CLL as you are putting down the cover slip it smudges the fragile cells, like stepping on a jelly donut. So these cells look normal but have abnormal function. * CLL Stages: 0 increased WBC. 1 lymph nodes. This is a 10-12yr survival. * CLL Stages: 3 anemia. 4 low platelets. This is a 1-2yr survival.

* CLL differential will be 90-95% lymphocytes. CML differential will be 90% neutrophils/polys. * You didnt want to get a Philadelphia chromosome on the patient with 62,000 lymphocytes particularly because you need to get a bone marrow biopsy for that. When you see the high neutrophils, then you are more confident it is CML which is associated with Philadelphia chromosome. * High white count with nearly all neutrophils on differential, then look for Philadelphia chromosome. * You wouldnt use metronidazole to try to kill viruses. You wouldnt use acyclovir to try to kill fungi. You wouldnt use fluconazole to try to kill bacteria. The same goes for leukemia drugs. * Treatment for AML is daunorubicin and Ara-C (cytosine arabinoside). Marrow transplant is an option. * Treatment for CLL is chlorambucil and prednisone. Fludarabine if that does not work. These patients are generally too old for bone marrow transplants. * Side Note: Treatment of multiple myeloma is melphalan and prednisone. * Treatment for ALL is daunorubicin, vincristine, and prednisone. Marrow transplant is an option. This leukemia is a little different because you can give CNS prophylactic therapy with intrathecal methotrexate. * Treatment for CML is bone marrow transplant if donor. Medical therapy is Gleevec (imatinib mesylate). Interferon is another option as it makes the Philadelphia chromosome negative. Philadelphia chromosome codes for tyrosine kinase, so this drug specifically inhibits the tyrosine kinase, a silver bullet. * For ALL and AML, do a round of chemotherapy to remove 99.9% of cells. One round of daunorubicin and Ara-C takes out 99.9% of leukemia cells in the body (remission induction). Then you stop so the patient doesnt die. Then you do another round and remove 99.9% of what was left (consolidation). Then a third round to remove 99.9% of what was left (maintenance). This seems like a huge change, but there were so many cells to start with that three rounds are needed. * You made leukemic cells today. Your immune system eliminated them. The ultimate goal of chemotherapy is to remove so many cells that your body can eliminate what is left. We would call this a cure, but hematologist/oncologists call this a sustained remission. If all the cells were not removed, they begin to divide and 6 months later you have leukemia again at the brink of death. * With recurrence after chemotherapy agents, do a bone marrow transplant.

* CML can convert to ALL in about 20-25% of patients (blast off). This is no good because these patients die. This is why we transplant immediately if a donor exists. * CML M3 is treated with all-trans retinoic acid (vitamin A derivative). Retin-A for acne is cis retinoic acid. * 73yo man comes to your office for advice on diarrhea prophylaxis prior to a trip to Mexico. Results from routine tests from a previous visit show a WBC of 75,000. What is the next step? Answer is differential. The differential shows 93% lymphocytes. What is the diagnosis? CLL. Now what? Answer is do nothing, no therapy. * Patient who is 50-60yo with CLL and advanced stage disease (hypersplenism, anemia, thrombocytopenia) should be treated with chlorambucil or fludarabine. There is no equivalent of Gleevec for CLL so were stuck with carpet bombing via chemotherapy. * Wait, what about the 73yo man with leukemia? You are going to just leave him with no therapy? He is at stage 0 with just a high white count, not at state 1 which is lymph nodes. Average age of man in United States is 76. Japanese women live the longest to age 84. Average survival for this guy is 10-12 years. * Chronic leukemias always have high white counts. Acute leukemias are high in 1/3, low in 1/3, normal in 1/3. Aplastic Anemia * CBC has WBC, RBC, platelets. WBC has neutrophils, eosinophils, lymphocytes. Lymphocytes have T-cells and - 82 B-cells. T-cell subtypes range from CD1 to CD134 thus far, T-killer. * So one day the T-killer cell wakes up and says, whoa, who are you guys? B-cells, platelets? What is this, how did you get into my marrow? T-killer cell develops homicidal ideation, cells are like yo buddy were the eosinophils youve always been with and T-killer cells is like uhh, no youre not, its a trick you want to kill me, well Im going to kill you first! The T cells start to kill the rest of the marrow, which is aplastic anemia. * Aplastic anemia can be from drugs, alcohol, autoimmune (for unknown reasons). * Treatment if chloramphenicol, drug, alcohol, benzene, radiation use is to stop the offending agent. * Most of the time with aplastic anemia it is autoimmune, a defect in the T-cell. * What drugs would you use to suppress T-cell function? Cyclosporine, specifically suppresses the T-cells, used in

kidney transplantation. * Treatment for autoimmune aplastic anemia is bone marrow transplant if you are young and have a donor, reset the bone marrow. If no donor, give medications (ATG, cyclosporine). * If your T-cells were injected into a horse, the horse would make antibodies against them. This is anti-thymocyte globulin (ATG). Give this to the patient, T-cells knocked out, bone marrow comes back. Myeloma * Plasma cells are making IgG and IgA that are detectable on serum protein electrophoresis. Plasma cells also making Bence-Jones protein (immunoglobulin parts that end up in urine), uric acid (hyperuricemia), osteoclast activating factor (OAF) which goes to bones and seeps calcium into serum. * Most common complaint is unexplained bone pain. Think myeloma with nontraumatic bone pain in old person. * Older patient who coughs and causes ribs to break, rolls over in bed and gets broken bone. * Immunoglobulins made against a single antigen (monoclonal). Pneumococcus shows up in the body, Hey buddy, Im here to kill you but body says dont bother me, Im fighting another antigen. The only person who can see this magic antigen is the body, with plasma cells delusional creating antigens against some antigen. It is so busy creating antigens and fighting this unknown/nonexistent thing that real infections run rampage. * Myeloma is > 10% plasma cells for diagnosis. * Treatment is melphalan and prednisone, but this is just to reduce the number of cells. It is difficult to transplant patients with myeloma because this occurs in older individuals and the transplant itself can kill the patient. Older people do not recover well when made pancytopenic and neutropenic with chemotherapy prior to marrow transplantation. Treatment is stem cell transplant, where patient is not immunosuppressed as much. * Man comes to you because his total protein was found to be elevated on routine visit. You do an electrophoresis that shows a monoclonal spike. Patient says he does not have bone pain, no pathologic fractures, no salt and pepper head, no humerus fracture from leaning against something. Calcium level is normal. No Bence-Jones protein in urine. Normal uric acid level. No anemia or renal dysfunction. Plasma cells are 5%. What do you do next?

* Answer is no treatment. 1% of these patients go on to myeloma but we cannot determine who does. This disease is called monoclonal gammopathy of undetermined significance (MGUS). Lymphoma * 32yo woman presents with neck mass, fever, weight loss, sweats. What would you do next to determine the diagnosis? Answer is excisional biopsy. Needle biopsy is not good enough. Excisional biopsy comes back positive for Hodgkin lymphoma. What is the best thing to do next? Answer is staging because treatment is based on stage. * A needle biopsy smear will look normal. You need to see architecture. CLL and lymphoma are very similar, they are abnormal lymphocytes that look relatively normal on smear. * Hodgkin disease 80-90% of the time presents in stage I or II. * Non-Hodgkin disease 80-90% of the time presents in stage III or IV. * Stages have A and B, B is symptoms of fever, weight loss, night sweats. This implies a worse prognosis. * Stage I: One group of lymph nodes. Stage II: Two groups of lymph nodes, same side of diaphragm. * Stage III: Both sides of the diaphragm. Stage IV: Diffuse disease, such as in marrow. * Early stage (I to IIa) is radiation and late stage (IIb to IV) is chemotherapy. There is some leeway here. * Staging is important. If you radiate her neck and she has disease in her pelvis, she is going to die. * How do you determine if local? You could start with CXR. If it showed mediastinal and hilar lymph nodes for this 32yo with a neck mass, she is at stage IIb. So she would get chemotherapy. * CXR negative, then could do chest CT. If chest CT negative, then abdominal and pelvic CT. Say abdominal and pelvic CT showed lymph nodes consistent with lymphoma in the 32yo with neck mass? Answer is chemotherapy because she is at stage IIIb. * Testing for localization: CXR, chest CT, abdominal and pelvic CT, bone marrow aspiration (lymphangiogram was old test), laparotomy (e.g. in spleen not seen on CT). If all of these are negative, then you can radiate a local lymphoma. - 83 * Treatment of non-Hodgkin lymphoma is CHOP. * Treatment of Hodgkin lymphoma is ABVD/MOPP. * Radiation of local disease results in a cure 90% of the time. Why dont we just chemo everyone? Side effects.

* Most common side effect of chemotherapy (CHOP, MOPP, ABCD) is sterility in 95% of patients. Testicles are more vulnerable to this effect. * Busulfan and bleomycin cause pulmonary fibrosis. Adriamycin gives cardiac toxicity. Vincristine and vinblastine causes peripheral neuropathy. Cisplatin causes nephrotoxicity and ototoxicity. Cyclophosphamide causes hemorrhagic cystitis. 1% a year develop AML, 1% will develop aplastic anemia. * 32yo woman with a neck mass, fever, weight loss, night sweats. Excision biopsy shows Reed-Sternberg cells. Pelvic CT scan is positive for lymph nodes consistent with lymphoma. What would you do next in the management of this patient? Stage is IIIb so do chemotherapy with ABVD/MOPP. Bleeding Disorders * 26yo woman presents with epistaxis and petechiae. Her prothrombin time is normal at 11s, PTT is prolonged at 52s, platelet count of 217,000. What is the most likely diagnosis? This is platelet-type bleeding with a normal platelet count; von Willebrand disease. * It is not vitamin K deficiency because PT is normal. It is not hemophilia because she is a woman. * First thing to do with a bleeding question is determine the nature of the bleeding before you look at the labs. Is this factor related bleeding (deep like hemarthrosis in joint, hematoma in muscle) or platelet bleeding (superficial like epistaxis, skin, gums, mucosa, vagina)? * Hemophilia: normal PT, prolonged PTT, normal platelets, factor-type bleeding. * Vitamin K deficiency: prolonged PT, normal PTT, normal platelets, factor-type bleeding. * Von Willebrand disease: normal PT, prolonged/normla PTT, normal platelets, platelet-type bleeding. * Glanzmann thrombasthenia and Bernard-Soulier syndrome are rare, so in the 26yo patient so most likely von Willebrand since it is the most common hereditary coagulopathy. * Say you have platelet-type bleeding with normal platelet count. What is the next step in management with the assumption of von Willebrand? Answer is bleeding time (prolonged). Then get a von Willebrand factor level. * How do you do bleeding time? You put on a BP cuff to occlude the veins of the arm, but enough so the artery is open. Then you take a razor blade and cut the patient. Then you time the bleeding. Bleeding time is a test of platelet function, so do not do a bleeding time if the platelet count is low (< 50,000) because bleeding time will always be

abnormal. So do bleeding time when you see platelet type bleeding with a normal platelet count. * Factor VIII antigen (vWF) travels bound to the factor VIII coagulant portion (hemophilia), so it is not unusual to see prolonged PTT in von Willebrand disease. * Treatment of von Willebrand disease is desmopressin (DDAVP). Cryoprecipitate is pooled blood products and can transmit disease, thus it is not used much. If DDAVP does not work, give factor VIII replacements because it comes with some von Willebrand factor (vWF). If that doesnt work, then you might give cryoprecipitate. * Desmopressin releases sub-endothelial stores of vWF, where it is made and stored. This rapidly brings up the vWF level, leading to good clotting. * Factor replacements (VIII, IX) are clean because they are no longer pooled blood products. They are made with E. coli and recombinant technology. Hepatitis B vaccine is made this way too. * In order for platelet and endothelium to stick to each other they need vWF. Factor VIII and vWF sit in the endothelial lining. To see if there is functional vWF, you need an artificial endothelial surface. This is the ristocetin test (artificial endothelial surface). If you have platelets and ristocetin with vWF present they will stick. If vWF is not there, the platelets do not stick to ristocetin. * After the initial clot is formed via vWF with platelets and endothelium, fibrin come in and cross-links the clot. Factor XIII helps preserve the clot. Fibrin is broken to D-dimers (fibrin parts) by plasmin. Tissue plasminogen activating factor (tPA) so that the plasminogen turns into plasmin and then the plasmin chops up the fibrin linked clot. Fibrin split products gets split off from fibrinogen to activate it to fibrin. * If you wanted to know how many beers someone drank, you could look at the empty beer bottles (D-dimer) or the bottle caps that were split off (fibrin split products). * 26yo with platelet-type bleeding. PT is normal, PTT normal, platelet count low. The spleen is not enlarged on exam. This is idiopathic thrombocytopenic purpura (ITP). What is the next test for diagnosis? Answer is antiplatelet antibody test. What is the treatment? Steroids. * They tell you the spleen is normal to rule-out splenic sequestration. * TTP is associated with renal failure and hemolysis. * DIC has low platelets, increased PT, increased PTT, with factor-type and platelettype bleeding.

* Megakaryocytes are precursors to platelets, so increased in ITP, but you need a bone marrow biopsy to see these. - 84 * Treatment of ITP is steroids. If patient recurs then do splenectomy. * This is very much like autoimmune hemolysis. Increased reticulocytes are like increased megakaryocytes. Test for autoimmune hemolysis is antiRBC antibody (Coombs) versus antiplatelet antibodies for ITP. Steroids for both. Splenectomy is the steroids fail and the patient recurs. * 26yo woman with platelet-type bleeding, melena, subdural hematoma, epidural hemorrhage, subarachnoid hemorrhage as well. PT is normal, PTT is normal, platelet count is low. So this patient has ITP and is bleeding into her brain and bowel. What is the next step in management? IV Ig or RhoGAM. * Plasmapheresis sounds like a good idea, but usually not necessary. There is a circulating antibody (Guillain-Barr, myasthenia crisis, Goodpasteur, ITP) why not get rid of it? Just not necessary in most cases. * Fresh frozen plasma (FFP) has no platelets. * Memorized learning or non-understood learning is one of two things. Pyloric stenosis because youll see it again [the food]. Or, radioactive material, it has a half-life and will decay. * Macrophages bring the platelets that have antiplatelet antibodies and drag them to the spleen for processing. Giving IV Ig stuffs up all the Fc receptors on the macrophage. Thus the platelets are not processed. Thus, this is the quickest way to raise the platelet count. * 62yo man has a prostate mass and is going to get a biopsy. The surgeon sends the patient to you because the PTT is prolonged. PT is normal, platelet count is normal. The patient has no bleeding history and has had prior surgeries without event. He did have a DVT three years ago and last year, a PE before. What do you do next? * Hemophilia presents as a child that slides into base playing baseball and gets a hemarthrosis. * Liver disease has very high PT, high PTT, normal platelets. If you give vitamin K and the patient gets better it was vitamin K deficiency, if labs do not change it is liver disease. * Lupus anticoagulant: Elevated PTT, normal PT, normal platelets. History of recurrent abortions, DVTs, PEs. * Treatment of lupus anticoagulant is to replace factors, give FFP. * The only hypercoagulable state that has an elevation in bleeding labs (PTT increased) is lupus anticoagulant.

-----------------------------------------------------------------------------------------------------------------------------------------Kaplan Videos (2001) Rheumatology with Dr. Asher Kornbluth, MD -----------------------------------------------------------------------------------------------------------------------------------------* Pay attention to the number of joints involve, like a monoarticular arthritis with a single red hot knee or do they have polyarticular arthritis affecting a few joints (oligoarticular arthritis) or more diffuse with or without symmetry. * Constitutional symptoms are fever, night sweats, weight loss, loss of appetite, loss of energy have corresponding inflammatory laboratory results such as anemia of chronic disease, high sedimentation rate (Sed rate), high platelet count, and other acute phase reactants like C-reactive protein (CRP). Rheumatoid Arthritis (RA) * 26yo woman with no prior medical history presents with a three week history of joint swelling and stiffness. She points to all her proximal interphalangeal (PIP) joints as well as the metacarpal phalangeal (MCP) joints. As a side point, this is 20 joints involved so polyarticular. She also has pain in her wrists. She has stiffness for two hours every morning since these symptoms started and the symptoms improve as the day progresses. She denies back stiffness or back pain. She has fatigue and low grade fever (think generalized inflammatory condition). On exam the wrists, MCPs, and PIPs are red and swollen on both hands. The distal interphalangeal (DIP) joints are not involved. * Morning stiffness is a classic disease marker for rheumatoid arthritis, although it can be seen in other disease. * Ratio is about 3:1 women to men for RA, usually presents in young adults. Classic MCP and PIP involvement and the joint involvement is symmetric. C1-C2 joint is the only area of the spine that can be involved in RA. * Rubor (red), calor (hot), dolor (pain) for joints inflammation. Arthralgias are joint aches, not inflammation. * In RA, DIPs are almost never involved. Psoriatic arthritis is similar to RA but psoriatic arthritis affects DIPs. * What is the most likely diagnosis in the 26yo woman? Rheumatoid arthritis, an autoimmune disease. * What imaging tests would be abnormal in this patient? Hand joint x-rays showing erosions in the joint space, loss of joint space and inflammation. Not CXR, not ECG, not CT scan of head, not x-rays of lumbosacral spine.

* Labs looking for anemia of chronic disease (MCV low normal, Fe low, TIBC low, ferritin normal or high because ferritin can act as an acute phase reactant), high ESR, increased platelet count, high CRP. Normal electrolytes, kidney function, liver function, glucose level. Glucose could be up as a complication of steroids. * Diagnosis is not made by a single serologic test being positive, they only help confirm diagnosis. * Serologic tests include rheumatoid factor (IgM antibody against IgG). * At least 70% of patients are have seropositive RA, meaning RA with positive rheumatoid factor. * Extra-articular manifestations of RA include splenomegaly and neutropenia (triad is Felty syndrome). Another manifestation is pulmonary nodules (Caplan syndrome), can get fibrosing changes causing restrictive disease. * If a joint is swollen and/or fluid filled compared with other joints, do an arthrocentesis. - 85 * Arthrocentesis: non-inflammatory (osteoarthritis/DJD) has WBC < 2,000/mL, inflammatory (RA) has WBC from 5,000 to 50,000/mL, septic joint has WBC > 75,000/mL. * Say patient has RA with symmetry except for a knee that is 3x larger than the other knee. Arthrocentesis shows WBC 100,000/mL, so that joint is also infected. * Treatment for mild RA is aspirin or NSAIDs, both shown to be as effective. * Major complication of aspirin (ASA) and NSAIDs is upper GI ulcer and bleed. This is still a cause of death in the elderly. Aspirin toxicity at high dose can cause tinnitus and high anion-gap metabolic acidosis that is also associated with a respiratory alkalosis (increased respiratory drive). Interstitial nephritis is an uncommon complication of aspirin. NSAID complications also include renal insufficiency, especially in the elderly who can be dry. NSAIDs can affect CNS, confusion and delirium in high doses. * Acetaminophen is not an anti-inflammatory, thus it is not used in inflammatory disease like RA. Side-effects of acetaminophen (APAP, paracetamol) include liver toxicity and does not cause upper GI ulcers. * Arachidonic acid is metabolized to phospholipase into 2 pathways, cyclooxygenase (COX) and lipoxygenase. Products of COX are prostaglandins and prostacyclin. Products of lipoxygenase are leukotrienes, which are proinflammatory.

Prostaglandins mediate inflammation and pain. Prostaglandins are also cytoprotection mediators in the stomach and are helpful for renal blood flow. * COX comes in COX-1 and COX-2. Cytoprotection in stomach is part of COX-1. Renal vasoconstriction is part of COX-1 blocking. Inflammation and pain are part of COX-2. Thus blocking only COX2 would relieve the pain and inflammation while preventing COX-1 renal vasodilation and maintaining COX-1 cytoprotection. * Celecoxib is a COX-2 selective inhibitor. Rofecoxib was pulled from the U.S. market in 2004. * Treatment for RA should not include long-term steroids because of complications. Steroids are used in the shortterm for a patient that is not relieved with aspirin or NSAIDs. * Disease modifying anti-rheumatic drugs (DMARDs) include gold, penicillamine, azathioprine, and methotrexate. The most useful drug is methotrexate for long-term maintenance of RA. * Methotrexate (MTX) side-effects include liver fibrosis, cirrhosis particularly with cumulative high doses, pulmonary hypersensitivity reaction, bone marrow suppression. * RA patient presents with fever, pleuritic chest pain, infiltrate, and is on DMARD therapy. Answer is stop the methotrexate and give steroids for pulmonary hypersensitivity reaction. * What lab test should you following in a RA patient on long-term MTX? Answer is LFTs and CBC. * Any patient on MTX should also be on folate to help reduce the risk of bone marrow suppression. * Steroid toxicity is common so might as well put steroids on the problem list if a patient is taking them long term. * Steroid toxicity includes cataracts, glaucoma, psychosis, personality disturbances (anxiety, depression, moodlability), uncommon CNS infections like Listeria monocytogenes meningitis (not seen in immunocompetent), candidal thrush, moon facies (big, round), osteopenia and osteoporosis even at a young age, aseptic necrosis (avascular necrosis, osteonecrosis), increased risk of GI bleed. * Osteoporosis with steroids is dependent on dose and duration of steroid use. * Lab test abnormalities with steroids include metabolic alkalosis and hyperglycemia. * Rheumatoid arthritis patient presents with a swollen painful calf. There is no palpable cord and the patient has no risk factors for DVT. This is ruptured Baker cyst in the popliteal space. No need to do diagnostic tests here. * Patient presents after a motor vehicle accident. They have a history of rheumatoid arthritis. What should you be

careful of during intubation? Do not hyperextend the neck due to chance of atlantoaxial joint (C1-C2) subluxation. Systemic Lupus Erythematosus (SLE) * 35yo woman is brought for the evaluation of confusion for a day. Her friends and family inform you that she did not know how to get home from work and lately she has not been herself. You find that the patient has elevated blood pressure, decreased air entry at the right lung base with dullness to percussion (pleural effusion), and symmetrical joint swelling of the wrists and MCPs. Chemistry profile shows an elevated creatinine to 2.4 and there is protein in the urine on urinalysis. What blood test might you use to confirm your suspicion? * Anti-mitochondrial antibodies seen in primary biliary cirrhosis (PBC). * Anti-histone antibodies seen in drug-induced lupus. * Anti-double stranded DNA antibodies seen in lupus, particularly specific in active lupus with renal involvement. * Anti-centromere antibodies seen in scleroderma (CREST). * Anti-smooth muscle antibodies seen in autoimmune hepatitis. * Anti-nuclear antibodies (ANA) is very sensitive in lupus, so it will be positive in most cases of SLE but there are many false-positives. If ANA is negative though, it helps rule out lupus. * You rarely see a patient with lupus that has joint involvement as their chief complaint. * Ratio is about 10:1 women to men for lupus. - 86 * Symptoms of lupus include CNS (lupus cerebritis causing change in mental status, change in personality, depression, psychosis, seizures), alopecia, malar butterfly purple rash, discoid lupus (purplish disc-shaped discoloration usually after sun exposure), aphthous ulcers, pleural effusions (lupus serositis), pericardial effusion, sterile vegetation on heart valves (less common, Libman-Sacks endocarditis), renal disease (glomerulonephritis which is hallmarked by RBC casts in urine and proteinuria in nephrotic range), recurrent second trimester abortions due to hypercoagulability from anti-phospholipid antibody (lupus anticoagulant) causing placental vessel thrombosis, joints typically symmetric with arthralgias more than arthritis. * Labs abnormalities for lupus include mild leukopenia, anemia of chronic disease, autoimmune hemolytic anemia, autoimmune thrombocytopenia (decreased platelets, ITP), renal insufficiency (increased creatinine), RBC casts and

protein in urine. * Serology for lupus includes positive ANA (peripheral rim pattern), anti-DS DNA, low complement level (CH50) particularly with active lupus nephritis, less likely is anti-Ro (SSA) and anti-LA (SSB) antibodies. * Anti-Ro antibody is associated with neonatal lupus (mother has anti-Ro), babies get complete heart block. * Drug induced lupus causes include hydralazine, procainamide, isoniazid, alpha methyl-dopa. * These patients have prominent serositis, so pleural effusions and pericardial effusions. * Symptoms include arthralgias, fever, fatigue, no CNS involvement, no kidney involvement. * Patient presents with pleural effusion, fever, joint soreness, malar rash, increased creatinine, personality changes. They are taking hydralazine for hypertension. What antibody should you check? Answer is anti-DS DNA because this is true lupus and not drug-induced. Drug-induced would not have CNS or kidney involvement. * Test for drug-induced lupus is anti-histone antibodies. These patients can get false positive VDRLs, they will not have a positive FTA (fluorescent treponemal antibody). * Treatment of SLE is complicated and usually done by the rheumatologist. Know basic drugs though. * Treatment of SLE is NSAIDs for arthralgias and serositis. Steroids used for CNS involvement, kidney involvement, and autoimmune hematologic problems. With nephritis, if steroids are not enough add cytotoxic drugs like cyclophosphamide or azathioprine. * Treatment for SLE with second trimester abortions includes aspirin or low molecular weight heparin to preserve the pregnancy and prevent a thrombotic event. * Treatment for SLE includes liberal use of sunscreens. Scleroderma * 36yo woman comes to you because of skin tightness and painful fingertips with exposure to cold. These symptoms have occurred for over one year. Exam notes tight skin on the face with a shiny appearance. BP is 165/100. Laboratory tests reveal a macrocytic anemia and elevated creatinine. What is the diagnosis? Scleroderma. * Scleroderma is more common in women. * Painful fingertips with exposure to cold is Raynaud phenomenon, which is not unique to scleroderma. Color

changes can occur (white, blue, red). Sensitivity can be so exquisite the patient cannot walk into a room that has air conditioning running without their hands becoming painful. Patients may wear gloves in the summer time or be unable to hold a cold iced drink. * Sclerodactyly is skin tightness of the fingers that is so tight it causes bone resorption. * Progressive systemic sclerosis (PSS) is a term to denote scleroderma patients with the most severe disease. Renal involvement is the organ to worry about here and can lead to death, not seen in CREST syndrome. * CREST syndrome is the milder form of scleroderma. * CREST: calcinosis (calcium deposits), Raynaud phenomenon, esophageal hypomotility (severe reflux), sclerodactyly, telangiectases (usually on face). * Esophageal smooth muscle and LES replaced with collagen, leading to worse GERD you may ever see. * Renal involvement in PSS is exacerbated by elevated blood pressure. This is because the renal arteries are replaced by collagen and cannot dilate appropriately, so low renal blood flow due to sclerosis of the renal vessels. The kidneys behave as if they are pre-renal (vasoconstricted), triggering the RAAS pathway. * Treatment of choice for scleroderma with hypertension is ACE-I (lisinopril, captopril, ramipril) to block RAAS. If the patient cannot tolerate ACE-I (e.g. cough), give angiotensin II receptor blocker (ARB) like losartan. * ACE-I cough is an allergic phenomenon related to bradykinin alterations. * 36yo woman with scleroderma, has painful fingers, horrible GERD, hypertension, calcinosis. What is the most important symptom to treat? Answer is treat hypertension aggressively; this is critical to prevent renal crisis. * Treatment should not involve steroids, they are not beneficial in scleroderma and have major side-effects. * Treatment for scleroderma is ACE-I primarily for hypotension. * Treatment for scleroderma can includes penicillamine (some skin manifestations) and calcium channel blockers like nifedipine or nitrates (Raynaud phenomenon). - 87 Sjgren Syndrome * 42yo woman presents with some peculiar symptoms that she has had over the past year or so. She informs you that

she feels there is constantly something in her eyes like dust or sand. She says dry and solid foods are painful to swallow. You are perplexed by her complaints but decide to examine her and find that she has bilateral parotid enlargement but otherwise an unremarkable examination. An ANA test is positive. What specific ANAs do you expect to be positive in this patient? * Triad of dry eyes, dry mouth, and parotid gland enlargement is Sjgren syndrome, an autoimmune disease. * Sjgren syndrome sometimes found by itself, often found in association with other autoimmune diseases such as lupus, rheumatoid arthritis, and scleroderma. A combination of diseases is considered a mixed connective tissue disease (MCTD). These patients tend to have an antibody known as anti-RNP (ribonucleoprotein). * Diagnosis is suspected based on clinical presentation, reinforced with anti-RNP if associated with MCTD. * Schirmer test measures moisture in the eyes (decreased tears). Biopsy of salivary glands shows lymphocytic infiltration. ANA can be positive, also anti-Ro, and anti-LA antibodies. * Treatment is symptomatic with artificial tears and sucking candies, not usually a majorly disabling disease. Ankylosing Spondylitis * 27yo man presents with complaints of severe lower back pain and stiffness that have been bothering him for the past five years. He informs you that the stiffness is more apparent in the morning when he awakes, lasting sometimes for more than two hours. The only thing improving these problems is exercise. On exam he has a 2/4 diastolic murmur over the second right intercostal space and decreased range of motion of the lumbar spine. What is the most likely diagnosis? Ankylosis spondylitis. * Ankylosis spondylitis also associated with sacroiliitis, fusion of the SI joint. The first x-ray finding may be this fusion, suspected when there is this presentation and pain in the pelvic girdle area. This is associated with IBD; the two conditions (IBD and ankylosing spondylitis) can follow independent courses. * Not many pains improve with exercise, this is one of them. RA spares the back, so even though there is morning stiffness you can basically rule out RA. * Seronegative arthropathies (spondyloarthropathies) are more common in men than women. These often have prominent extra-articular manifestations. Genetic marker HLA-B27 positive in most of these.

* Aortic insufficiency often seen in ankylosing spondylitis, unknown cause. * There is typically a lumbar lordosis and thoracic kyphosis in normal patients. Ankylosing spondylitis patients lose this curvature giving a straight spine. Lumbar lordosis is lost first, then thoracic kyphosis. Typically the neck is spared or involved only later in the disease progression. These patients walk hunched over with a straight back, associated with a lot of discomfort. * Over time, AP x-ray of spine shows squared-off vertebral bodies and loss of intervertebral spaces. This is referred to as bamboo spine or bamboo shoot spine. * We do not know what causes ankylosing spondylitis and we do not have medications to reverse it. * Treatment is NSAIDs (not steroids), physical therapy, IBD treatment if needed, supportive therapy. Reactive Arthritis (Reiter Syndrome) * Patients get an arthritis as a reaction to an infection elsewhere in the body. Infections usually non-gonococcal urethritis (NGU) like chlamydia and infectious diarrheas (campylobacter most common, yersinia, shigella). * Reactive arthritis usually small joints like hand, usually symmetric. * Symptoms include conjunctivitis, oral ulcers, genital ulcers, urethritis. * Mnemonic: cant see, cant pee, cant climb a tree for conjunctivitis, urethritis, arthritis. * Blood test to look for in this clinical situation is HLA-B27. * Treatment for NGU reactive arthritis is treat underlying infection and treat arthritis with NSAIDs. * Treatment for diarrheal reactive arthritis is to treat invasive diarrhea cause (prior to getting arthritis). * Do not make the diagnosis of reactive arthritis unless you know what the arthritis is reacting too. Psoriatic Arthritis * Severe destructive erosive changes in the MCP, PIP, and DIP joints. * HLA-B27 positive, more prevalent in men than women. * Symptoms include prominent pitting of the nails, psoriatic skin changes, psoriatic patches on the scalp. Swelling of the fingers can lead to a condition referred to as sausage fingers. * Treatment with some success is methotrexate. Enteropathic Arthritis * Enteropathic arthritis seen with inflammatory bowel disease. * This does follow a similar clinical course to the underlying IBD (ankylosing does not). * Arthritis is typically symmetric, small joints, hands.

- 88 * Treatment should avoid NSAIDs since leukotrienes are pro-inflammatory and bad for IBD. * Other symptoms include erythema nodosum (painful red nodules on lower extremities) and pyoderma gangrenosum (pus ulcers on lower extremities). Erythema nodosum gets better with treatment of arthritis. Monoarticular Arthritis * Generally a single hot joint presents acutely. Causes include sepsis and crystalinduced. * Sepsis is most often caused by staphylococcus aureus infection or gonococcal infection. * Crystal arthritis causes are gout or pseudogout. * There are many other causes, like Lyme disease, but there are much less common. * Gonococcal arthritis associated with sexual activity. Staph arthritis associated with IV drug abuse. * Middle aged man with a history of gout tells you to the minute he remember when his foot became painful. * No matter how classic the story is, you cannot make a diagnosis of the cause of monoarticular arthritis. * Test every time for the first step in the work-up of monoarticular arthritis is arthrocentesis. * Joint fluid analysis should include WBC count, 25-50000/mL in gout/pseudo gout and > 75000 in septic usually, but a very hot gout joint could be 75000. A WBC < 5000 is more likely traumatic than gout or septic. Do examination of joint fluid under polarizing light microscopy (for crystals) or do a Gram stain and culture. * Most patients with a septic joint, even painful swollen hot red, will have negative Gram stain and cultures. But, if you do find the Gram stain or culture you have a diagnosis (so not very sensitive but very specific). * Gout crystals are monosodium urate which is negatively birefringent (double refraction) needle-shaped. Crystal is yellow when parallel to the slow ray of the compensator. * Pseudogout crystals are calcium pyrophosphate which is positively birefringent rhomboid-shaped. Crystal is blue when parallel to the slow ray of the compensator. * Staph aureus gets into joint via direct introduction (e.g. trauma from MVC or skiing accident) or hematogenous spread (e.g. infected IV catheter, infected central line, IV drug abuser, endocarditis, trauma elsewhere in the body). * Staph aureus Gram stains positive, blue cocci in clusters. Treatment is nafcillin or vancomycin if allergy.

* Gonococcus Gram stains negative, pink cocci in pairs and chains. Treatment is ceftriaxone, safe in pregnancy. * Neisseria gonorrhea is always hematogenous spread, which implies the patient has had a period of bacteremia where the gonococcus is floating through the blood stream. They have urethritis with discharge followed by a flulike illness with constitutional symptoms then perhaps a fleeting rash (petechial, rarely vesicular or pustular) and even tenosynovitis (red streaking along the path of a tendon). * Gonococcal urethritis can be treated with normal dose IM ceftriaxone. A gonococcal joint implies there has been a bacteremia, so you cannot use the same dose. Treatment is several days of parenteral (IV) ceftriaxone. An expected response is prompt relief of joint swelling and fever. If joint space remains inflamed then repeat the tap and the WBC should be coming down day by day if the patient is adequately treated. If you do arthrocentesis the next day and the WBC count has not moved, you have the wrong bug or the wrong drug. * Example would be Gram positive clusters seen on arthrocentesis. Patient put on nafcillin. A day later they spike fevers still and have high WBC on arthrocentesis. This is probably nafcillin-resistant staph, so give vancomycin. Gout * 32yo man comes in with a history of right ankle swelling that occurred the night before. He has noticed his ankle is red, warm, and very painful. He has no prior medical history and takes no medications. He has not had recent trauma to his ankle. He occasionally drinks alcohol. Examination shows red swollen ankle with evidence of an effusion. The range of motion is restricted. What is the first step in the evaluation of this patient? Answer is tap the joint; arthrocentesis. * Ankle tap shows WBC of 40,000. What is this consistent with? Gout or pseudogout. Now what do you do? Answer is polarizing light microscopy. Negatively birefringent crystals are seen in association with polys (PMNs) and some crystals inside polys (phagocytized). But, the patient doesnt care about a diagnosis. They came to you to feel better. So what are you going to do? NSAIDs now and repeat every few hours. The patient should feel better within the first dose or two, the inflammatory response comes way down with NSAIDs (e.g. indomethacin). * Another option is colchicine repeatedly until there is decreased pain. The problem with frequent high doses of PO

colchicine is diarrhea. It can be dose-limiting when giving repeated doses. * Another option is short-term steroids. Best choice though is oral NSAIDs. * Patients with even the most severe acute gouty arthritis might have a normal serum uric acid or even a low serum uric acid at the time they are having their attack. A serum uric acid that is normal or low should not rule out the diagnosis of gout. All patients that have gout have a predisposition to hyperuricemia, but that does not mean it occurs at the time of their gouty attack. * Uric acid is a product of nucleic acids from cell nucleus DNA. DNA broken down to purines to uric acid. Normally, we excrete uric acid into urine (no kidney or liver metabolism). - 89 * Hyperuricemia occurs if you cannot excrete fast enough or you product too much. 90% of adult patients with hyperuricemia is due to under excretion. Only about 10% are over producers, typically tumor lysis syndrome, so we give the patients lots of hydration and diuretics if needed to flush the kidneys, plus we give allopurinol to inhibit xanthine oxidase which is an enzyme important to the production of uric acid. * Probenecid is given to under excreters of uric acid to increase the excretion of uric acid; which is the majority of patients with gout. Notice acute gout medications (NSAIDs) have nothing to do with serum uric acid levels. * Colchicine inhibits neutrophil chemotaxis, again nothing to do with uric acid. When the neutrophils are floating around in the joint space and find a gout crystal, they gang up on the crystals and cause inflammation. Colchicine prevents this process from happening. * Do most patients with hyperuricemia develop gout? Probably not. The most common cause of hyperuricemia is renal insufficiency. Whatever causes the patient to have renal failure will result in under secretion of uric acid. * Patient has a history of gout and hyperuricemia. They begin to develop kidney stones. Which drug would be best for them? Answer is allopurinol, which reduces the amount of uric acid in the urine. The last thing you would want to do is put more uric acid in the urine, so avoid probenecid in this situation. * Say the gout patient with an inflamed ankle came back 6 months later with an inflamed knee. Do we assume gout and treat? No, tap the joint. Should you tap the joint every single time? Not necessarily, if the joint is new and it is

only the second episode then you should repeat the paracentesis to make sure you do not miss a septic joint. If you miss a septic joint the patient will have rapid destruction of the joint. * If septic joint is in the differential it is essential to do a paracentesis to ensure the joint is not septic. * If patient only has gout attack a couple times a year, they may be willing to suffer through some pain and take NSAIDs to relieve the attack. Then that patient could avoid daily medication with allopurinol or probenecid. * Say patient has four documented episodes of gout and has limited the major factors you discussed, which are alcohol (uric acid is alcohol metabolite in liver) and purine foods (red meat). Youve treated this patient for an acute attack and he is back with more episodes. Now what? Decide if he is an under secreter (give probenecid to cause more urinary excretion) or over producers (give allopurinol to reduce production). Another maintenance medication is colchicine; sometimes the patient may get diarrhea. Pseudogout * Similar presentation as gout with arthrocentesis WBC in 25-50,000 range. Crystals are of calcium pyrophosphate. * Pseudogout associations, 4 Hs: hyperparathyroidism (hypercalcemia), hemochromatosis (hyperpigmentation, diabetes), hypophosphatasia (congenital bone disease), hypomagnesemia. * Pseudogout usually occurs in patients older than 50. If you see pseudogout in a younger patient, think about the other diagnoses (4 Hs). Look for electrolyte abnormalities, bone defects, hemochromatosis clinical presentation. * X-rays can show radiodense linear deposits in the articular surfaces or menisci, this is called chondrocalcinosis. * Treatment is the same as gout. Colchicine is a good maintenance medication for recurrent episodes. Degenerative Joint Disease (DJD) * 64yo man comes to you for the evaluation of knee pain. He says he has had right knee pain for many years but recently it has gotten worse. He denies constitutional symptoms and other joint pain, except for his left second and third DIPs. He has noticed stiffness in the morning. On exam you hear crepitations when he moves the joints but otherwise there is no evidence of swelling, warmth, or erythema of the knee. Laboratory testing is unremarkable. So we have a guy with asymmetric poly arthralgia (not arthritis), this is degenerative joint disease (DJD).

* DJD also known as osteoarthritis (OA), but that is not a very good name for the disease. * DJD is the most common form of arthritis, even though it is not truly an inflammatory joint disease. * DJD found in the joint cartilage, versus RA which affects the synovium. * Disability of patients with DJD is very common. Predisposing factors are increasing age (more use). * Most commonly affected joints are those used the most, such as hips and knees. This can depend on occupation as well, for example a manual laborer may have their hands affected. * DIP joint nodes are Heberden nodes. PIP joint nodes are Bouchard nodes. * DJD most commonly occurs as an isolated condition due to overuse of joints. * Since DJD is a wear and tear disease, it is commonly seen in older patients. However, it can be seen in anyone who has extensive use of their joints such as an athlete. * Morning stiffness in DJD is short, typically lasting less than 30 minutes in the morning. * There is no medication currently to treat the underlying degenerative process. The disease is progressive. * Treatment is aimed at symptom relief. Acetaminophen, aspirin, NSAIDs are effective. * Any patient with DJD who is taking over-the-counter medication should be assumed to be on acetaminophen, aspirin, and/or NSAIDs like ibuprofen or naproxen. * Acetaminophen has no useful anti-inflammatory effect. That is fine in DJD because this is not an inflammatory - 90 disease. The analgesic effect is what we want, and acetaminophen is safer in regards to renal effects and ulcers. * Osteoarthritis/degenerative joint disease by definition does not give a systemic inflammatory response. * X-ray can show joint narrowing and osteophytes (bone spurs) in any joint involved including spine, hands, knees. * Treatment for painful joint should include rest, ice (not heat), and mobilize the joints early to help prevent muscular atrophy. Treatment can also include physical therapy. * Treatment can include surgical joint replacement of the knee or hip, a common operation. Indicated when the patient has intractable pain from their arthritis that cannot be relieve with standard therapies or they are severely disable in terms of function or loss of quality of life. Polyarteritis Nodosa (PAN)

* 45yo complains of headaches for several months and worsening abdominal pain after food. He also complains of numbness in his foot and weakness with dorsiflexion. Physical exam shows hypertension, diffuse mild abdominal tenderness, and loss of sensation in the right lower extremity with weakness and loss of deep tendon reflexes in that region. This is a mononeuritis (one nerve) but multiplex (motor and sensory). There is hypertension and abdominal involvement. What is the diagnosis? Answer is polyarteritis nodosa (PAN). * Look at urine for RBCs and protein as PAN patients can get glomerulonephritis. * PAN is inflammation of medium-caliber blood vessels, with stricturing and beading (dilatation). * Serology can be positive ANCA (protoplasmic p-ANCA). * Treatment is steroids and immunosuppressive agents if necessary. Churg-Strauss Syndrome * Young person with non-productive cough, wheezing, normal chest x-ray. Serum blood counts show eosinophil counts of 32%, sputum shows many eosinophils. What diagnosis is this? This is Churg-Strauss syndrome. * Churg-Strauss affects medium sized vessels. * Serology can be positive ANCA (protoplasmic p-ANCA). * Churg-Strauss is a profound eosinophilic picture with bronchospasm. * Treatment is steroids. Wegener Granulomatosis & Goodpasture Syndrome * Patient has several episodes of hemoptysis, recurrent episodes of severe sinusitis, and now has developed hematuria. What is the diagnosis? Answer is Wegener granulomatosis. * Wegener granulomatosis affects small vessels. * Biopsy of the lung or kidney will show vasculitis and granulomas. * Treatment is steroids and often cyclophosphamide (potent immunosuppressant). * Every time Wegener is an option, you will see Goodpasture syndrome as both have hematuria and hemoptysis. But, Goodpasture syndrome does not have upper respiratory involvement. Biopsy and serology are different. * Wegener serology can be positive ANCA (classic c-ANCA). * Goodpasture serology is for anti-glomerular basement membrane (anti-GBM). Henoch-Schnlein Purpura * Patient presents with non-blanching purpura all over their body. Platelet count and function are normal. The patient complains of abdominal pain with occasional diarrhea. Biopsy of purpura shows small vessel vasculitis. What is the diagnosis? Answer is Henoch-Schnlein purpura. * Typically seen in young adults or kids. * Can involve the kidneys with renal insufficiency.

Takayasu Arteritis * Young woman tells you that recently she has arm pain particularly when she uses it for activity. The pain is worse when she raises it. You examine the arm and the pulse seems diminished compared to the other arm and femorals. Sedimentation rate is very high. This is Takayasu arteritis. * Takayasu arteritis affects the large vessels/branches of the aorta. * It is most often seen in young women. Sed rates are very high. * Diagnosis made with angiogram, showing narrowing of the take-off of various branches of the aorta. * Treatment is steroids. Temporal Arteritis & Polymyalgia Rheumatica * 65yo with fatigue, weight loss, low grade fever for several weeks. Now they have a bad headache. * Any older person with constitutional symptoms who develops a new bad headache, think temporal arteritis. * This is not a common disease but it is one you cannot afford to miss. * Diagnosis is made with biopsy of the temporal artery. This is a large vessel vasculitis. * Treatment is steroids (e.g. prednisone). * The complication we worry about is sudden onset of blindness. So, place patient on high dose steroids at the time - 91 of suspicion and schedule them for a temporal artery biopsy. Do not wait for the biopsy results. * Other artery branches can be involved, thus another prominent symptom is jaw claudication with chewing. * Temporal arteritis is associated with polymyalgia rheumatic. It is pain and stiffness of proximal muscles. Mnemonic is PolyMyalgia and PM for Proximal Muscles. * Quadriceps biopsy of patient with polymyalgia rheumatic would not show inflammation. EMG would not be abnormal, no evidence of muscle inflammation (no elevated CPK or aldolase); these things you would find in polymyositis. * Polymyalgia rheumatic diagnosis is made clinical, often but not always associated with temporal arteritis. Always associated with a very high sed rate. * Treatment of polymyalgia rheumatic is steroids. * Polymyositis is associated with dermatomyositis and the presence of an internal malignancy. * Small vessel vasculitis is Wegener and Henoch-Schnlein. Medium vessel vasculitis polyarteritis and Churg-

Strauss. Large vessel vasculitis is temporal arteritis and Takayasu. -----------------------------------------------------------------------------------------------------------------------------------------Kaplan Videos (2001) Cardiology with Dr. Conrad Fischer, MD -----------------------------------------------------------------------------------------------------------------------------------------Chest Pain * 47yo woman comes to your office complaining of substernal chest pain. The pain has been going on for months. It is intermittent and exertional, sometimes when she goes up two flights of stairs, sometimes at rest, sometimes she goes up three flights of stairs and nothing happens. She also has nausea and vomiting. She has not past medical history. What is the most likely diagnosis? Answer is gastroesophageal reflux disease. * What is the most common cause of non-cardiac chest pain? Gastrointestinal problems like gastritis, esophagitis, duodenitis, peptic ulcer disease, reflux disease, etc. * What is the best initial diagnostic test for her? Answer is ECG. Why not pH monitor or upper endoscopy? To rule out the most dangerous thing first. * Which of the following is the most accurate test for her? Well then it might be pH monitor or upper endoscopy. * 47yo woman now suddenly remember she has had diabetes since she was a child and has been taking insulin for 30 years. She also remembers hypertension and hyperlipidemia with low HDL. She forgot to mention she smoked three packs of cigarettes a day. Oh yea also most of her family members had heart attacks in the 40s and she is a high-stress type A person. Now, what is the most likely diagnosis in this patient? Answer is ischemic heart disease. * Correcting which of the risk factors will improve her long term survival? Answer is correcting all of them. Correcting which of the results in the most immediate improvement in outcome? Tobacco smoking. But what kind of question is that, its not like a managed care plan would only allow for you to correct a single problem so you would need to choose the best one to correct. In real life you try to fix all of the risk factors. * What is the most common risk factor for coronary disease and myocardial infarction? Hypertension. What is the most common cause of hemoptysis? Bronchitis. Not TB, mitral stenosis, cancer, Wegener, Goodpasture, etc. What is the most common cause of atrial fibrillation in the United States? Hypertension. * Smoking related deaths are about 2/3 COPD and 1/3 lung cancer.

* In real life, you can have an MI with no chest pain and with a normal ECG. But for a board exam, if they want you to know an answer they have to tell you something. Otherwise no one can know the answer. 4% of the time myocardial infarction pain is pleuritic. For boards, that is too small a chance so if a patient has pleuritic pain on a board exam question it is not an MI. * Pleuritic chest pain (worse with respiration) causes include viral pleurisy (i.e. pleurodynia, Devils grip, Bornholm disease), pneumothorax, pneumonia, pericarditis, pulmonary embolism. They all start with Ps. * Pneumothorax and pulmonary embolism are different from the other pleuritic chest pain in that they are associated predominantly with shortness of breath. Patient complains of not being able to breath, with a little chest pain. Myocardial infarction patient complains of crushing chest pain, with a little shortness of breath. * Myocardial infarction chest pain changes with position 4% of the time. Most of the time, it is not positional. Pericarditis chest pain is positional, better when leaning forward or worse with leaning back. * Chest wall is non-tender in myocardial infarction. Chest wall is tender in costochondritis (Tietze syndrome). * Anything that can give pleuritic pain can simulate shortness of breath. The patient is splitting and does not want to take a deep inspiration. * Fever is not specific enough to differentiate between causes of chest pain. Fever seen in infections and inflammation like pneumonia and pleuritis as well as others like MI, PE, pneumothorax (any form of atelectasis). * Any clot or collection of blood can give fever; like DVT clot, MI clot, hemarthrosis in the knee, subdural hemorrhage, subarachnoid hemorrhage, ruptured ectopic pregnancy. - 92 * Sympathetic outflow (diaphoresis, pain, tachycardia, pale, cold and clammy extremities) does not help you specify between causes of chest pain. * Sense of impending doom and Levine sign also do not help you determine the cause of chest pain. * Most common cause of death in the United States is myocardial infarction. For every 100 people who go to the hospital with chest pain, 50 have a non-cardiac problem, 10 have unstable angina, < 10% have an MI.

* Therefore, you must have the ability to distinguish between the causes of chest pain. They are largely based on positional changes, pleuritic pain, tenderness on chest wall. Positional, Pleuritic, Tender. * How is it that an attending can know about a list of patients while you can only remember information about 2-3 as a new student? It is because they are not listening to most of what you say. They are not smarter than you, they just know what is important to listen to. The attending knows the important symptoms that distinguish between diseases and also knows the most appropriate management. * In terms of determining the diagnosis, which is the most important: risk factors or individual presentation? Answer is individual presentation. Even with a history that includes every risk factors, it is not as important as presentation of positional, pleuritic, and tender. Profiling is less accurate. Stress Testing & Coronary Artery Disease Management * 47yo lady with intermittent chest pain, what is the best initial diagnostic test? Answer is ECG. No matter what happens, you want to rule out the most dangerous thing first. * Say the ECG is normal. Now what will you do for her? Answer is cardiac stress test. * Reasons to stop a stress test include chest pain, hypotension, shortness of breath, lightheadedness. * Major measure of sufficient exercise (test adequacy) is heart rate. Patient must be able to exercise, doctor must be able to read ECG: look for ST segment depression (ischemia, not infarction). * What if patient has baseline ST segment changes, like hypertension (left ventricular hypertrophy) or drugs (patient on digoxin) or pacemaker spike or LBBB? Then do a stress echo or stress thallium test, both have similar indications. Thallium is injected or an echocardiogram done right after walking on treadmill. * Stress echo should look for dysmotility or hypokinesis (decreased wall motion); the ischemic portion does not move well during ischemia. If infracted, you see akinesis (no wall motion). * Thallium looks like potassium to the cardiac myocyte. If you are a myocyte in the heart, you will pick up potassium because of the sodium-potassium ATPase. If there is poor perfusion, the capillary cannot delivery the thallium. An ischemic or infracted area will have decreased thallium uptake. How do you tell the ischemia from the infarct if they both have decreased thallium uptake? You re-scan 4 hours later. The scan will be the same 4 hours

later if there is infarct. If there is ischemia, the area will reperfuse showing thallium uptake. * 47yo lady is morbidly obese, has claudication with peripheral vascular disease, diabetic foot ulcers, dyspnea at rest due to COPD, and one leg. Can she do an exercise stress test? Nope. No need to memorize the list, just think of things that make it so you cannot exercise. You could memorize digoxin, LHV, pacemaker for abnormal baseline ECG, but what if your test has procainamide (causes ST down scooping)? Or some other drug/condition that was not in a list to memorize? Understand the basic question, is there a baseline abnormality to the ECG such that I would need to order an echo or thallium stress test? * Dipyridamole (Persantine) thallium stress test used when patient cannot exercise. Also called a chemical stress test. Or, dobutamine echo stress test can be used when patient cannot exercise. Both have similar indications. * Maximum heart rate = 200 - age. * Sub-maximal stress test should reach 85% of maximum heart rate: 85% of (220 age) * Exercise tolerance decreases as you get older, just as indicated in the equation. * Dipyridamole thallium stress test abnormalities found by looking for decreased thallium uptake. Dipyridamole dilates coronary arteries, so more thallium should be picked up normally. * Dobutamine echo stress test abnormalities found by looking for decreased wall motion. Dobutamine increases contractility and can provoke ischemia, so increased wall motion normally. * Real exercise is always better than simulated exercise. * Bicycle ergometry (hand bicycle) is not enough exercise to get the heart rate up. There is much more muscle in the thighs than arms in nearly all the population. * 47yo lady gets an exercise stress test and it is positive. What is the next best step? As a side note, what does this question mean? Youre being asked what is the next step in management, so diagnosis or treatment. If you were asked next step in diagnosis then pick angiogram, or best diagnostic test then pick angiogram. * Best next step in management for patient with chest pain and ischemic heart disease seen on stress test is medication. Give aspirin (lowers mortality), nitrates (no change in mortality), beta blockers, ACE-I (if left ventricular dysmotility, congestive failure), calcium channel blockers (only if patient cannot tolerate beta blockers as beta blockers can decrease mortality and calcium channel blockers do not).

- 93 * What is the most accurate test for any disease? Autopsy. * How to you differentiate Prinzmetal angina (from unstable angina)? Angiography, not history, not ECG. Angiography shows clean coronary arteries for Prinzmetal angina. * 47yo woman with positive stress test and aspirin allergy. What do you give? Clopidogrel or ticlopidine. * Nitrates and digoxin do not lower mortality. Hyperlipidemia * When determining if a patient should get anti-lipid medications, do you look at LDL, HDL, total cholesterol, triglycerides, VLDL, or waist circumference? Answer is LDL. We do not know if triglycerides are associated with disease as well as we know that LDL are related to disease. * 47yo woman with chest pain being treated and LDL is 191. What do you do next? Answer is statins, not cholestyramine, niacin, or fibric acid derivatives. Statins are first. All of those drugs lower total cholesterol and all raise HDL. All lower triglycerides as well. Statins are first because they lower mortality. * Which raises HDL the most? Niacin. Which lowers triglycerides the most? Fibric acids like gemfibrozil. * 47yo woman with positive stress test but no risk factors and LDL is 171? Answer is statins. 141? Statins. What is most important, risk factors or individual presentation? Again, individual presentation. She has disease. Once you have disease, it does not matter how many risk factors you have. * Diet and exercise only prevent the disease. So when disease is present, diet and exercise is not primary treatment. * Risk factors <=1, start diet at 160 LDL and drugs at 190 LDL. * Risk factors >=2, start diet at 130 LDL and drugs at 160 LDL. * Disease, start diet at 100 LDL and drugs at 130, get LDL < 100. * Patient with coronary artery disease (or MI, angina, unstable angina) who has LDL > 130, start statin. CABG & PCI * 47yo woman with positive stress test, why do angiography? To determine if she needs a coronary artery bypass graft (CABG) or percutaneous coronary intervention (PCI) with medications. * For every 100 people who undergo CABG, 1 person dies; Surgical mortality is 1%, which seems small when it is someone elses mortality. Its better to be alive with chest pain. * Bypass if severe disease, meaning 3 vessel disease or left main affected. * PCI (angioplasty, stenting) if 1 or 2 vessel disease.

* This is why you hear he had triple bypass or she had quadruple bypass or David Letterman had quintuple bypass. You do not hear yea, he had single bypass. * CABG is not done for single or double vessel disease because there is no benefit in mortality. Let the mortality difference drive you. Unstable Angina & Myocardial Infarction Management * 64yo man with a history of hypertension and hyperlipidemia comes to the ED with chest pain. The pain goes to his neck and arm, lasting for the past hour. He is diaphoretic, short of breath, S4 gallop, Levine sign, sense of impending doom. You wont be asked diagnosis (unstable angina) or what test to order (ECG). ECG shows ST segment depression in V2-V4. What is the next step in management? Give aspirin and beta blockers. * What if the patient is allergic to aspirin? Give clopidogrel or ticlopidine. * Would giving oxygen help? Not really, it does not remove the thrombus from the heart and has not been shown to significantly improve mortality. The patient is not hypoxic in the sense of low arterial PO2. * Thrombolytics do not lower mortality either. * What else should the man having a possible MI get? Nitrates, although they do not lower mortality they help relieve chest pain. Aspirin and beta blockers reduce mortality. Oxygen also given as well as morphine (analgesic) but they are not the most important part. Mnemonic is MONA for morphine, oxygen, nitroglycerine, aspirin. Only aspirin matters there. Better mnemonic is MONAB for morphine, oxygen, nitrates, aspirin, beta blockers. * This is unstable angina because the patient has ST depression. ST elevation is the best you can do to tell if there is a myocardial infarction now. * ACE-I only helpful if CHF or decreased left ventricular function. * Calcium channel blockers helpful when you cannot use beta blockers or patient has Prinzmetal angina. * What other medication lowers mortality in addition to the beta blockers and aspirin? Answer is heparin. * In a myocardial infarction, a clot has formed and completely occluded the artery. * In unstable angina, a clot is forming and has not occluded the artery, although it is working on occluding. * Heparin prevents clots from forming. Thrombolytics do not prevent clots from forming, they lyse clots. * Heparin is useful for unstable angina. Thrombolytics is useful for myocardial infarction.

* How can you tell now to give heparin or thrombolytics? Answer is ST elevation or not. * If ST segment elevation present, give thrombolytics. If no ST segment elevation, give heparin. - 94 * Beside thrombolytics or heparin, the management is the same for unstable angina and MI. * For MI, thrombolytics dissolves the clot then give heparin afterwards to keep it open. * 64yo man with a history of hypertension and hyperlipidemia comes to the ED with chest pain. The pain goes to his neck and arm, lasting for the past hour. ECG shows ST segment depression in V2-V4. He has melena, a subdural hematoma, epidural hematoma, subarachnoid hemorrhage, petechiae, heavy periods. Now what do you do? * What do you do different when you have a major contraindication to thrombolytics and heparin? Answer to open the vessel is do angioplasty. * Glycoprotein IIb/IIIa inhibitors are abciximab, eptifibatide, and tirofiban. These are antiplatelet drugs and can make you bleed, thus you would not give these in an acutely bleeding patient. * Most common cause of death immediately post-MI is arrhythmias. Prophylactic lidocaine suppresses those arrhythmias, so why not use lidocaine? Because it does not change the mortality. Lidocaine also causes arrhythmias. * All patients who get angioplasty also get a stent to keep the vessel open. Atheromatous plaque gets covered by a little later of endothelium. During times of exertion, the plaque ruptures which causes a sudden clot to occlude the artery. Angioplasty is when a catheter is placed through the partially occluded artery then a balloon is expanded to open the vessel. The problem is when you remove the catheter the plaque can bulge back out. Thus you leave a meshwork stent scaffolding to keep it expanded. * CK-MB and troponins do not begin to arise for 4 hours. If they do not change management, why order them? If the CK-MB is up with ST segment elevation, you will still give aspirin, beta blockers, thrombolytics. If CK-MB not up, maybe it is just not up yet since they begin to rise at 4-6 hours and take 12-24 hours to reach peak. * Also, thrombolytics can be given up to 12 hours from the onset of chest pain (3 hours for stroke).

* CK-MB last 1-2 days, troponins last 1-2 weeks. CK-MB (97-99%) is specific, but troponins are even more specific (99.9%). Troponin I is the most sensitive and specific. Uterus can still make a little CK-MB but no troponins. * LDH is no longer used in the diagnosis of myocardial infarction. LDH is not abnormal from 12-24 hours. * When a patient comes in with chest pain and ECG abnormalities, do not wait around for enzyme changes. The mortality benefit with thrombolytics is enormous in the first 1-2 hours, up to a 50% reduction in mortality within the first hour. Overall reduction in mortality with thrombolytics is 25%. * Efficacy is the same with tPA (tissue plasminogen activator) and streptokinase. Use tPA if patient previously got streptokinase, because streptokinase can produce antibodies resulting in anaphylaxis. Post Myocardial Infarction Management * 65yo man in the coronary care unit (CCU) after having an inferior wall myocardial infarction last night. He received all the appropriate therapy. You are called by the nurse because he is confused. Patients blood pressure is 70/40, pulse is 40, has cannon a waves, a chest that is clear to auscultation. Heart exam reveals 3/6 murmur. Which of the following is the most likely diagnosis? * Right coronary artery feeds the right ventricle, AV node, and inferior wall. 40% of inferior wall MIs also have RV infarcts. This is not the right answer though. * Myocardial wall rupture with cardiac tamponade would lead to hypotension and confusion. But no. * Think about what it could be. We could go to sermons all day long to become saints, but it doesnt work that way. * Valve rupture with chordae tendineae rupture (after a week) could cause a murmur and hypotension. Nope. * What about extension of the MI with cardiogenic shock? Not that either. * Cardiogenic shock would give rales. Valve rupture would regurgitate into the lungs, causing rales. This patient has a clear lung auscultation exam. RV infarct and cardiac tamponade could give clear lungs, but the heart would beat faster to compensate (here the patient has bradycardia). * Complications of an MI can all cause hypotension and hypotension can cause confusion. * Answer is third degree complete AV heart block, described first in 1826 by Stokes and in 1846 by Adams near the University of Dublin. Wait, they didnt have ECG back then. Stethoscope invented shortly before by Laennec in

France (1816). They saw lightheadedness, syncope, cannon a waves, bradycardia. No hypotension because the blood pressure cuff did not exist then. * What is the best initial diagnostic test? ECG, showing complete heart block. * What is the best initial management? Atropine. This blocks the parasympathetic, allowing the sympathetics to work without inhibition and thus speeds up the impulses through the AV node. It is like a car that has an accelerator (sympathetics) and brakes (parasympathetics). Atropine cuts the brake lines for a while. * Transcutaneous pacer makes the heart beat with capture, but it also makes all the chest muscles contract. * Transvenous pacing cannot be setup fast enough and requires an invasive procedure. * If cardiac tamponade, do a pericardiocentesis or a cardiac window then repair the hole. * If valve rupture, may need to go in and emergently replace the valve. * If cardiogenic shock and extension of the MI, may need emergency bypass surgery. - 95 Congestive Heart Failure (CHF) * 67yo woman with a history of MI, valve disease, hypertension, adriamycin use, radiation to her heart and chest, Chagas disease, beri beri, and alcoholic cardiomyopathy. She comes in while chowing down on an entire pizza. Presentation is dyspnea with rales to apices, JVD to ears, S3 gallop, edema to waist, enlarged liver, enlarged spleen, ascites, and hemorrhoids. She says she has shortness of breath while laying flat but feels better when she goes to the window and puts her head outside (what makes her feel better in reality is just getting up). You will not be asked the diagnosis here, which is CHF in its worst form called pulmonary edema. * What is the most common cause of developing congestive heart failure in the United States? Post-MI. Myocardial infarction killed off part of the heart, so you have a weak leaky heart. * There are many causes of CHF and the initial diagnostic tests and management are all the same. * There needs to be an event causing (acute) pulmonary edema in a patient with underlying CHF. Common causes are stopping medications, increased salt load, myocardial infarction (more ischemia), arrhythmia. Wall motion gets worse when your heart becomes ischemic, worsening the congestive failure. * How much cardiac output is based on atrial contribution? Answer is 10-20% in normal person. So you can live

without the atrial kick. But that is because your heart is not weak (cardiomyopathy), dead (post-MI), leaky (valve regurgitation). These patients need atrial contribution. * If you have congestive failure and you develop atrial fibrillation, now you may have lost 40-50% of your cardiac output because you need the atrium to push blood forward. * Valvular rupture occurring a week after an MI would cause regurgitation leading to acute pulmonary edema. * For this 67yo patient, what is the next best step in management? Answer is oxygen then loop diuretics. * Patient will not die from lower extremity edema or organomegaly. They die from pulmonary edema. This patient has lungs filled with fluid and thus is not oxygenating. The fastest was to get fluid out of your body is diuretics. * ACE-I will decrease mortality in the long-term, but they will not help acutely. Getting fluid out of the lungs now will help reduce preload. Preload reduction will save this patients life, give loop diuretics. * Many years ago before IV furosemide was around physicians did phlebotomy and rotating tourniquets. * Loop diuretics are used because they are intravenous and work fast on the ascending loop of Henle. Loop diuretics are furosemide, bumetanide, ethacrynic acid, torasemide. * Nitrates also work at pre-load reduction because they dilate both arteries and veins, but dilate veins more. * Morphine can reduce pre-load as well. * 67yo woman with CHF and now acute pulmonary edema. Give her medication for her shortness of breath and to reduce preload: oxygen, loop diuretics, nitrates, morphine. 80-90% of patients will get better with this therapy. * CXR would show enlarged heart with congestion, pulmonary vascular redistribution, effusions, Kerley B lines, but we know all this. A CXR does not change our management in this patient. Patient comes in short of breath saying they feel like they are filled up with fluid. You do a CXR and tell her that her lungs are filled with fluid. Thats great doc, but she already knew that and she doesnt care. What are you going to do to help her is the question. * ABG would show hypoxia and low CO2 (hyperventilating). But you know this, it does not change management. * Say you give this patient oxygen and preload reduction but she is still short of breath. This is dilated cardiomyopathy, also known as systolic dysfunction. The goal now is to decrease afterload.

* Why not use digoxin now since it is a positive inotrope? It takes time, weeks to fully take effect. * Dopamine and dobutamine are both positive inotropes. Which do you give for this patient though? Dobutamine. They will both increase contractility (positive inotropes) and myocardial oxygen consumption. * Dopamine increases the afterload. Dobutamine decreases the afterload, and thus does not change blood pressure. * Dopamine is only used if you need to raise the blood pressure. It is only used as a pressor here. * Afterload reduction is best achieved with ACE-I (-pril drugs). Most common adverse reaction to ACE-I is cough and hyperkalemia. If you have an intractable dry cough, give an ARB (-sartan drugs, like losartan or valsartan). * ARB drugs are angiotensin II receptor blockers. * Most accurate way to measure ejection fraction is MUGA scan. * Catheterization and angiogram is less accurate, but more accurate than an echocardiogram. * The least accurate method to assess ejection fraction is an echocardiogram. Echo is the best initial method of measuring ejection fracture. They are cheap, clean, easy, but not the most accurate test. * MUGA scan is a nuclear ventriculogram (radionuclide ventriculography). Nuclear isotope is injected into the body and the scan measures the amount of nuclear energy emitted from the heart in diastole and systole. The difference in this measurement corresponds to the ejection fraction, stroke volume to be precise. Since it is nuclear energy, it is not based on a cross sectional diameter like an echo or catheterization. You could look at a soda bottle and take a cross sectional diameter to get an estimate of the amount of soda inside. But the better way is to pour fluid into the bottle and measure how much actually comes out, and that is what a MUGA scan is about. - 96 * 67yo lady with pulmonary edema and CHF was treated with oxygen, furosemide, nitrates, morphine. She feels all better the next day, what is the most important medication to discharge her on? Answer is ACE-I, diuretics, and beta blockers. Sometimes you may need digoxin. * ACE-I can raise potassium and help offset diuretics. You give potassium if the patient is only on diuretics and

digoxin, as the diuretic would make you hypokalemic and the digoxin would then become toxic. * Diuretics and digoxin have not been show to lower mortality. ACE-I has been shown to improve mortality. * Aspirin has been shown to decrease mortality in coronary disease but not congestive failure. * But wont beta blockers worsen ejection fraction? Actually, they increase ejection fraction and cardiac output. Basic sciences in medical school will tell you over and over that beta blockers are negative inotropes that worsen ejection fraction and worsen cardiac output. That was because beta blockers had not been studied in congestive failure because everyone thought it would make things worse. * There is an interesting thing that happens to patients with congestive failure who are on beta blockers and they like it quite a bit. That thing is that they do not die. * Beta blockers lower mortality more than ACE-I in congestive failure. * What is the bigger problem in cardiac failure with systolic dysfunction, the weak heart or the ischemia? Ischemia, and that is why beta blockers are good as they lower heart rate and cardiac contractility, because they are antiischemic. Going back to stress testing, what is the major determinant of exercise? Heart rate. When you lower the heart rate you reduce ischemia and the patient is less likely to die. Valvular Heart Disease * Valvular disease is organized into stenosis and regurgitation, more than by valve location. * Fundamental symptom of all valve disease is CHF (rales, S3 gallop, edema). * The ease of availability and use of the echocardiogram has led to a decline in our ability to auscultate murmurs. The echo has greater sensitivity and specificity than the stethoscope. * What is the most common cause of mitral stenosis? Rheumatic fever, but it can cause any valvular disease. * What else causes mitral stenosis? Congenital, but it can cause any valvular disease. * Calcification can cause mitral stenosis and aortic stenosis, but not a regurgitant loose/floppy lesion. * Infections (e.g. endocarditis) can cause regurgitant lesions from vegetations, but not stenotic lesions. * When your heart dilates, the valve leaflets do not change in size. Anything that dilates the heart has to cause regurgitation because the leaflets separate. Infarction can cause regurgitant lesions through this mechanism. * Rare aortic regurgitation causes include ankylosing spondylitis, reactive arthritis, syphilis, Marfan syndrome,

Ehlers-Danlos, HLA-B27 stuff. These all added together and multiplied by 10 are still not even close to causes of aortic regurgitation compared with myocardial infarction and dilation of the heart. * Mitral stenosis is the most likely valvular disease to have hemoptysis, atrial fibrillation (with emboli to brain), and dysphagia. Dysphagia because the enlarged left atrium (most posterior heart structure) presses on the esophagus. There can also be hoarseness as the atrium presses against the recurrently laryngeal nerve. * Aortic stenosis is associated with syncope, CHF, and angina. Mnemonic: SAD for syncope, angina, dyspnea (CHF). Most common symptom of aortic stenosis is angina, because the ostia (entry points) for the coronary arteries are distal to the aortic valve. The stenotic aortic valve blocks blood flow into the coronary arteries. Also, coronary artery disease is common in patients with aortic stenosis (older men), thus another cause of angina. Worse prognosis is with CHF, 1-2 years average until death. Syncope prognosis is 3 years, angina prognosis is 5 years. * What can be done to restore a heart to normal shape and size once it dilates? Nothing, heart transplant needed. Cardiomyoplasty, slicing a piece of the dilated heart out, does not work (e.g. Batista partial ventriculectomy). The actin and myosin filaments have become separated and cannot contract well. When your butt gets really big and your underwear stretches, do you cut out a piece of underwear to fix the size? No, you need an underwear transplant. * ACE-I may stop the dilation problem from getting worse, but they do not alter anatomical heart size. * Since nothing can be done after the heart dilates, if you plan to replace the valve you should not wait for the heart to dilate up in size. If you wait, it will be too late. * Exotic findings for aortic regurgitation are largely archaic, like Watson water hammer pulse, Corrigan pulse (rapid upstroke and collapse of the carotid artery pulse), de Musset sign (head nodding in time with the heart beat), Quincke sign (pulsation of the capillary bed in the nail), Traube sign (a double sound heard over the femoral artery when it is compressed distally), Duroziez sign (systolic and diastolic murmurs described as 'pistol shots' heard over the femoral artery when it is gradually compressed). Dont forget the even less common Lighthouse sign, Landolfi sign, Becker sign, Mller sign, Mayen sign, Rosenbach sign, Gerhardt sign, Hill sign, Lincoln sign, Sherman sign,

and Ashrafian sign. None of these have utility in making a diagnosis. Hearing a diastolic murmur is more reliable. * These exotic findings are usually only seen in longstanding non-treated disease. We replace the valve before these patients join the American Iatrogenic Association. - 97 * Initial test of choice for any valvular disease is echocardiogram. * Cardiac catheterization is the only way to get direct pressure readings for any valvular disease. * Transthoracic echo (TTE) or transesophageal echo (TEE) first? Answer is TTE first. Which is best? TEE. * Mitral stenosis treatment includes preload reduction (diuretics, salt restriction). * 27yo pregnant woman with much worse symptoms from her mitral stenosis. Why? Plasma volume goes up but the size of the mitral valve does not. She delivers here baby but still has bad shortness of breath despite diuretics and salt restriction. What is the next best step in the management of this patient? Balloon valvotomy. * ACE-I help get blood out of the ventricle, and we do not have a problem with that in mitral stenosis. Digoxin will not help because it helps blood squeeze out of the ventricle. Digoxin would help with atrial fibrillation because it helps decrease the heart rate and ventricle fills during diastole. * Commissurotomy would help increase the size of the stenotic hole. But this is very invasive. * The ultimate step in management for all valvular heart disease is replacement of the valve. But what can you do in the meantime before replacing the valve, that is the question. How do you get the valve into the heart? Open heart surgery in general with median sternotomy. If the porcine valve needs to be replaced every 10 years and we have a 27yo pregnant female here, is valve replacement at the top of your list for management? Lets hope not. * With balloon valvotomy a vascular catheter is threaded across the valve and a balloon dilates, opening the valve. * With aortic stenosis, balloon valvotomy does not work well. Valve replacement with vessel bypass is the therapy. * Aortic stenosis calcifications are from calcium, thus balloons do not work as well. * Mitral regurgitation and aortic regurgitation are managed with the same medications used for CHF. Treatment is preload reduction, positive inotrope, and afterload reduction. ACE-I and vasodilators are the most important. This

makes it easier for blood to get out of the heart. Treatment is valve replacement if symptoms persist. Cardiac Auscultation * Auscultation locations are APTM (all physicians take money), aort ic at upper right sternal border, pulmonic at upper left sternal border, tricuspid at lower left sternal border, mitral at lower left midclavicular area. * Valsalva maneuver decreases preload. Exhalation against closed glottis, which increases intrathoracic pressure and decreases venous return to the heart. Have patient bear down like they are having a bowel movement is another way. * Squatting maneuver increases preload. Leg veins are compressed pushing blood up into the heart. * Standing suddenly decreases preload, blood pools in the legs so less blood in the heart. * Leg raising increases preload. Blood is brought away from the legs and into the heart. * Right atrial pressure when sitting is zero, when standing it could be almost subatmospheric, 5mmHg laying flat. There is no pressure pushing blood back into the heart, blood basically just falls into the right atrium. If you increase the intrathoracic space pressure (valsalva) it pushes blood out of the thorax. * Mitral stenosis (MS): opening snap moving closer to S2 as stenosis worsens because left atrial pressure increases so it pushes the valve open earlier. Murmur will increase with more blood in heart (leg raise, squatting). Decreases with preload reduction (standing, Valsalva). * Aortic stenosis (AS): crescendo-decrescendo murmur, upper right sternal border. Murmur will increase with more blood in heart (leg raise, squatting). Decreases with preload reduction (standing, Valsalva). * Mitral regurgitation (MR): S1 and S2 are obscured by constant murmur, pansystolic or holosystolic murmur. Decreases with preload reduction (standing, Valsalva). * Aortic regurgitation (AR): decrescendo murmur because blood gets shot up out of the aortic valve and you do not hear anything, then you hear the blood crashing down on the ventricle. Decreases with preload reduction (standing, Valsalva). * Do not give beta blockers to AS, MS, AR, MR because it would make the murmur/problem worse. * Hypertrophic obstructive cardiomyopathy (HOCM): crescendo-decrescendo murmur, lower left heart apex. Most

common presentation is dyspnea. It is the most common cause of sudden death in healthy young athletes. Unlike the other murmurs of the left heart, there is more murmur with less blood and less murmur with more blood. Asymmetric septal hypertrophy is in the way, so anything that makes the heart larger (more blood) pushes away the obstruction and decreases the murmur. When a patient exercises, the heart contracts more completely causing more obstruction. When dehydrates, the heart empties more fully causing more obstruction. Digoxin, diuretics, and ACE-I would thus cause more obstruction. Do not give HOCM or MVP patients diuretics. Best initial therapy for HOCM is beta blockers because it decrease the heart rate thus more filling and thus less obstruction. * Occasionally a myomectomy is needed for HOCM to remove the obstruction. * Mitral valve prolapse (MVP): mid-systolic click due to leaflets stopping short. Most common cause is congenital. Presentation is most commonly pain (atypical chest pain), palpitations, panic attacks. Mnemonic: Prolapse, Pain, Palpitations, Panic attacks. Initial therapy is beta blockers. * Note treatment and maneuvers are the opposite in HOCM and MVP compared with AS, MS, AR, and MR. - 98 * If a diuretic is used to treat a murmur (preload reduction), then a Valsalva will make the murmur less intense by the same concept of preload reduction. Thus all left sided murmurs get more intense with more blood (leg raise, squatting) and less intense with less blood (Valsalva, standing). Exceptions are HOCM and MVP. Restrictive Cardiomyopathy, Pericarditis, & Pericardial Tamponade * Beta blockers are not used in restrictive cardiomyopathy because it is not hypertrophic. ACE-I and diuretics are not used because it is not dilated. * Causes of restrictive cardiomyopathy include amyloidosis (protein accumulation from inflammatory disease), sarcoidosis, hemochromatosis, cancer, and fibrosis. * Causes of pericarditis include tuberculosis, viral (e.g. coxsackie), strep, staph, klebsiella, pseudomonas, rickettsia, cytomegalovirus, toxoplasmosis, anaerobes, Gram negatives, syphilis, yep anything. Viral is the most common. * Inflammatory causes include SLE, scleroderma, rheumatic fever, rheumatoid arthritis, Sjgren syndrome, Dressler

syndrome, uremia, mixed connective tissue disorder, yep any connective tissue disorder or inflammatory disorder. * Trauma causes include open heart pericardiotomy trauma, chest wall trauma, yep stop cause its any trauma. * Say youre in a lecture and someone is listing all the causes of pericarditis. By the 20th item on the list the audience will look like the old Maxell tape ads with the guy in front of the speaker being blown back. What are you going to learn from this? Pretty much nothing. All the attendings and residents who know the lists will love the ever so complete lecture, while the medical students will look at each other and say Uhh, wanna go get a beer? * There are over 50 separate causes of pericarditis. Any major textbook will list these items in completeness, but you can bundle any one of the causes into these main topics. Try to do this for any disease with a long list. * Cancer causes include...yea no point, its any cancer near the heart. Lungs, breast, esophagus, mediastinal lymph nodes are all near the heart. * Symptom of pericarditis is pain that is positional and pleuritic. * ECG shows ST elevations everywhere. Pathognomonic ECG finding is PR segment depression. * Treatment of pericarditis is to correct the underlying etiology. If no etiology (mainly viral), NSAIDs. If NSAIDs do not work, then use prednisone. * Causes of pericardial tamponade are pneumonia, pleuritis, pleural effusion. * What are the causes of pneumonia? Anything, pneumococcus most often. * Which connective tissue disorders have pulmonary involvement? Any connective tissue disorder. * What cancer can affect the lung? The ones that are anatomically near the lung. * Can trauma cause a pleural effusion? Yep. And pulmonary contusion with atelectasis leading to pneumonia. * Causes of pericardial tamponade are the same as pericarditis. Infections, inflammatory, trauma, cancer. * What about peritonitis? Yep, same causes there too. * Chronic pericardial tamponade becomes constrictive pericarditis from fibrosis. * Presentation is jugular venous distension (JVD), tachycardia, and hypotension. But that could be pulmonary edema, so how do you distinguish? Answer is pulsus paradoxus and clear lungs. * First test of choice is echocardiogram. * Treatment of choice is needle pericardiocentesis. * The problem with the pericardiocentesis is that the fluid can keep re-accumulating. So what do you do to treat a

chronic pericardial effusion? Pericardial window, a hole in the pericardium and fluid drips into pleural space. * Constrictive pericarditis symptoms are JVD, ascites, enlarged liver. * What occurs in constrictive pericarditis that does not occur in CHF. Pericardial knock. * CXR or CT for diagnosis. * Treatment is to remove the pericardium. Arrhythmia Management * The type of arrhythmia management that you need to know is the first hour of management, what you would be expected to managed as a physician intern at a hospital. * 47yo man comes to the office seeking advice about diarrhea prophylaxis prior to going to a vacation to Thailand. You notice that he feels alright but his heart rate of 47. What is the next best step in management? This patient does not have symptoms so ECG. If he did have symptoms, give atropine acutely to increase the heart rate then a pacemaker later on. * Side note, pulse rate is not the same as heart rate. Heart rate is seen on ECG, pulse is palpated. * Normal heart rate of 60 to 100 is based on thousands of military recruits between the ages of 18 and 24, two hours post prandial in the supine position. * Sinus bradycardia at a rate of 47 in an asymptomatic man, next step is leave him alone. What if the heart rate was 37 without symptoms and ECG showing sinus bradycardia? Do nothing. It does not matter how low it gets because he has no symptoms. - 99 * First degree AV block without symptoms do nothing. This does not progress to second degree AV block. * Mobitz type I AV block (Wenckebach) without symptoms do nothing. This does not usually progress and is part of the normal aging of the conduction system. * Mobitz type II AV block is the dividing line, this can progress to complete block and ischemia. * Mobitz type II AV block and type III complete AV block get a pacemaker even without symptoms. We do not wait for the patient to have their first syncopal episode while they are driving down the road. * 28yo female medical student who has been preparing for an exam and using the four basic food groups, alcohol,

nicotine, caffeine, chocolate. She comes in complaining of palpitations. ECG shows supraventricular tachycardia (SVT) at a rate of 160. What is the first step in management? * Multifocal atrial tachycardia includes SVT, atrial fibrillation (Afib), atrial flutter (Aflutter). They are all the same in their management except adenosine. * Determine if the patient is unstable. If unstable, do synchronized cardioversion. What does unstable mean though? Hemodynamic instability means systolic blood pressure < 90, chest pain, shortness of breath, or confusion. * Does that mean a person with a normal systolic blood pressure of 88 gets shocked in this situation? Yes. We have to agree on a definition for hemodynamic instability. * Conduction jelly is used to increase the surface area for the shock and to decrease the burn because dry skin burns. * Stable patient atrial arrhythmia (e.g. SVT) first step is vagal maneuvers like carotid sinus massage, Valsalva maneuver, mammalian dive reflex (mostly for kids, ice back to face), circumferential digital rectal examination. * Eyeball pressure does a good job of slowing the heart rate...and blinding the patient, so dont do that. Eyeball massage can cause retinal detachment and it is not needed. * Stable patient with SVT and vagal maneuvers did not work, then adenosine. Adenosine only used for SVT. * If vagal maneuvers and adenosine do not work, then slow the rate. You can give digoxin, beta blocker, calcium channel blocker. How do you know which one to use? These all slow the rate but do not convert the rhythm. The indications for digoxin, calcium channel blocker, and beta blocker in this scenario are the same so it does not matter. * Say patient had a history of asthma? Then do not use beta blockers because it could cause bronchospasm. * Mnemonic for SVT meds: ABCD, adenosine, beta blocker, calcium channel blocker, digoxin. * 72yo former president of the U.S. is at an important dinner in Japan, gets up to make a point, leans forward and vomits all over the Japanese ambassador. He is found to have rapid atrial fibrillation secondary to Graves disease. What is the next best step in the management of this patient? If stable, beta blockers because it will block the effect of the sympathetic outflow from the hyperthyroidism. * Rate controlling meds for non-SVT atrial arrhythmias are BCD: beta blocker, calcium channel blocker, or digoxin.

* Coumadin is used when patient has chronic atrial fibrillation, which means > 48 hours. Because after you have had Afib for more than two days you are at risk for emboli. * 57yo man at the opera who gets up, becomes lightheaded and confused. He is found to have ventricular tachycardia (Vtach). Blood pressure is 128/88. What is the next best step in the management of this patient? * Does he have hemodynamic instability? No, he has lightheadedness and confusion. * Vtach that is stable gets lidocaine, unstable gets defibrillation. * 57yo man at the opera who gets up, falls to the floor and is not moving. What is the next best step in the management of this patient? Answer is determine if he is conscious or unconscious. Determine if the patient is responsive and not just sleeping. * Say patient is truly unresponsive to even painful stimuli. What is the next step in management? Call 911. American Heart Association has spent a great deal of time working the algorithm out. Say we skip this step and open the airway, do a couple of rescue breaths, he has no pulse so we start compressions. Whats the problem now? Well, no one is coming to bring a defibrillator or take this man to the hospital. * Survival without defibrillation declines to zero after ten minutes. After ten minutes without defibrillation, the patient has no good chance of survival. There is a 10% decline in mortality per minute. Thus, you better be sure about the next step of management. Call for help before moving on to the rest of the algorithm. Chest compressions do not restart the heart, they keep you alive while waiting for the important equipment (defibrillator) to arrive. * Say the patient is unresponsive and pulseless. What is the differential? Pulseless ventricular tachycardia, ventricular fibrillation (Vfib), asystole, pulseless electrical activity (PEA). What is the only way to distinguish these? ECG. Thus, you better call for help to get the equipment there so you can distinguish these. * After calling for help, then you do ABCs. Open airway, check breathing, give two breaths if not breathing, check pulse, start compressions if no pulse. * Say ECG shows up and patient is in Vfib with no respirations. Now what? Defibrillation. Not intubation, how long would it take to intubate in the field? Maybe 3-4 minutes with getting equipment out, suction, and get the tube in. In that time, 40% of your patients just died since 10% are lost per minutes.

- 100 * If defibrillation did not work, do a cycle of CPR then shock again. Epinephrine can be given between each cycle as well. So shock, drug, CPR, shock, drug, CPR, etc. After three cycles with refractory VT or Vfib, you can give amiodarone or lidocaine. * Intracardiac medication has not been show to have greater efficacy so it is not an answer. * Thoracotomy with direct cardiac massage is also not an answer, it will not help. * Magnesium is given for Torsades de Pointes only. * Precordial thump is like a little defibrillation, 10-20J of energy. The amount of energy needed for successful defibrillation is little at the very beginning. Thus, precordial thump used in witness arrest only with no defibrillator present. If you have a defibrillator, use that instead. * What about the dosing of these drugs and shocks? Drug dosing is not asked on board exams. Exception may be the advanced cardiac life support (ACLS) drugs here. Epinephrine is 1mg, atropine is 1mg for cardiac arrest and 0.5mg for bradycardia, lidocaine is 1mg/kg, adenosine is 6mg to start, amiodarone is 300mg. * Resuscitation of patients beyond 10 minutes only in hypothermia or induced hypothermia. CO2 production of each cell and oxygen consumption is directly proportional to body temperature. * For asystole or PEA, atropine and epinephrine can be given between CPR cycles. * Do not shock asystole. It may be important to know the most common wrong answer in situations like this because both will be listed on the exam and likely they are the two answer choices you narrow down to. Defibrillation stops the heart with the hope that it will reboot normally. Defibrillating asystole will do nothing because the electrical system is already stopped; its not like jumper cables for a car, its more like rebooting an electronic device when it is not functioning well. Side note: always confirm asystole in a second lead. * Say youve given epinephrine and atropine through several cycles of CPR and the patient is still dead. You can consider a pacemaker. For true asystole, this does nothing. You consider pacemaker because it could possibly be very slow bradycardia and you may have missed a beat. Algorithm says consider because beyond that you are considering burial or cremation, there is nothing else to do, the patient is dead anyway. Electrocardiograms * There are very few ECGs on USMLE Step 2, maybe two or three.

* Each small box is 40ms. Each large box is 200ms. * PR interval should be less than 5 small boxes (200ms). QRS should be less than 3 small boxes (120ms). * Counting rate is done by the 300 method. Count the number of large boxes between QRS complexes and divide 300 by that number. So if there were 5 large boxes between QRS complexes it would be 300/5 = 60bpm. * 300 method points are 300, 150, 100, 75, 60, 50. You can differentiate tachycardia from bradycardia this way. * Arrhythmia associated with digoxin toxicity is bigeminy, trigeminy, any arrhythmia. The most common arrhythmia seen in digoxin toxicity is SVT with variable block. * Bigeminy is every other beat being a PVC. Couplets are two PVCs, not uncommon. * Supraventricular can be differentiated from ventricular by the width of the QRS complexes. Wide complexes always comes from the ventricles. Wide complexes are > 120ms. Supraventricular impulses follow the normal conduction pathway through the AV node, which makes them narrow on the QRS. * Ectopic beats originate from an ectopic focus. One is PVC. Two is couplet. Three is triplet. Four or more is ventricular tachycardia. * ECG showing flat line. Dont think something like the leads came off the patients chest. This is asystole. There is some biological variability, so it might not be perfectly flat line. What is the best next step in management? CPR, then epi and atropine for a few rounds, then consider a pacemaker, then consider the patient dead. * You are at the bedside for a patient with chest pain. They become unconscious and the ECG shows flat line. Should you do a precordial thump for the witnessed arrest? No. Precordial thump is like defibrillation and you do not defibrillate flat line. * Consistent wide complexes is ventricular tachycardia. What do you do next if the patient is conscious and stable? Give lidocaine. * What if ECG shows Vtach and patient is unconscious? Defibrillation. * Know what ventricular fibrillation looks like and do not expect Torsades. Irregular waves with no consistency across the rhythm strip. Patient is always pulseless. What do you do? Defibrillate. * ECG shows Vfib and youve gone through cycles of CPR with defibrillation. Now you intubated the patient and give epinephrine during the cycles. How much time do you have to save this patient? 10 minutes. What medication can you give for refractory VT or VF? Amiodarone.

* Important Note: There is no minimum and no maximum for running a resuscitation. * You get called into a room by a nurse because the monitor shows the patients rate at 140 per minute. She has the defibrillator ready and is asking you what to do. You feel a pulse and note that it is 70 per minute. The patient states - 101 she missed her dialysis treatment this week. What could it be? Hyperkalemia. The peaked T-waves are so high that the monitor is reading them as QRS complexes. * When talking about peaked T-waves, the size does not matter as much as the shape. The pointier the worse. * The most important ECG to know is regular sinus rhythm. * Keep in mind when you are studying for the exam, if your highest point to attain is the exam it will feel painful. If you are conscious of the goodness that comes from the knowledge after that and what it can do to be of service, then the exam will feel smaller and your process will be filled with more joy. ------------------------------------------------------------------------------------------------------------------------------------------

También podría gustarte